You are on page 1of 209

COMMON LAW ADMISSION TEST -2020-PG

―9. We now come to the Division Bench judgment of this Court reported as Rajeev Kumar
Gupta & Others v. Union of India & Others – (2016) 13 SCC 153. In this judgment, the posts
in Prasar Bharati were classified into four Groups–A to D. The precise question that arose
before the Court is set out in para 5 thereof in which it is stated that the statutory benefit of 3
per cent reservation in favour of those who are disabled is denied insofar as identified posts in
Groups A and B are concerned, since these posts are to be filled through direct recruitment.
After noticing the arguments based on the nine-Judge bench in Indra Sawhney vs. Union of
India, 1992 Supp (3) SCC 217, this Court held:
14. We now examine the applicability of the prohibition on reservation in promotions as
propounded by Indra Sawhney. Prior to Indra Sawhney, reservation in promotions were
permitted under law as interpreted by this Court in Southern Railway v. Rangachari, AIR
1962 SC 36. Indra Sawhney specifically overruled Rangachari to the extent that reservations
in promotions were held in Rangachari to be permitted under Article 16(4) of the
Constitution. Indra Sawhney specifically addressed the question whether reservations could
be permitted in matters of promotion under Article 16(4). The majority held that reservations
in promotion are not permitted under our constitutional scheme.
15. The respondent argued that the answer to Question 7 in Indra Sawhney squarely covers
the situation on hand and the reasons outlined by the majority opinion in Indra Sawhney at…
must also apply to bar reservation in promotions to identified posts of Group A and Group B.
16. We do not agree with the respondent‟s submission. Indra Sawhney ruling arose in the
context of reservations in favour of backward classes of citizens falling within the sweep of
Article 16(4).
21. The principle laid down in Indra Sawhney is applicable only when the State seeks to give
preferential treatment in the matter of employment under the State to certain classes of
citizens identified to be a backward class. Article 16(4) does not disable the State from
providing differential treatment (reservations) to other classes of citizens under Article 16(1)
if they otherwise deserve such treatment. However, for creating such preferential treatment
under law, consistent with the mandate of Article 16(1), the State cannot choose any one of
the factors such as caste, religion, etc. mentioned in Article 16(1) as the basis. The basis for
providing reservation for PWD is physical disability and not any of the criteria forbidden
under Article 16(1). Therefore, the rule of no reservation in promotions as laid down in Indra
Sawhney has clearly and normatively no application to PWD.‖
Source: Excerpt taken from a Judgment of three judge bench comprising of R.F. Nariman,
Aniruddha Bose & V. Ramasubramaniyam., JJ.
1. The above passage has been taken from which of the following recent judgments, relating
to the question of reservation in promotions for the disabled persons?
a) National Federation of the Blind v. Sanjay Kothari, Secy. Deptt. of Personnel and
Training.
b) Siddaraju v. State of Karnataka & Ors.
c) Rajeev Kumar Gupta & Ors. v. Union of India & Ors.
d) Ashok Kumar v. Union of India & Ors.
CORRECT ANSWER : OPTION B

Page 1 of 35
2. Which of the following is true in context of the scheme provided under Article 16 of the
Indian Constitution, relating to reservation in promotion?
a) Reservation in promotion can only be granted to the class of citizens mentioned under
Article 16 (4).
b) Reservation in promotion cannot be granted to a class of citizen provided by the virtue of
Article 16 (1).
c) The scheme of reservation in promotion can be extended to any class of citizens under the
scheme of Article 16 (1).
d) Reservation in promotion defeats the scheme of Article 16 (1) and Article 15 (1).
CORRECT ANSWER : OPTION C

3. The Union government has issued an office memorandum under which 3% reservation
has been provided to the persons with disability, apart from the reservations provided to
different class of citizens such as 27% for OBCs, 14% to SCs and & 7% to STs. Now, the
total percentage of reservation has reached 51%, which is against the judgment given in
Indira Sawhney v. Union of India. Now, choose the most appropriate option amongst the
following.
a) The reservation provided to persons with disability is constitutionally valid as it falls
within the horizontal scheme of reservation.
b) The judgment in Indira Sawhney is not applicable to the persons with disability and hence
such reservation is valid.
c) The reservation provided to persons with disability is invalid as in no case reservation can
increase 50%.
d) The reservation to PWD does not fall under the scheme of Article 16 (4) and hence
unconstitutional.
CORRECT ANSWER : OPTION A

4. What is the meaning of the ―Catch-up‖ rule associated with the matters of seniority in
reservation in promotion?
a) If the junior candidate promoted on the basis of reservation gets promoted to further grade
by the time senior general category candidate is promoted to earlier grade, the question of
seniority does not arise.
b) A reserved category candidate promoted on the basis of reservation earlier than his senior
general category candidates in the feeder category, shall become junior when general
category senior candidate too gets promoted.
c) The candidate promoted to higher grade on the basis of reservation remains senior even if
his senior is promoted to the same grade.
d) None of the above.
CORRECT ANSWER : OPTION B

Page 2 of 35
5. The Article 16 (4A), provides for which of the following?
a) Catch-up rule.
b) Carry forward rule.
c) Consequential seniority.
d) All of the above.
CORRECT ANSWER : OPTION C

6. The scheme of reservation in promotion is limited to which of the following as per the
text of Article 16 (4A)?
a) Schedule Castes and Schedule Tribes.
b) Backward class of citizens.
c) PWD candidates.
d) All of the above.
CORRECT ANSWER : OPTION A

7. Government policy of no reservation in promotion for class I and II posts was initially:
a) Struck down in C.A. Rajendra case.
b) Struck down in M. Nagraj case.
c) Upheld in Jarnail Singh case.
d) Upheld in C.A. Rajendra case.
CORRECT ANSWER : OPTION D

8. Jarnail Singh case overruled the M. Nagraj on the issue of:


a) Collection of quantifiable data to determine inadequacy of representation of SCs and STs.
b) Collection of quantifiable data to determine the backwardness.
c) Collection of data on efficiency of administration.
d) All the above.
CORRECT ANSWER : OPTION B

9. Creamy layer concept is applicable to


a) All reservations
b) SC ST reservations
c) OBC reservation
d) Only horizontal reservation
CORRECT ANSWER : OPTION C

Page 3 of 35
10. In Vivekanand Tiwari, Supreme Court held that the unit for reservation in universities
should be:
a) University as a whole
b) Faculties of the University
c) Departments of the University
d) (a) and (b)
CORRECT ANSWER : OPTION C

It will be relevant to refer to the statement made by the contemnor which was made and read
out before this Court by the contemnor on 20.08.2020, which reads as under:
―I have gone through the judgment of this Hon‟ble Court. I am pained that I have
been held guilty of committing contempt of the Court whose majesty I have tried to uphold -
not as a courtier or cheerleader but as a humble guard - for over three decades, at some
personal and professional cost. I am pained, not because I may be punished, but because I
have been grossly misunderstood. I am shocked that the court holds me guilty of
“malicious, scurrilous, calculated attack” on the institution of administration of justice. I am
dismayed that the Court has arrived at this conclusion without providing any evidence of my
motives to launch such an attack. I must confess that I am disappointed that the court did not
find it necessary to serve me with a copy of the complaint on the basis of which the suo-motu
notice was issued, nor found it necessary to respond to the specific averments made by me in
my reply affidavit or the many submissions of my counsel. I find it hard to believe that the
Court finds my tweet “has the effect of destabilizing the very foundation of this
important pillar of Indian democracy”. I can only reiterate that these two tweets
represented my bona-fide beliefs, the expression of which must be permissible in any
democracy. Indeed, public scrutiny is desirable for healthy functioning of judiciary itself. I
believe that open criticism of any institution is necessary in a democracy, to safeguard the
constitutional order. We are living through that moment in our history when higher
principles must trump routine obligations, when saving the constitutional order must
come before personal and professional niceties, when considerations of the present must
not come in the way of discharging our responsibility towards the future. Failing to speak up
would have been a dereliction of duty, especially for an officer of the court like myself. My
tweets were nothing but a small attempt to discharge what I considered to be my highest duty
at this juncture in the history of our republic. I did not tweet in a fit of absence mindedness. It
would be insincere and contemptuous on my part to offer an apology for the tweets that
expressed what was and continues to be my bona-fide belief. Therefore, I can only humbly
paraphrase what the father of the nation Mahatma Gandhi had said in his trial: I do not ask
for mercy. I do not appeal to magnanimity. I am here, therefore, to cheerfully submit to
any penalty that can lawfully be inflicted upon me for what the Court has determined to be an
offence, and what appears to me to be the highest duty of a citizen.‖

Source: Excerpt taken from the Judgment delivered by Arun Mishra, B. R. Gavai & Krishna
Murari, J.J.

Page 4 of 35
11. The above passage has been taken from which of the following recent cases relating to
the Criminal Contempt of Court?
a) In Re: Prashant Bhushan & Anr.
b) The Registrar General, Supreme Court of India v. Prashant Bhushan & Anr.
c) Amicus Curiae v. Prashant Bhushan
d) Union of India v. Prashant Bhushan & Anr.
CORRECT ANSWER : OPTION A

12. The Source of power of the Supreme Court to take suo-motu cognizance of Contempt of
the Court has been provided under which of the following?
a) Section 15 of the Contempt of Courts Act, 1971.
b) Article 129 r/w Section 13 of the Contempt of Courts Act, 1971.
c) Article 129
d) Article 129 r/w Article 141.
CORRECT ANSWER : OPTION C

13. Which of the following could be a valid defence for the contemnor in a contempt
proceeding against him?
a) Statements are bona-fide fair criticism without attributing motives to the judges.
b) Statements are the personal opinion of the person and do not have the capacity to
influence the thinking of public at large.
c) Statements are based on the quotes from retired judges of the Supreme Court.
d) Statements are mere opinions which does not fall under the category of the term
―scandalising the court.‖
CORRECT ANSWER : OPTION A

14. In which of the following cases, the apex court held that, ―Contempt jurisdiction should
not be used by judges to uphold their own dignity. In the free market-place of ideas,
criticism about the judicial system or the judges should be welcomed, so long as
criticisms do not impair or hamper the „administration of justice‘.‖?
a) Amicus Curiae v. Prashant Bhushan
b) P.N. Duda v. V. P. Shivshankar
c) A.K. Gopalan v. Noordeen
d) Hari Singh Nagra v. Kapil Sibal
CORRECT ANSWER : OPTION B

Page 5 of 35
15. Which of the following can be stated as not true about the intent of the contemnor as
mentioned in the passage above?
a) He believes in the dignity and independence of judiciary and his act, further strengthens
his belief.
b) His statements hold the sanctity of the institution to be of utmost importance and his
actions will uphold the same.
c) He compares himself with the father of the nation Mahatma Gandhi and puts himself at
the same pedestal.
d) His statements are criticism of an individual and not the institution itself and such
criticism is quintessential for a healthy democracy.
CORRECT ANSWER : OPTION C

16. A comparison of the Freedom of Speech and Expression between the text of Constitution
of India and the U.S. Constitution may lead to many conclusions.
Which of the following is not a conclusion of such a comparison?
a) The U.S. Constitution expressly mentions about the Freedom of Press but does not
mention about the Freedom of Expression.
b) The Freedom of Press though not expressly mentioned under Article 19 (1) (a), it is
implicit under the Freedom of Speech.
c) The idea of Freedom of Speech and Expression is much broader in India as compared to
that in the U.S. Constitution.
d) The Freedom of Speech and Expression under both the constitutions is identical in terms
of its extent.
CORRECT ANSWER : OPTION C

17. Which of the following case is not related to the Contempt of Court as a restriction to the
Freedom of Speech and Expression enshrined under Article 19 (1) (a)?
a) In Re Arundhati Roy, (2002) 3 SCC 343.
b) Hari Singh Nagra v. Kapil Sibal, AIR 2010 SC 55.
c) In Re Harijai Singh, (1996) 6 SCC 466.
d) Subramaniam Swamy v. UOI, (2016) 7 SCC 221.
CORRECT ANSWER : OPTION D

18. In which of the following cases it was held that holding Dharna in front of Supreme Court
in which lawyers too, take part is not by itself Contempt of Court if the access to the
Court is not hindered?
a) Hiralal Dixit v. Union of India
b) J.R. Parashar v. Prashant Bhushan
c) Tarun Bharat Singh v. Union of India
d) P. N. Duda v. V. P. Shivshankar
CORRECT ANSWER : OPTION B

Page 6 of 35
19. Justice Krishna Iyer in (1) observed that normative guideline for Judges to observe in
contempt jurisdiction is not to be (2) even where distortions and criticism oversteps the
limitation.
a) (1) S. Mulgaokar, (2) hypersensitive
b) (1) Shamsher Singh, (2) provocative
c) (1) Hira Lal, (2) emotional
d) (1) Ediga Annama, (2) sensitive
CORRECT ANSWER : OPTION A

20. Late Arun Jaitely, in Parliament had said that the Supreme Court is destroying the edifice
of Parliament brick by brick. Another member responded by saying transparency in
judicial appointments is required as half the judges in the country lack integrity. Are these
statements Contempt of Court after the Prashant Bhushan 2020 judgment?
a) Yes, because MPs are also bound by Contempt law
b) No, because MPs are exempted from Contempt law.
c) Jaitely can‘t be punished as he is no more but the other member can be held liable.
d) No, because the statements made in Parliament are protected.
CORRECT ANSWER : OPTION D

In taking this view, Justice Rajagopala Ayyangar, speaking for a majority of five judges,
relied upon the judgment of Justice Frankfurter, speaking for the US Supreme Court in Wolf
v Colorado, which held:
―The security of one's privacy against arbitrary intrusion by the police … is basic to a free
society… We have no hesitation in saying that were a State affirmatively to sanction such
police incursion into privacy it would run counter to the guarantee of the Fourteenth
Amendment.‖
While the Court observed that the Indian Constitution does not contain a guarantee similar to
the Fourth Amendment of the US Constitution, it proceeded to hold that:
―Nevertheless, these extracts would show that an unauthorised intrusion into a person‘s home
and the disturbance caused to him thereby, is as it were the violation of a common law right
of a man an ultimate essential of ordered liberty, if not of the very concept of civilisation. An
English Common Law maxim asserts that ―every man‘s house is his castle‖ and in Semayne
case [5 Coke 91: 1 Sm LC (13th Edn) 104 at p. 105] where this was applied, it was stated that
―the house of everyone is to him as his castle and fortress as well as for his defence against
injury and violence as for his repose‖. We are not unmindful of the fact that Semayne case
[(1604) 5 Coke 91: 1 Sm LC (13th Edn) 104 at p. 105] was concerned with the law relating to
executions in England, but the passage extracted has a validity quite apart from the context of
the particular decision. It embodies an abiding principle which transcends mere protection of
property rights and expounds a concept of ―personal liberty‖ which does not rest on any
element of feudalism or on any theory of freedom which has ceased to be of value.‖
Source: Excerpt taken from the Judgment delivered by a 9 Judge bench of the Supreme Court
in 2017 and authored by Dr. D. Y. Chandrachud. J.

Page 7 of 35
21. The above passage is from which of the following judgments?
a) Justice K. S. Puttaswamy (Retd.) v. Union of India, (2018) 10 SCC 1.
b) Justice K. S. Puttaswamy (Retd.) v. Union of India, (2017) 1 SCC 10.
c) Justice K. S. Puttaswamy (Retd.) v. Union of India, (2017) 10 SCC 1.
d) Justice K. S. Puttaswamy (Retd.) v. Union of India, (2018) 1 SCC 10.
CORRECT ANSWER : OPTION C

22. Which of the following is directly related with the central idea of the passage mentioned
above?
a) Right to Privacy of an individual, being part of the Right to Life.
b) The extent and scope of the ‗ordered liberty‘ as a tenet of liberty under Article 21 of the
Constitution.
c) Right to Life of an individual apart from mere animal existence.
d) All of the above
CORRECT ANSWER : OPTION D

23. The above passage mentions, ―Every man‟s house is his castle.‖ Who amongst the
following has stated this quote in the Semayne‘s Case?
a) Justice Rowland.
b) Justice Holmes.
c) Lord Justice A.W. Semens.
d) Justice Blackburn.
CORRECT ANSWER : OPTION B

24. The above passage makes a mention of the judgment delivered by Justice Rajagopala
Ayyangar. Which one of the following judgments is in context here?
a) Kharak Singh v. State of U.P.
b) M.P. Sharma v. Satish Chandra
c) Maneka Gandhi v. Union of India
d) Rustom Cavasji Cooper v. Union of India
CORRECT ANSWER : OPTION A

25. Which of the following is not an interpretation of the Right to Privacy as explained by the
Supreme Court in Puttaswamy judgment?
a) The destruction by the State of a sanctified personal space, of body and mind is violative
of the guarantee against arbitrary state action.
b) The intersection between one‘s mental integrity and privacy entitles the individual to the
freedom of self-determination.
c) The privacy of an individual recognises an inviolable right to determine how freedom
shall be exercised.
d) The guarantee of privacy is a guarantee against the arbitrary State action.
CORRECT ANSWER : OPTION C

Page 8 of 35
26. Which of the following is true in relation to the scope of the newly evolved ‗Right to
Privacy‘?
a) The Right to Privacy cannot be denied, even if there is a miniscule fraction of the
population which is affected.
b) The majoritarian concept applies to the Constitutional Rights and the Courts must adhere
to the majoritarian view.
c) One‘s sexual orientation is undoubtedly not an attribute of privacy.
d) Right to Privacy is an unrestricted and inviolable right, outside the fetters of any State
action.
CORRECT ANSWER : OPTION A

27. In which of the following cases the Supreme Court held that, “Sexual orientation is an
attribute of privacy. Discrimination against an individual on the basis of sexual
orientation is deeply offensive to the dignity and self-worth of the individual. Equality
demands that the sexual orientation of each individual in society must be protected on an
even platform. The right to privacy and the protection of sexual orientation lie at the core
of the fundamental rights guaranteed by Article 14, 15 and 21 of the Constitution.”?
a) Justice K.S. Puttaswamy (Retd.) v. Union of India.
b) Navtej Singh Johar v. Union of India.
c) NALSA v. Union of India.
d) Suresh Kumar Koushal v. Naz Fundation.
CORRECT ANSWER : OPTION A

28. Speaking for four of the nine judges, Justice D.Y. Chandrachud, observes, “Individually,
these information silos may seem inconsequential. In aggregation, they disclose the
nature of the personality; food habits, language, health, hobbies, sexual preferences,
friendships, ways of dress and political affiliation.” Which of the following is in context
of the above statement?
a) Right to Privacy.
b) Informational/Data Privacy.
c) The Aadhaar Act, 2016.
d) None of the above.
CORRECT ANSWER : OPTION B

29. Which of the following is not a tenet of the term ‗Life‘ under Article 21 of the
Constitution?
a) Right to Die with Dignity
b) Right to Live with Dignity
c) Freedom of Sexual Orientation
d) Right to Reputation
CORRECT ANSWER : OPTION D

Page 9 of 35
30. Recently, it has been reported that in Uttar Pradesh more than 50% of the people booked
under the National Security Act were involved in cow slaughter. In Puttaswamy, which of
the following judges has included food preferences in his judgment?
a) Justice D.Y. Chandrachud
b) Justice Dipak Misra
c) Justice R. F. Nariman
d) Justice Jasti Chelameshwar
CORRECT ANSWER : OPTION D

The requirement of balancing various considerations brings us to the principle of


proportionality. In the case of K. S. Puttaswamy (Privacy-9J.) (supra), this Court observed:
“310…Proportionality is an essential facet of the guarantee against arbitrary State
action because it ensures that the nature and quality of the encroachment on the right
is not disproportionate to the purpose of the law...” Further, in the case of CPIO v.
Subhash Chandra Aggarwal, [(2019) SCC OnLine SC 1459], the meaning of proportionality
was explained as: ―225. It is also crucial for the standard of proportionality to be
applied to ensure that neither right is restricted to a greater extent than necessary to fulfil
the legitimate interest of the countervailing interest in question…”
The proportionality principle can be easily summarized by Lord Diplock‘s aphorism ‗you
must not use a steam hammer to crack a nut, if a nutcracker would do?‘ [Refer to R v.
Goldsmith, [1983] 1 WLR 151, 155 (Diplock J)]. In other words, proportionality is all about
means and ends. The suitability of proportionality analysis under Part III, needs to be
observed herein. The nature of fundamental rights has been extensively commented upon.
One view is that the fundamental rights apply as ‗rules‘, wherein they apply in an ‗all¬ or
¬nothing fashion‘. This view is furthered by Ronald Dworkin, who argued in his theory that
concept of a right implies its ability to trump over a public good. Dworkin‘s view necessarily
means that the rights themselves are the end, which cannot be derogated as they represent the
highest norm under the Constitution. This would imply that if the legislature or
executive act in a particular manner, in derogation of the right, with an object of
achieving public good, they shall be prohibited from doing so if the aforesaid
action requires restriction of a right. However, while such an approach is often taken by
American Courts, the same may not be completely suitable in the Indian context, having
regard to the structure of Part III which comes with inbuilt restrictions.
Source: Excerpt taken from a judgment delivered by the bench of N. V. Ramanna, R. Subhash
Reddy, B.R. Gavai, J.J. on 10th January, 2020.
31. The above passage has been taken from which of the following judgments, which decided
the state of affairs relating to internet ban in Jammu & Kashmir?
a) Sita Ram Yechury v. Union of India
b) Anuradha Bhasin v. Union of India
c) In Re: State of Jammu & Kashmir
d) Bhim Singh v. State of Jammu & Kashmir
CORRECT ANSWER : OPTION B

Page 10 of 35
32. The above passage discusses about the proportionality, in the context of which of the
following issues?
a) The limitations of state action sanctioned under the Constitution.
b) The powers of the President under Article 370 of the Constitution.
c) The extent of restrictions to be imposed by the State on the exercise of Fundamental
Rights.
d) The proportionality of powers and duties under the Constitution.
CORRECT ANSWER : OPTION C

33. In the above passage, the Court expresses the limitations on application of ‗All-or-
nothing‘ approach to the fundamental rights. Which of the following statements truly
explains such limitations?
a) The application of Part-III is subject to the interest of the majority and it overrides the
rights of an individual.
b) The state may act in derogation of the Fundamental Rights of the people to achieve a
higher public good and the social equilibrium.
c) The Fundamental Rights of citizens are secondary and the authority of State over its
citizens is primary and of utmost importance.
d) The above approach is suitable only for the American Constitution as no limitation on
rights is mentioned in the text of the Constitution.
CORRECT ANSWER : OPTION B

34. In which of the following cases the Supreme Court held that Right to Access Internet is
protected under Article 19 of the Indian Constitution?
a) Anuradha Bhasin v. Union of India.
b) Irtiqa Iqbal v. Union of India
c) Kapil Sibal v. Union of India.
d) None of the above.
CORRECT ANSWER : OPTION A

35. Which of the following State actions passes the ‗Proportionality Test‘?
a) A complete internet ban for indefinite period in a state effected by terrorism, insurgency
and local militia.
b) A slowdown of the internet speed in a State, affecting the Right of Free speech and
expression and trade, business and occupation.
c) House arrest of eminent political leaders for an indefinite period.
d) A complete lockdown in an area effected by militant attacks for an indefinite period.
CORRECT ANSWER : OPTION B

Page 11 of 35
36. Which of the following is not an essential, validating the restrictions imposed upon the
exercise of Fundamental Rights?
a) Legality, which postulates the existence of law which is enacted to restrict the
application of Fundamental Rights.
b) Need, defined in terms of a legitimate State aim.
c) Proportionality, which ensures a rational nexus between the
objects and the means adopted to achieve them.
d) Consequence, which the restriction would ensue after its application.
CORRECT ANSWER : OPTION D

37. The Right to Access to the internet is useful for exercising which of the following
Fundamental Rights?
a) Freedom of Speech and Expression.
b) Right to carry out trade, business and occupation
c) Right to Life.
d) All of the above.
CORRECT ANSWER : OPTION D

38. Who amongst the following can order for suspension of telecom services in a state under
the Temporary Suspension of Telecom Services (Public Emergency or Public Safety)
Rules, 2017?
a) The Secretary to the Government of India in the Ministry of Home Affairs.
b) The Secretary to the Government of India in the Ministry of Information and
Broadcasting.
c) A District Magistrate, exercising his powers under S.144 of the Cr. P. C.
d) The Chief Secretary to the Government of State in the Ministry of Home Affairs.
CORRECT ANSWER : OPTION A

39. Which of the following was true prior to 5th August, 2019 in relation to power of the
Parliament under Article 3 of the Constitution for the State of Jammu & Kashmir?
a) Any Bill to alter the boundaries of the State shall be introduced in the Parliament after the
recommendation of the Governor of the State.
b) No Bill can be introduced in the Parliament for the alteration of the boundaries of the
State without the consent of the Legislature of the State.
c) No Bill can be introduced in the Parliament for the alteration of the boundaries of the
State without the consent of the Constituent Assembly of the State.
d) No Bill can be introduced in the Parliament for the alteration of the boundaries of the
State without the consent of the President.
CORRECT ANSWER : OPTION B

Page 12 of 35
40. Which of the following is not true in relation to the Jammu and Kashmir Reorganisation
Act, 2019?
a) The Act has amended Schedule 1 of the Constitution
b) The Act has amended Article 4 of the Constitution.
c) The Act has amended Schedule 4 of the Constitution.
d) None of the above.
CORRECT ANSWER : OPTION B

―The main argument on behalf of the Respondents was that the Government was bound by its
promise and could not have resiled from it. They had an indefeasible legitimate expectation
of continued employment, stemming from the Government Order dated 20.02.2002 which
could not have been withdrawn. It was further submitted on behalf of the Respondents that
they were not given an opportunity before the benefit that was promised, was taken away. To
appreciate this contention of the Respondents, it is necessary to understand the concept of
legitimate expectation.

14. The principle of legitimate expectation has been recognized by this Court in ―Union of
India v. Hindustan Development Corporation & Ors.‖ If the promise made by an authority is
clear, unequivocal and unambiguous, a person can claim that the authority in all fairness
should not act contrary to the promise.
15. M. Jagannadha Rao, J. elaborately elucidated on legitimate expectation in ―Punjab
Communications Ltd. v. Union of India & Ors.‖ He referred to the judgment in 2 (1993) 3
SCC 499 ―Council of Civil Service Unions and Ors. v. Minister for the Civil Service‖ in
which Lord Diplock had observed that for a legitimate expectation to arise, the decisions of
the administrative authority must affect the person by depriving him of some benefit or
advantage which, (i) he had in the past been permitted by the decision- maker to enjoy and
which he can legitimately expect to be permitted to continue to do until there has been
communicated to him some rational grounds for withdrawing it on which he has been given
an opportunity to comment; or (ii) he has received assurance from the decision-maker that
they will not be withdrawn without giving him first an opportunity of advancing reasons for
contending that they should not be withdrawn.
Rao, J. observed in this case, that the procedural part of legitimate expectation relates to a
representation that a hearing or other appropriate procedure will be afforded before the
decision is made. The substantive part of the principle is that if a representation is made that a
benefit of a substantive nature will be granted or if the person is already in receipt of the
benefit, that it will be continued and not be substantially varied, then the same could be
enforced.
16. It has been held by R. V. Raveendran, J. in ―Ram Pravesh Singh v. State of Bihar‖ that
legitimate expectation is not a legal right. Not being a right, it is not enforceable as such. It
may entitle an expectant:
(a) to an opportunity to show cause before the expectation is dashed; or
(b) to an explanation as to the cause of denial. In appropriate cases, the Courts may grant a
direction requiring the authority to follow the promised procedure or established practice.‖

Page 13 of 35
Source: Excerpt taken from the judgment delivered by the bench of L. Nageshwar Rao &
Hemant Gupta, J.J. in Kerala State Beverages (M and M) v. P P Suresh & Ors.,(2019) 9 SCC
710.
41. Which of the following statements cannot be identified as a limitation upon the Doctrine
of Legitimate Expectation?
a) The concept of Legitimate Expectation is only procedural and has no substantive impact.
b) The doctrine does not apply to legislative activities.
c) The doctrine does not apply if it is contrary to Public Policy or against the Security of
State.
d) There are parallels between the Doctrine of Legitimate Expectation and Promissory
Estoppel.
CORRECT ANSWER : OPTION A

42. Which of the following is not true in relation to the Doctrine of Legitimate Expectation as
observed by the Supreme Court in ‗Monnet Ispat & Energy Ltd. v. Union of India‟?
a) The Doctrine of Legitimate Expectation cannot be invoked as a substantive and
enforceable right.
b) The Legitimate Expectation is different from anticipation and an anticipation cannot
amount to an assertable expectation.
c) The Doctrine fails where an overriding public interest justifies the change in
Administrative Policy.
d) The Doctrine of Legitimate Expectation is founded on the principles of reasonableness
and fairness.
CORRECT ANSWER : OPTION A

43. Which of the following cases can be traced as the origin of the Doctrine of Legitimate
Expectation?
a) Attorney General of Hong Kong v. Ng Yeun Shiu, (1983) 2 AC 629.
b) Schmidt v. Secy. Of State for Home Affairs, (1969) 2 Ch 149 (CA).
c) Food Corporation of India v. Kamdhenu Cattle Feed Industries, AIR 1993 SC 1601.
d) Breen v. Amalgamated Engg. Union, (1971) 2 WLR 742.
CORRECT ANSWER : OPTION B

44. Which of the following is not a ground for judicial review of a discretionary action of an
Administrative Authority in India?
a) Failure to exercise discretion
b) Excess or abuse of discretion
c) A breach of rules of Natural Justice
d) None of the above
CORRECT ANSWER : OPTION A

Page 14 of 35
45. Which of the following statement is true in relation to ―Empty formality‖ theory of the
Principle of Natural Justice?
a) The plea for not following the Principle of Natural Justice is not sustainable on the
grounds of Empty Formality.
b) The plea for not following the Principle of Natural Justice is sustainable on the grounds of
Empty Formality.
c) The Empty Formality affords a legitimate ground for the avoidance of Principle of
Natural Justice.
d) Both options (b) & (c) are correct.
CORRECT ANSWER : OPTION B

46. What is the meaning of a writ of ―Certiorarified mandamus‖?


a) A writ of Mandamus, issued against an adjudicating body to quash a decision.
b) A writ of Certiorari issued to an administrative body to quash its decision.
c) A simultaneous writ to quash a decision and also to give a direction.
d) A simultaneous writ to quash a direction and give a decision.
CORRECT ANSWER : OPTION C

47. Which of the following is not a ground for holding a Delegated Legislation as invalid?
a) Parent Act delegates non-essential legislative functions.
b) Delegated legislation is inconsistent with the general law.
c) Parent Act itself is unconstitutional.
d) Delegated legislation is inconsistent with the Parent Act.
CORRECT ANSWER : OPTION A

48. In which of the following cases, Hegde J observed that, ―Whenever a complaint is made
before a court that some Principle of Natural Justice had been contravened, the court had
to decide whether the observance of that rule was necessary for a just decision on the
facts of that case.‖ ?
a) A.K. Kraipak v. Union of India
b) Maneka Gandhi v. Union of India
c) Union of India v. P.K. Roy
d) Dharampal Satyapal Ltd. v. CCE
CORRECT ANSWER : OPTION A

49. Which of the following is not an essential condition before pressing the Doctrine of
Estoppel into service or benefit contract?
a) A representation or conduct amounting to representation have been made.
b) He must have acted to his detriment or suffered as a result of such representation.
c) The other party to whom representation was made must have acted upon such
representation.
d) The representation must have been made with the intention of not fulfilling it.
CORRECT ANSWER : OPTION D

Page 15 of 35
50. Suppose, students filled up JEE Mains form for 2020. Due to repeated postponements of
JEE, IIT Kharagpur decided to opt out of JEE mains and conducted its own separate Test.
On which of the following grounds the decision of IIT Kharagpur can be challenged?
a) Procedural fairness
b) Legitimate expectation
c) Manifest arbitrariness
d) IIT Kharagpur‘s decision cannot be challenged as it was taken in the interest of the
students.
CORRECT ANSWER : OPTION B

Read the piece written by Prof. Upendra Baxi and answer the questions below:

No matter how the matters are for the time being resolved (and swiftly on all indications), the
present crisis in the Supreme Court involves mainly a contention on how judicial business
should be conducted. The extraordinary movement of four justices in making public a letter
addressed to the Chief Justice of India (CJI) in November 2017, and assorted observations at
the press conference last week are very unusual judicial happenings. At that conference,
Justice Chelameswar said that ―less than desirable things have happened‖ and the protesting
Justices vainly ―tried to collectively persuade‖ the CJI to take ―remedial measures‖. These
happenings are now made even more unusual by Justice Ranjan Gogoi reportedly denying
any ―crisis‖ and Justice Kurian Joseph saying the matter is now settled leaving little scope for
―outside intervention‖. However, the letter released at the press conference said otherwise; it
spoke of the ways in which ―the overall functioning of the justice delivery system‖, the
―independence of the high courts‖, and the functioning of the office of the CJI have been
―adversely affected‖. A moving appeal to the Indian ―Nation‖ was issued at the press
conference and Justice Chelameswar justified speaking out, lest ―wise people‖ say later that
they were complicit. A situation where four senior-most justices went public to express their
discontent with the present CJI‘s exercise of authority to constitute Benches raises grave
constitutional questions. Although only an in-house rectification can save matters, it is an
anti-democratic error of grave proportions to think that co-citizens should have no interest,
stake, or say in the matter. Undoubtedly, the Chief Justices, whether of the High courts or the
Supreme Court, have the power to order the roster. The question is whether that power is
coupled with a constitutional duty to follow certain conventions. Obviously, there are a few:
Chief Justices have a primary duty of accountability to the Brother Justices, the Bar, and a
general obligation through the Bar to the litigating public and people at large. But when a
letter by four Senior Justices has been ignored for about two months, is going public with a
copy of that letter and holding even a press conference unjudicial? On this question opinions
are varied. Some have lauded this step as heroic while others regard this as ―sheer trade union
tactics‖ and some even say the step was extremely unfortunate but now some institutional
solidarity should pave the path ahead. What are the other conventions? First, a part-heard
matter may not be divested from the co-justices who are seized with it. Second, the CJI may
not deny a request for recusal on grounds of conflict of interest. Third, the Chief Justice may
not ignore the requests by co-justices to form a larger Bench. Fourth, a Chief Justice may not
selectively assign sensitive or important cases to the same judges. However, fifth, it is
doubtful whether there is, or ought to be, a convention requiring such matters to be heard
only by the senior-most justices. No, because the decision to elevate a citizen to judgeship

Page 16 of 35
must involve all relevant considerations; once elevated, a justice is co-equal to all other
brethren. Sixth, it is true that co-equality occurs within a hierarchy: Not every justice
becomes a Chief Justice, and the SC collegium must comprise the five senior-most justices.
Outside this framework, the question about the rank-ordering may not arise; all Justices speak
for the constitutional court. Any discussion about benches headed by ―junior‖ justices is
therefore injudicious. The second issue looming large is the finalisation of Memorandum of
Procedure (MoP). In early July 2017 (in Justice Karnan‘s case), at least two Justices observed
a need ―to revisit the process of appointment of judges and establishment of a mechanism for
corrective measures other than impeachment‖. The letter also suggests that the issue of MoP
―cannot linger on for indefinite period‖ and since the government has not responded to the
MoP sent as far back as March 2017, the Court must now presume this long ―silence‖
amounts to acceptance. Convening a full court and/or an agreement of the Chief Justices‘
conference stand was suggested. The highest court in the land cannot endlessly wait for the
government.

The remedies of impeachment and removal for judicial misconduct and review, and now
curative jurisdiction, constitutionally exist. And further, the spectre of the call of conscience
to go to the ―Nation‖ will now haunt all Chief Justices. Informed criticism has some impact
on judicial dispositions. But the ultimate guarantee of fairness as justice lie with the Justices
themselves. As Eugene Ehrlich, a founder of European sociology of law, said: ―The best
guarantee of justice lies in the personality of the judge.‖ Justices must be seen practising what
they preach to the other holders of public power. It is only when they collectively fail to do so
that a democracy is truly imperilled.

51. Consider the following statements:


1. CJI is the boss of High Court Judges
2. CJI is superior to other Judges of the Supreme Court
3. CJI and other four members of Collegium for appointment of Judges in Supreme Court are
equal
4. Chief Justice is one amongst equals and vested with many administrative powers.

Which of the statement given above is / are correct?


a) 1 and 3 only
b) 2 and 3 only
c) 3 only
d) 1, 2, and 4 only

CORRECT ANSWER : OPTION C

52. Consider the following statements:


1. A Bench of the Supreme Court must follow a decision delivered by a Bench of a larger or
even equal strength.
2. In case of inability to agree, the only option available is to refer the matter to the CJI,
requesting that a larger Bench be constituted to resolve the conflict.
3. This basic principle was laid down by Supreme Court in Central Board of Dawoodi Bohra
Community vs State Of Maharashtra & Anr (December 17, 2004).

Page 17 of 35
Which of the statement given above is / are correct?
a) 1 and 3 only
b) 1 and 2 only
c) 3 only
d) 1, 2 and 3
CORRECT ANSWER : OPTION D

53. Consider the following statements:


1. The Constitution does not make CJI the ―Master of Roster‖
2. The Supreme Court Rules vests in CJI the power of the ―Master of Roster‖
3. The Constitution of India read with Supreme Court Rules vests in CJI the Power of the
―Master of Rolls‖
4. The Power is neither given by the Constitution not by the Supreme Court Rules. It‘s just a
convention.

Which of the statement given above is / are correct?


a) 1 and 3 only
b) 2 only
c) 3 and 4 only
d) 1, 3 only
CORRECT ANSWER : OPTION B

54. Consider the following statements:


1) A judge of Supreme Court can be removed from his office by the Parliament.
2) A judge of Supreme Court can be impeached from his office by the President on the
recommendation of Chief Justice of India.
3) The removal of Supreme Court judge is based on two grounds - proved misbehaviour or
incapacity to act.

Which of the statement given above is / are correct?


a) 1 and 2 only
b) 2 and 3 only
c) 3 only
d) 1, 2 and 3
CORRECT ANSWER : OPTION C

55. Accountability makes the exercise of power more efficient and effective. The ancient
Greek historian Herodotus said: "The Greeks though free [were] not absolutely free; they
[had] a master called the law." Which of the following statement correctly describes the law?

a) CJI as Master of Rolls is not bound by any law.


b) CJI is bound by the conventions mentioned in the passage above.
c) CJI in his administrative capacity is bound by law.
d) CJI as Master of Roster must act fairly, justly and in non-arbitrary manner.
CORRECT ANSWER : OPTION D

Page 18 of 35
56. Supreme Court Rules framed under Article 145 of the Constitution provide CJI as the
Master of Rolls. These rules-
a) Cannot be challenged as per Justice Dinakaran Judgment
b) Can be challenged before the President of India who is the appointing authority of CJI
and other Judges
c) Rules made by the Court violative of Fundamental Rights may be struck down as ultra
vires the Constitution as per Prem Chand Garg (1963) judgment of the Supreme Court.
d) Supreme Court is supreme and no authority can question it.
CORRECT ANSWER : OPTION C

57. In S. P. Gupta v. Union of India, it was held that the word ‗Consultation‘ means:
a) Discussion
b) Ascertainment of opinion
c) Concurrence
d) Advice
CORRECT ANSWER : OPTION B

58. Per Incuriam means:


a) Judgment given against law
b) Judgment given contrary to people‘s conscience
c) Judgment given contrary to natural law
d) All of the above
CORRECT ANSWER : OPTION A

59. In which of the following cases, Supreme Court held that ―Chief Justice is an institution
himself‖?
a) Kamini Jaiswal v. Union of India
b) Asok Pande v. Union of India
c) S.P. Gupta v. Union of India
d) Prashant Bhushan v. Union of India
CORRECT ANSWER : OPTION B

60. In which of the following cases, Chief Justice was held to be as ‗Master of Roster‘, who
alone has prerogative to constitute bench?
a) Prakash Chandra v. Union of India (UOI) through Secretary to the Government of India
b) S P Gupta v. Union of India
c) Third Judges Case
d) Justice C. S. Karnan v. The Hon‘ble Supreme Court of India
CORRECT ANSWER : OPTION A

Page 19 of 35
Read the excerpts from an opinion piece entitled “Labour Law Suspension: Hit The
Workers When They Are Down” by Pranab Bardhan, Professor of Graduate School at
the Department of Economics at the University of California, Berkeley, published by
Bloomberg Quint and answer the questions below:
It is interesting that while Indian states are trying to suspend labour protection and make it
easier for employers to sack workers, many other countries are trying to minimise lay-offs in
this period of crisis by giving wage subsidy to employers to induce them to keep the workers
on the payroll. These programs are an effort to reduce displacement, distress, and loss of
worker morale, and at the time of economic recovery less friction and de-skilling. The wage
subsidies are quite substantial in Europe, Canada, Australia, and New Zealand. It is also
being attempted in some developing countries like Argentina, Bangladesh, Botswana, China,
Malaysia, Philippines, South Africa, Thailand, and Turkey.

In the continuing sordid saga of callousness and brutality with the millions of suddenly
unemployed migrant workers over the last six weeks since lockdown, an interesting fact to
note is that employers who mostly had stopped paying them over this period, thus causing
widespread hunger and homelessness, have lobbied with state governments to stop sending
them back to their villages so that they remain available when the industries restart. I am
actually in favour of a thorough overhaul. The current labour laws, tangled and outdated as
they are, serve the long-term interests of neither the employers nor the workers. At the
beginning of this century, the Second National Commission of Labour made a whole set of
sensible recommendations for such an overhaul, but they remain largely unimplemented. I
would support abolishing the firm size limit on labour retrenchment altogether, provided
there is a provision for adequate unemployment benefits, both for regular and contract
workers, and there is something like a state-provided universal basic income supplement as a
fall-back option for everybody. “Allowing more flexibility in hiring and firing has to be
combined, as part of a package deal, with a reasonable scheme of unemployment
compensation from an earmarked fund, to which employers and employees should both
regularly contribute.” For far too long businesses in India, with some notable exceptions,
have considered labour as a necessary but troublesome cog in the production machine, and
the focus is to squeeze the maximum out of it with minimum pay and benefits while
brandishing the threat of job insecurity. Organised labour, often under politicised partisan
leadership from outside, has played that adversarial game. It is in the long-term interests of
both sides to see at the ground level that labour-friendly practices can actually enhance long-
term productivity and profitability. If cooperation can replace mutual suspicion and labour
representatives can be trusted to participate in corporate governance—as is the practice, say,
in Germany and a few other European countries—labour organisations can play a responsible
role in achieving mutually beneficial goals. Taking the cover of the pandemic to unilaterally
whittle down labour protections is going the opposite way, to distrust, and labour unrest.

61. The Government of Uttar Pradesh and many other state governments promulgated
Ordinances for Temporary Exemption from Certain Labour Laws that would suspend the
operation of all labour laws applicable to factories and manufacturing establishments in
their respective state for a period of three years, with the exception of:
a) Bonded Labour System (Abolition) Act, 1976
b) Employees‘ Compensation Act, 1923,

Page 20 of 35
c) The Building and Other Construction Workers (Regulation of Employment and
Conditions of Service) Act, 1996
d) All the above
CORRECT ANSWER : OPTION D

62. On which of the following areas, Central Government is exclusively competent to enact
legislations?
a) Trade unions; industrial and labour disputes.
b) Social security and social insurance; employment and unemployment.
c) Welfare of labour including conditions of work, provident funds, employers‘ liability,
workmen‘s compensation, invalidity and old age pensions and maternity benefits.
d) Regulation of labour and safety in mines and oilfields.
CORRECT ANSWER : OPTION D

63. Which of the following laws has been enacted to prevent exploitation of inter-state
migrant workers and, to ensure fair and decent conditions of employment for them?
a) The Inter-State Migrant Workmen Act, 1979
b) Contract Labour (Regulation and Abolition) Act, 1970
c) Bonded Labour System (Abolition) Act, 1976
d) Migrant Workers (Protection) Act, 1979
CORRECT ANSWER : OPTION A

64. The Employees‘ State Insurance Act, 1948 protects the interest of workers in
contingencies such as —
I. Sickness
II. Maternity,
III. Temporary or permanent physical disablement,
IV. Death due to employment injury resulting in loss of wages or earning capacity.

Select the correct answer from the codes given bellow:


a) I only
b) II only
c) I, III and IV
d) I, II, III and IV
CORRECT ANSWER : OPTION D

65. As per the provisions contained in Chapter VB of the Industrial Dispute Act, 1947
establishment employing _____________ persons or more are required to seek prior
permission of Appropriate Government before effecting lay-off, retrenchment and closure.
a) 50
b) 100
c) 250
d) 500
CORRECT ANSWER : OPTION B

Page 21 of 35
66. ‗First come last go and last come first go‟ is the principle of:
a) Lay-off
b) Closure
c) Retrenchment
d) Dismissal
CORRECT ANSWER : OPTION C

67. Which of the following is an illegal industrial action as per law?


a) Mutual Insurance
b) Collective Bargaining
c) Lock out
d) Gherao
CORRECT ANSWER : OPTION D

68. Choose the correct objective of the Industrial Disputes Act, 1947.
a) To prevent illegal strikes
b) To promote measures for securing and preserving good relations between the employers
and the employees
c) To provide relief to workmen in matters of lay - off,
retrenchment, wrongful dismissals
d) All of the above
CORRECT ANSWER : OPTION D

69. Contract Labour (Regulation and Abolition) Act, 1970 applies to every establishment/
contractor in which ____________ workmen are employed or were employed on any day of
the preceding twelve months as contract labour.
a) Ten or more
b) Fifteen
c) Twenty or more
d) Twenty-five or more
CORRECT ANSWER : OPTION C

70. The Contract Labour (Regulation and Abolition) Act, 1970 shall not apply to
establishments in which work is of:
a) An intermittent or casual nature
b) In nature of Permanent work
c) Both a) and b)
d) None of the above.
CORRECT ANSWER : OPTION A

Page 22 of 35
Read the following passage carefully and then answer the accompanying questions
employing the concepts provided in the passage:
According to Hohfeld, legal relationships can exist only between two legal persons and one
thing. One of the two persons always has a legal advantage (that‘s the right) over the other.
The other person has the corresponding legal disadvantage. … For example employer -
employee. The basic building block of legal rights is liberty. It allows one person to do
exactly as she pleases with no duty to do otherwise. … But … the important thing about a
liberty: No one is required to respect it. It is merely a ―permission without a protection‖. …
For example, I can enjoy the view of my neighbour‘s garden but he is not under a duty to
protect my view and can screen it off.
A ―claim‖ entitles one person to limit the liberty of another, who then has a duty either to act
or not to act in certain ways toward the claimant. For example, a child‘s claim to maintenance
from parents places a duty on parents to provide maintenance. In personam claims can be
made against a definite number of persons whilst in rem claims are available against every
person in the world. …An immunity disables one person from interfering with the liberty of
another…Claims tell us what we should not do. Immunities tell us what we cannot do. … For
example, a public official cannot be prosecuted without special permission. A ―power‖ is an
ability that the law gives a person to (realise) her own legal rights or the rights of someone
else (for example the power to sue). Its correlate, the liability carries the sense of exposure to
having one‘s legal status changed. For example, only a person with locus in a case can file a
litigation to press his claims.
[Adapted from Steven Wise, Rattling the Cage Towards Legal Rights of Animals) ( 2000)]

71. The boundary wall surrounding A‘s property was broken which caused a number of
villagers to cross through his property to reach the adjoining market. A repaired the wall and
stopped the villagers. What was the nature of legal relationship?
a) Liberty-no right
b) Claim-duty
c) Immunity-disability
d) Power-liability
CORRECT ANSWER : OPTION A
72. ‗A‘ does not repair the boundary wall for more than twenty-five years and does not stop
the villagers from crossing his property. Now the villagers have:
a) An immunity
b) A claim
c) A power
d) A liberty
CORRECT ANSWER : OPTION B
73. After changing her religion, an adult medical student Bareta got married to a man of her
choice. The medical student‘s right to marry was:
a) A claim
b) A liberty
c) A power
d) A liability
CORRECT ANSWER : OPTION A

Page 23 of 35
74. The situation for Bareta would change if she were a minor. Her minority would impact on
her:
a) Claim
b) Liberty
c) Power
d) Immunity
CORRECT ANSWER : OPTION C

75. Article 21 of the Constitution of India lays down that no person can be denied life and
liberty except according to procedure established by law. By reason of this article, the
State cannot deny liberty through executive order. What does the Article impose on the
State?
a) Duty
b) Liability
c) Disability
d) No Right
CORRECT ANSWER : OPTION C

76. A person can be taken into custody only if there is a legislation specifying a procedure
which allows the deprivation of liberty. This protection from wrongful arrest granted to
people is:
a) An immunity
b) A liberty
c) A disability
d) A power
CORRECT ANSWER : OPTION A

77. The Civil Procedure Code lays down the conditions that have to be fulfilled before a
Plaint can be filed or defended. These conditions impact on the individual‘s:
a) Claim
b) Power
c) Disability
d) Liability
CORRECT ANSWER : OPTION B

78. As a rule, only States can move international institutions for the enforcement of their
rights. Special requirements have to be fulfilled before individuals can move international
institutions. These requirements impact on the __________ of individuals to obtain their
rights.
a) liberty
b) power
c) claim
d) immunity
CORRECT ANSWER : OPTION B
Page 24 of 35
79. When A enters into a contract with B, then the rights A has under the contract are:
a) In personam
b) In rem
c) Both (a) and (b)
d) None of the above
CORRECT ANSWER : OPTION A

80. Recognition of the Right to Privacy by the Indian Supreme Court has:
a) Converted a liberty into a claim right.
b) Placed a duty on the State to protect the right
c) Prevented tate from undertaking any activity which intruded on the privacy of the people
d) All of the above
CORRECT ANSWER : OPTION D

The Archimedean point of Habermas‘ philosophy of law is not the concept of natural law.
His approach to positive law differs from both Han‘s and Hobbes‘. For him, positive laws are
democratically established human artifacts. In the democratic procedure for legislatures to
make laws, even if there may be arguments appealing to the concept of natural law,
democratically established positive laws are not duplications of natural laws. Instead, they
differ from natural laws both in content and form. The legitimacy and validity of positive
laws come exclusively from the democratic process in which laws are established and
published. By the same token, the rationality of positive laws comes exclusively from a
democratic legislature based upon rational communication under the guidance of the
communicative rationality. In social management, morality is complementary to positive law.
But positive law is not subordinate to [1]. Instead, the two are parallel institutions. Habermas
shares with Han and Hobbes the view that positive laws have two salient features. First, they
are written and publically published. Second, they are backed by those who have a monopoly
on force. The second feature of positive laws is dubbed by Habermas as the ―facticity‖ of
law. The facticity or social reality of positive laws is that they are compulsory and backed by
sanctions. As Habermas puts it, ―Such laws appear as the will of a lawgiver with the power to
punish those who do not comply; to the extent that they are actually enforced and followed,
they have an existence somewhat akin to social facts.‖. Also, for Habermas, as it is for Han
and Hobbes, positive law differs from natural law in the sense that positive law is a social
institution, a human artifact, not a natural institution. Positive law comes into existence by a
historical and public action—that is, the democratically legislation of it and its being
publically published.

81. Habermas is a:
a) German Philosopher
b) French Philosopher
c) Western Social Scientist
d) English Jurist
CORRECT ANSWER : OPTION A

Page 25 of 35
82. For Hobbes, the key tenet of his philosophy is:
a) Natural law is necessary for a good positive law and they are not identical in content.
b) Natural law is not necessary for a good positive law, though they are identical in content.
c) Natural law is necessary for a good positive law, and they are identical in content but differ
in form.
d) Natural law is necessary for a good positive law but they differ in both form and content.
CORRECT ANSWER : OPTION C

83. Which of the following words has been replaced by ‗[1]‘ in the above paragraph?
a) Culture
b) Ethics
c) Morality
d) None of the above.
CORRECT ANSWER : OPTION C

84. Which one of the following statements, correctly conveys the Fuller‘s Inner Morality of
Law?
a) Every piece of Law, in order to be valid, must fulfil minimum moral standard comprising
of certain procedural requirements like generality, Prospectively promulgation, intelligibility
and consistency.
b) The contents of every law in order to be valid must be mere minimum moral standard
without anything more.
c) The question of morality of every law is a matter for the inner conscience of the legislators
and judges have nothing to do with it.
d) The question of morality of law is not for the courts to determine.
CORRECT ANSWER : OPTION A
85. In which of the following cases, the Supreme Court of India remarked, ―Whenever the
Court is entering into a new territory and is developing a new legal norm, discussion of
normative jurisprudence assumes greater significance as the Court is called upon to decide
what the legal norm should be. At the same time, normative jurisprudence has been to be
preceded by analytical jurisprudence which is necessary for the Court to underline existing
nature of law.‖?
a) Common Cause v. Union of India (2018)
b) Bhupinder Singh v. State of H. P (2011)
c) Kumar v. State of T. N (2013)
d) Gargi v. State of Haryana (2019)
CORRECT ANSWER : OPTION A

86. Out of the following jurists, whose theory has earned the name of ―Natural Law with a
Variable Content‖?
a) St. Thomas Aquinas
b) John Locke
c) Hobbes
d) R. Stammler
CORRECT ANSWER : OPTION D

Page 26 of 35
87. Which of the following philosophers gave the theory of ―Communicative Action‖?
a) Habermas
b) John Locke
c) Savigny
d) Lon L. Fuller
CORRECT ANSWER : OPTION A

88. According to Habermas, the existence and legitimacy of Positive Laws hinges upon
which of the following?
a) Morality
b) Publication of Laws
c) Rational Democratic Process
d) Judicial recognition
CORRECT ANSWER : OPTION C

89. Positive Law is called ‗Positive‘ because


a) It is made as a result of divine providence
b) It is made as a result of collective positive action
c) It is made by person in authority
d) It is followed by everybody
CORRECT ANSWER : OPTION C

90. Positivists were romanticists because:


a) They were running away from the realities of post-industrial Britain.
b) They were not imagining a perfectly ordered society.
c) They were depicting the state of law and order of contemporary Britain.
d) None of the above.
CORRECT ANSWER : OPTION A
Criminal law is the most direct expression of the relationship between a state and its citizens.
Criminal sanction is indeed the most coercive method of regulating an individual‘s behaviour
which any state may deploy. The degree of coercion under criminal law is qualitatively
different from the outcome in a dispute under civil law. The purpose of criminal law is to
forbid and prevent conduct that unjustifiably and inexcusably inflicts or threatens substantial
harm to individual or public interests. Feinberg explains the harm principle in following
words: 'It is always a good reason in support of penal legislation that it would probably be
effective in preventing (eliminating, reducing) harm to persons other than the actor and there
is probably no other means that is equally effective at no greater cost to other values'.
91. Which of the following reformed Criminal Law?
a) Malimath Committee
b) Justice M. N. Venkatachaliah Commitee
c) 52nd Law Commission Report
d) All of the above.
CORRECT ANSWER : OPTION A

Page 27 of 35
92. Under Indian Penal code ‗Culpable homicide‘ is first defined, but ‗homicide‘ is not
defined at all. ‗Culpable homicide‘, the genus, and ‗Murder‘, the species, are defined in terms
so closely that it is difficult to distinguish them. The distinction between ‗Culpable homicide‘
and ‗Murder‘ was criticised as the ‗weakest part of the code‘ by-

a) Glanville Williams
b) Jamesh Stephen
c) Jeremy Bentham
d) Simith & Hogan
CORRECT ANSWER : OPTION B

93. Lately Indian Criminal Law has been moving away from the above mentioned classical
Principles of Criminal Law. Which one of the following does not demonstrate this shift?
a) Creation of new crimes.
b) Shift in Burden of Proof
c) Presumption of Guilt
d) Broader definitions of crimes
CORRECT ANSWER : OPTION A

94. The accused must be given death penalty to satisfy the ‗collective conscience of the
society‘. Is this the correct method of determining sentence?
a) Yes
b) No
c) Yes, in Terror and Sedition Cases
d) No, as what others think is irrelevant in deciding punishment
CORRECT ANSWER : OPTION D

95. In determining the sentence, which of following factors are to be taken into
consideration?
a) Aggravating Factors
b) Mitigating Factors
c) Both Aggravating & Mitigating Factors
d) Collective Conscience of the Society
CORRECT ANSWER : OPTION C

96. The Supreme Court itself admitted in Santosh Kumar Bariyar (2009) that death penalty is
imposed ‗arbitrarily or freakishly‘. The court made a candid admission in saying that ‗there is
no uniformity of precedents‘. In Sangeet (2013), the Court yet again acknowledged that
‗principled sentencing‘ has become ‗judge centric‘. In Swami Shraddhananda (2008), the
Court said, award of death sentence depends on the ‗personal predilection of judges‘ and
there is ‗lack of uniformity‘ in capital punishment. Which of the following statements is
correct?
a) Award of Death Penalty depends on law and is given in rarest of rare cases.
b) Award of Death Penalty depends on personal ideologies of judges.

Page 28 of 35
c) a) & b) both are correct.
d) a) is wrong.
CORRECT ANSWER : OPTION B

97. In Machhi Singh (1983) a three judge bench listed five parameters to decide whether
case falls within ‗rarest of rare‘ such as the manner of commission of crime i.e. brutality,
motive, anti-social or abhorrent nature of crime, magnitude of crime and personality of victim
i.e. child, women or leader loved by people etc. Which parameter laid down by the
constitution bench in Bachan Singh was left out?
a) Too much importance was given to ‗Crime‘ but ‗Criminal‘ was left out.
b) Impact on society
c) Intent
d) Weapons used in the commission of crime
CORRECT ANSWER : OPTION A

98. Who had said that ‗The Indian Penal Code is to the English criminal law what a
manufactured article ready for use is to the materials out of which it is made. It is to the
French Penal Code and, I may add, to the North German Code of 1871, what a finished
picture is to a sketch. It is far simpler, and much better expressed, than Livingston‟s Code for
Louisiana; and its practical success has been complete.‘?
a) Lord Macaulay
b) James Stephen
c) Hari Singh Gaur
d) Justice Krishna Iyer
CORRECT ANSWER : OPTION B

99. Justice Fitzgerald observed: ‗The law of conspiracy is a branch of our jurisprudence to be
narrowly watched, to be zealously regarded and never to be pressed beyond its true limits.‘
Under Section 149, mere membership of the assembly without any participation in the crime
is sufficient. In the light of this statement, whether punishment of conspiracy by mere
agreement and under Section 149 by mere presence be deleted from the IPC?
a) Yes, if we believe in liberal and enlightened criminal jurisprudence
b) No, if we are status quoist
c) No, Conspiracy must remain punishable by mere agreement
d) No, mere presence should be enough
CORRECT ANSWER : OPTION A

100. Criminal Law Revision must reflect-


a) Deterrent theory with the aim to prevent crime.
b) Retributive theory consistent with the scheme of victim compensation.
c) Reformative theory consistent with democratic values and civil liberties
d) None of the above.
CORRECT ANSWER : OPTION C

Page 29 of 35
Read the extracts of leading judicial pronouncement and answer the questions below:
1. What is bad in theology was once good in law but after Shariat has been declared as the
personal law, whether what is Quranically wrong can be legally right is the issue to be
considered in this case. Therefore, the simple question that needs to be answered in this
case is only whether triple talaq has any legal sanctity. That is no more res integra. This Court
in [1] has held, though not in so many words, that triple talaq lacks legal sanctity. Therefore,
in terms of Article 141 [1] is the law that is applicable in India.
2. Having said that, I shall also make an independent endeavour to explain the legal position
in [1] and lay down the law explicitly.
3. [2] was enacted to put an end to the unholy, oppressive and discriminatory customs and
usages in the Muslim community. Section 2 is most relevant in the face of the present
controversy.

Application of Personal law to Muslims. – Notwithstanding any custom or usage to the


contrary, in all questions (save questions relating to agricultural land) regarding intestate
succession, special property of females, including personal property inherited or obtained
under contract or gift or any other provision of Personal Law, marriage, dissolution of
marriage, including talaq, ila, zihar, lian, khula and mubaraat, maintenance, dower,
guardianship, gifts, trusts and trust properties, and wakfs (other than charities and charitable
institutions and charitable and religious endowments) the rule of decision in cases where the
parties are Muslims shall be Muslim Personal Law (Shariat).

101. The name of which of the following judgments has been replaced by ‗[1]‘ in the passage
above?
a) Rukia Khatun v. Abdul Khalique Laskar
b) Shamim Ara v. State of UP and Another
c) Fuzlunbi v. K Khader Vali and Another
d) Mohd. Ahmad Khan v. Shah Bano Begum
CORRECT ANSWER : OPTION B

102. Which of the following legislations has been replaced by ‗[2]‘ in the passage above?
a) The Muslim Personal Law (Shariat) Application Act, 1937
b) Special Marriage Act of 1872
c) The Muslim Women (Protection of Rights on Divorce) Act 1986
d) The Muslim Women (Protection of Rights on Marriage) Act, 2019
CORRECT ANSWER : OPTION A

103. The Supreme Court constitution bench led by Chief Justice J. S. Khehar gave a
landmark judgement in Shayara bano v. Union of India. Consider the following statements:

1. Chief Justice J. S. Khehar‘s decision, along with Justice S Abdul Nazeer, concluded that
despite many findings the practice abhorrent, the Supreme Court does not have the power to
strike it down.
2. The five-member bench was divided 3-2 on the matter. The dissenting opinion instead
called for an injunction on the practice of talaq-e-biddat for six months, while also prodding
the legislature to take up the matter.
3. The majority struck it down with two judges holding it arbitrary and third judge holding it
unislamic.

Page 30 of 35
4. The Majority verdict was given by Justice Rohinton Nariman, Justice U U Lalit and Justice
D. Y. Chandrachud.

Select the correct statements about judgement:

a) 1& 2 are correct


b) 1, 2 & 3 are correct
c) 1, 2 & 4 are correct
d) All are correct

CORRECT ANSWER : OPTION B

104. Which one of the following is not correctly matched?

1. Talak Ahsan--This consists of three pronouncement of divorces made during a tuhr


(period between menstruations) followed by abstinence from sexual intercourse for the
period of iddat.
2. Talak Hasan- This consists of three pronouncements made during successive tuhrs, no
intercourse taking place during any of the three tuhrs. The first pronouncement should be
made during a tuhr, the second during the next tuhr, and the third during the succeeding
tuhr.
3. Talak-ul-Bidaat - This consists of – (i) Three pronouncements made during a single tuhr
either in one sentence, e.g., ―I divorce thee thrice,‖ - or in separate sentences e.g., ―I
divorce thee, I divorce thee, I divorce thee‖,
4. Talak-ul-Bidaat - This consists of – (ii) a single pronouncement made during a tuhr
clearly indicating an intention irrevocably to dissolve the marriage, e.g., ―I divorce thee
irrevocably.‖

a) Only 1
b) 1 and 2
c) 3 and 4
d) 1, 2 and 3
CORRECT ANSWER : OPTION A

105. In which one of the following cases, the proposition was laid down that Personal Laws
are beyond the pale of the Fundamental Rights Chapter of the Constitution and hence cannot
be struck down by this Court?

a) State of Bombay v. Narasu Appa Mali


b) Mohd. Ahmad Khan v. Shah Bano Begum
c) Daniel Latifi v. Union of India
d) Sarla Mudgal v. Union of India
CORRECT ANSWER : OPTION A

Page 31 of 35
106. What does the phrase Res integra connote in the above passage?
a) Issues of law which have not been decided or untouched by dictum or decision.
b) Issues of law which have been settled by court.
c) Issues of law where so many inconsistent decisions are present
d) Issues of law which should be resolved by the legislature and not by the court
CORRECT ANSWER : OPTION A

107. In which of the following cases, the Delhi HC had observed that the ―Introduction of
constitutional law in the home is most appropriate. It is like introducing a bull in a china
shop. It will prove to be the ruthless destroyer of the marriage institution and all that it
stands for. In the privacy of home and the married life, neither Article 21 nor Article 14 have
any place.‖?
a) Harvinder Kaur v Harmender Singh Chaudhry
b) Maneka Gandhi v. Indira Nehru Gandhi
c) T. Saritha v. Union of India
d) Prakash v. Phulwati
CORRECT ANSWER : OPTION A

108. Which of the following statement is correct?


1. Hindu Marriage Act, 1955 recognizes Personal Law of Hindus
2. Hindu Marriage Act, 1955 does not recognize Personal Laws and is a landmark
legislation in the direction of Uniform Civil Code.
3. If two Hindus register their marriage under Special Marriage Act, they will continue to be
governed by the Hindu Succession Act rather than Indian Succession Act.
4. Hindu Marriage Act, 1955 recognized widow remarriage for the first time in India.

a) Only 1
b) 1 and 2 only
c) 2, 3 and 4.
d) 1 and 3 only
CORRECT ANSWER : OPTION D

109. What is the meaning of ‗Khula‘ in the above passage?


a) Khula is the right of a woman in Islam to divorce and it means separation from her
husband
b) Khula is the right of a man in Islam to divorce and it means separation from his wife
c) Khula is ‗obtaining release from each other‘.
d) Khula is a form of Talaq practiced in Shia community.
CORRECT ANSWER : OPTION A

Page 32 of 35
110. Pick up the correct statement-

a) Muslim women cannot get divorce on the grounds of cruelty.


b) Hindu woman can get divorce on the conversion to any religion by her husband.
Conversion by itself shall not dissolve marriage of Muslim woman but she can obtain
divorce on other grounds after conversion such as cruelty, impotency, disappearance etc.
c) Hindu woman can get divorce on the non -payment of maintenance for two years
d) All the above
CORRECT ANSWER : OPTION B

The Principles of state responsibility dictate that states are accountable for breaches of
International Law. Such breaches of treaty or customary international law enable the injured
state to maintain a claim against the violating state, whether by way of diplomatic action or
by way of recourse to international mechanisms where such are in place with regard to
subject matter in issue. Recourse to International Arbitration or to the International Court of
Justice is also possible provided the necessary jurisdictional basis has been established.
Customary International Law imposes several important fundamental obligations upon the
States in the area of environmental protection. The view that the International law supports an
approach predicated upon absolute territorial sovereignty, so that a state could do as it liked
irrespective of the consequences upon other states has long been discredited. The basic duty
upon states is not so to act as to injure the rights of other states. This duty has evolved partly
out of the regime concerned with international waterways. In the [1] case, the Permanent
Court of International Justice noted that ‗this community of interest in a navigable river
becomes the basis of common legal right, the essential features of which are the perfect
equality of all riparian states in the use of the whole course of the river and the exclusion of
any preferential privileges of any riparian state in relation to others.‘. But the principle is of
far wide application. It was held in [2] case that the concept of territorial sovereignty
incorporated an obligation to protect within the territory the rights of other states. It has now
been established that it was an obligation of every state to not to allow knowingly its territory
to be used for acts contrary to the rights of other states. This judicial approach has now been
widely reaffirmed in international instruments. Article [3] of the Law of Sea Convention,
1982 provides that ‗states shall take all measures necessary to ensure that activities under
their jurisdiction and control are so conducted as not to cause damage by pollution to other
states and their environment.‘ It is sometimes argued that the appropriate standard for the
conduct of states in this field is that of strict liability. In other words, states are under an
absolute obligation to prevent pollution and are thus liable for its effects irrespective of fault.

111. In relation to State responsibility, the Permanent Court of International Justice has
observed that: ― It is a principle of international law and even a greater conception of law,
that any breach of an engagement involves an obligation to make reparation‖, in which of
the following case?
a) Chorzow Factory Case.
b) Home Missionary Society Case
c) Corfu Channel Case
d) Genocide Convention Case
CORRECT ANSWER : OPTION A

Page 33 of 35
112. The name of which one of the following cases has been replaced by ‗[1]‘ in the above
paragraph?
a) Corfu Channel Case
b) Trial Smelter Arbitration
c) International Commission on the River Oder Case
d) Case Concerning auditing of accounts between Netherlands and France.
CORRECT ANSWER : OPTION C

113. The name of which of the following cases has been replaced by ‗[2]‘ in the above
paragraph?
a) Island of Palmas Case
b) Nuclear Tests Case
c) Corfu Channel Case
d) None of the above.
CORRECT ANSWER : OPTION A

114. Which Article has been replaced by ‗[3]‘ in the above paragraph?
a) 191
b) 192
c) 193
d) 194
CORRECT ANSWER : OPTION D

115. In which of the following cases, the International Court of Justice pointed out that when
in regard to any matter of practice, two states follow it repeatedly for a long time, it becomes
a binding customary rule?
a) West Rand Central Gold Mining Company Ltd Case
b) South West Africa Case
c) Right of Passage over Indian Territory Case
d) North Sea Continental Shelf Case
CORRECT ANSWER : OPTION C

116. Doctrine of ―Sic utere tuo ut alienum non leadas‖ is contained in which of the
following?
a) Basel Convention, 1989
b) Principle 21 of Rio Declaration
c) Kyoto Protocol, 1997
d) Principles 21 and 22 of Stockholm Declaration
CORRECT ANSWER : OPTION D

Page 34 of 35
117. Advisory Opinion can be given by the International Court of Justice on Legal question:
a) On the request of Security Council Only
b) On the request of General Assembly only
c) On the request of General Assembly or Security Council or both.
d) On the request of Economic and Social Council if authorised by the Security Council.
CORRECT ANSWER : OPTION C

118. In International Law, a good example of the application of principle of Sovereignty is


the ―theory of auto-limitation‖. This theory was given by which of the following schools of
thought?
a) Positivist
b) Historical
c) Sociological
d) Naturalist
CORRECT ANSWER : OPTION A

119. Which of the following instruments refer to the ―Polluter Pays‖ Principle for fixing the
liability in environmental cases?
a) Principle 16 of Rio Declaration
b) International Convention on Oil Pollution Preparedness, Response and Cooperation, 1990.
c) Convention on Transboundary Effects of Industrial Accidents 1992.
d) All of the above.
CORRECT ANSWER : OPTION D

120.International Court of Justice is different from the Supreme Court of India because:
a) It is an International Court having jurisdiction on all countries.
b) Its judgements are binding on all the members of United Nations.
c) Its judgements have no binding force.
d) Its jurisdiction is limited to States which have consented to its jurisdiction and its
judgements are binding only on the parties to the dispute.
CORRECT ANSWER : OPTION D

Page 35 of 35
2
CONSORTIUM OF NATIONAL LAW UNIVERSITIES

LLM. – COMMON LAW ADMISSION TEST, 2019

PART-A

1. The minimum number of judges of the Supreme Court who are to sit to decide any case
involving a substantial question of law as to the interpretation of the Constitution shall be:
(A) Three (B) Five
(C) Seven (D) Nine

2. Which of the following duties was inserted by way of an amendment in 2002?


(A) To uphold and protect the sovereignty, unity and integrity of India.
(B) To value and preserve the rich heritage of our composite culture
(C) To safeguard public property and abjure violence
(D) Who is a parent or guardian to provide opportunities for education to his child or, as
the case may be, ward between the age of six and fourteen years.

3. The Parliament of India consists of:


(A) Two Houses of Parliament
(B) President and Two Houses of Parliament
(C) President, Prime Minister and two Houses of Parliament
(D) President, Vice President and two Houses of Parliament.

4. The Goods and Services Tax Council shall make recommendations to the Union and the
States on a number of issues. Special provisions to how many States the Council can make
recommendations?
(A) Six (B) Eight
(C) Eleven (D) Thirteen

5. How many High Courts are there in India?


(A) 21 (B) 22
(C) 23 (D) 24

6. What is the total strength of the Supreme Court including the Chief Justice of India?
(A) 27 (B) 29
(C) 31 (D) 32

B 3 PG 2019
7. In which case the doctrine of severability was extended by the Supreme Court to
constitutional amendments?
(A) A.K. Gopalan v. State of Madras
(B) Golaknath v. State of Punjab
(C) Keshavananda Bharati v. State of Kerala
(D) Kihota Hollahan v. Zachilhu

8. Among the following judges, against whom the motion for removal were initiated in either
House of Parliament?
(A) Justice Ramaswamy and Justice Dinakaran
(B) Justice Ramaswamy and Justice Soumitro Sen
(C) Justice Dinakaran and Justice Soumitro Sen
(D) Justice Ramaswamy and Justice Karnan

9. Recently, the Supreme Court in Ram-Janmabhumi case:


(A) Has ordered mediation by a panel
(B) Has declined to decide on the case
(C) Has upheld the Allahabad High Court‘s decision
(D) None of the above

10. The Constitution Day is celebrated on:


(A) 26th January (B) 15th August
(C) 26th November (D) 10th December

11. If the Anglo-Indian Community is adequately represented, the President may nominate not
more than __________ members to the Lok Sabha:
(A) Three (B) Two
(C) One (D) Four

12. In which case the Supreme Court applied the doctrine of Prospective Over-ruling for the first
time to resolve a constitutional question?
(A) A. K. Gopalan v. State of Madras (B) Sajjan Singh v. State of Rajasthan
(C) Shakari Prasad v. Union of India (D) Golaknath v. State of Punjab

13. How many judges of the Supreme Court were superseded in the appointment of Justice
A.N. Ray as the Chief Justice of India?
(A) One (B) Three
(C) Two (D) None

14. The Directive Principles of State Policy were framed based on the provisions of:
(A) Swiss Constitution (B) Australian Constitution
(C) Irish Constitution (D) None

PG 2019 4 B
15. The Chief Justice of India who passed away while in office was:
(A) Justice A.N. Ray (B) Justice Sabayasachi Mukharji
(C) Justice J.S. Verma (D) Justice E.S. Venkataramiah

16. Every promise and every set of promises, forming the consideration for each other is known
as
(A) Consideration
(B) Agreement
(C) Contract
(D) Reciprocal Promises

17. The correct sequence in the formation of a contract is


(A) Offer, acceptance, agreement, consideration
(B) Agreement, consideration, offer, acceptance
(C) Offer, consideration, acceptance, agreement,
(D) Offer, acceptance, consideration, agreement

18. Match List-I with List-II and select the correct answer using the codes given below the lists:
List-I List-II
(a) Mohiri Bibi Case 1. Remoteness of damage
(b) Satyabrata Ghose Case 2. Frustration of Contract
(c) Hadley v. Baxendale 3. Invitation to treat
(d) Carlill v. Carbolic 4. Minor‘s contract

Codes :

(a) (b) (c) (d)


(A) 4 2 1 3
(B) 2 3 1 4
(C) 4 1 2 3
(D) 1 2 3 4

19. Acceptance sent through post:


(A) can be revoked at any time
(B) cannot be revoked at all
(C) can be revoked before it comes to the knowledge of the offeror
(D) can be revoked only if it does not reach the offeror

B 5 PG 2019
20. ‗A‘ applied for allotment of 100 shares in ‗B‘ company. A letter of allotment addressed to ‗A‘
was posted in due time, but it never reached ‗A‘. The posting of letter of allotment:
(A) completes the contract (B) does not completes the contract
(C) makes the contract voidable (D) makes the contract void

21. A takes a life insurance policy making a false statement about his health and does not
disclose the fact that he has been treated for a serious illness. In this case which one of the
following statements is correct:
(A) The Contract is void
(B) The Contract is valid
(C) Contract is voidable on the ground of fraud
(D) Contract is voidable on the ground of misrepresentation

22. In which case it has been laid down that a promise to pay subscription is binding on the
promisor if promisee has undertaken some liability on the faith of the promisee:
(A) Lalman Shukla v. Gauri Dutt (B) Kedar Nath v. Gorie Mohd
(C) Bhagwandas v. Girdhari Lal (D) Banwari Lal v. Sukhdarshan Dayal

23. Is past consideration for a promise valid to create a contract?


(A) It is valid only in Indian Law and not in English Law
(B) It is valid only in English Law and not in Indian Law
(C) It is valid both in Indian Law and English Law
(D) It is neither valid in English Law nor in Indian Law

24. An agreement not enforceable by law is:


(A) Void (B) Contract
(C) Voidable Contract (D) Valid Contract

25. Where the relations subsisting between the parties are such that one of the parties is in a
position to dominate the will of the other and uses that position to obtain an unfair
advantage over the other is known as
(A) Coercion (B) Misrepresentation
(C) Fraud (D) Undue Influence

26. The Universal Declaration of Human Rights was adopted on:


(A) December 08, 1948 (B) December 09, 1948
(C) December 10, 1948 (D) December 11, 1949

27. The United Nations Commission on Human Rights meets every year at:
(A) The Hague (B) Geneva
(C) France (D) Spain

PG 2019 6 B
28. CEDAW was adopted by the U. N. General Assembly in the year:
(A) 1979 (B) 1989
(C) 1999 (D) 2009

29. Human Rights Treaty Bodies:


(A) Draft human rights treaties (B) Negotiate human rights treaties
(C) Monitor human rights treaties (D) Amend human rights treaties

30. The following Committee cannot hear individual complaints of human rights violations:
(A) Committee that monitors ICCPR
(B) Committee that monitors human rights bodies
(C) Committee on the Elimination of Racial Discrimination
(D) Committee on Torture

31. Section —————— of the Copyright Act grants an author ―special rights,‖ which exist
independently of the author‘s copyright, and subsists even after the assignment (whole or
partial) of the said copyright.
(A) 77 (B) 66
(C) 57 (D) 87

32. The —————— Amendment to the Patents Act stated that ―computer programs per se‖ is
not an ―invention‖ - raising a debate whether a computer program (―CP‖) with any additional
features such as technical features, would be patentable.
(A) 2000 (B) 1999
(C) 2001 (D) 2002

33. If any dispute arises regarding publication of work, —————— will be the final authority
to decide the same
(A) Copyright Board (B) Patent Board
(C) Appropriate government (D) None of the above

34. What are the types of inventions which are no patentable in India?
(A) invention which is frivolous or which claims anything obviously contrary to well
established natural laws;
(B) an invention the primary or intended use or commercial exploitation of which could be
contrary to public order or morality or which causes serious prejudice to human, animal
or plant life or health or to the environment;
(C) the mere discovery of scientific principle or the formulation of an abstract theory or
discovery of any living thing or non-living substance occurring in nature
(D) All the above

B 7 PG 2019
35. What is the term of a patent in the Indian system?
(A) 20 years (B) 40 years
(C) 30 years (D) 12 years

36. What are the classes of works for which copyrights protection is available in India?
(A) Original literary, dramatic, musical and artistic works
(B) Cinematograph films
(C) Sound recordings
(D) All the above

37. In the case of a work made or first published by or under the direction or control of any
public undertaking, —————— shall, in the absence of any agreement to the contrary, be
the first owner of the copyright therein.
(A) Government (B) Such public undertaking
(C) Both (A) and (B) (D) None of the above

38. If the period of assignment copyright is not stated, it shall be deemed to be ----years from the
date of assignment.
(A) 4 (B) 5
(C) 6 (D) 7

39. Which of the following statement is right?


(A) An invention must not possess utility for the grant of patent. No valid patent can be
granted for an invention devoid of utility.
(B) An invention must possess utility for the grant of patent. No valid patent can be
granted for an invention devoid of utility.
(C) An invention must possess utility for the grant of patent. Valid patent can be granted
for an invention devoid of utility.
(D) An invention must not necessarily possess utility for the grant of patent. Valid patent
can be granted for an invention devoid of utility.

40. How many GI Tags have been issued in India so far?


(A) About 287 (B) About 321
(C) About 344 (D) About 406

41. ‗The Courts are the capitals of law‘s empire, and judges are its princes‘ wrote:
(A) H.L.A. Hart (B) Ronald Dworkin
(C) Robert Nozick (D) John Rawls

PG 2019 8 B
42. For H.L.A. Hart, the ‗open texture of law‘ means that the regulation of areas of conduct must
be left to be developed by:
(A) law teachers (B) courts
(C) legislature (D) people at large

43. According to Immanuel Kant rational formal knowledge is:


(A) material knowledge
(B) concerned with some object
(C) concerned with the form of understanding and reasons themselves
(D) a misnomer

44. Austinian notion of ‗positive morality‘ is:


(A) a priori (B) socially constructed
(C) religiously constructed (D) law strictly so called

45. For Roscoe Pound ‗jural postulates‘ are to be discovered:


(A) in the law itself (B) outside the law
(C) in the juristic thought (D) in all of the above

46. According to John Austin, the relationship between the sovereign and political independent
society is:
(A) Symmetrical (B) Asymmetrical
(C) Elliptical (D) Relative

47. John Rawls‘s concept of justice is a:


(A) Legal Concept (B) Political Concept
(C) Sociological Concept (D) Philosophical Concept

48. According to Realists:


(A) Custom is real law (B) Precedent is real law
(C) Statute is real law (D) Rule is real law

49. ‗What are States without justice, but robber bands enlarged?‘ asked:
(A) St. Aquinas (B) St. Augustine
(C) St. Joseph (D) St. Patrick

50. The following thinker is normally associated with the secularization of natural law:
(A) Hugo Grotius (B) Antonio Gramsci
(C) Martin Heidegger (D) Auguste Comte

B 9 PG 2019
51. Which one of the following is not formally considered as an amendment to the Constitution
under Article 368?
(A) Creation of new states (B) Change in the Preamble
(C) Change in the Part IV A (D) Change in Part XII

52. Who presides over the joint sitting of the two Houses of Parliament?
(A) Speaker (B) President
(C) Vice President (D) Nominee of the Chief Justice of India.

53. Which of the following category of judges is not mentioned in the Constitution?
(A) Acting Judge. (B) Additional Judge
(C) Adhoc Judge (D) Puisne Judge

54. The right to vote in elections in India is a:


(A) Fundamental right (B) Constitutional right
(C) Statutory right (D) Customary right

55. The fundamental right to form co-operative societies is provided under:


(A) Article 19 (1) (C) (B) Article 21
(C) Article 14 (D) Article 51 A (j)

56. The Ordinance making power of the President under the Indian Constitution is:
(A) An Executive power (B) A legislative power
(C) Quasi-legislative power (D) Quasi executive power

57. The concurrent power to impose tax has been provided under:
(A) Article 246 (B) List III of Schedule VII
(C) Article 307 (D) Article 246 A

58. The Goods and Services Tax Council is set up under :


(A) Article 263 (B) Article 269-A
(C) Article 279-A (D) Article 281

59. Who was the Constitutional Advisor to the Constituent Assembly?


(A) B N Rau (B) B R Ambedkar
(C) Rajendra Prasad (D) K M Munshi

60. Which one of the following is the correct statement in relation to the Collegium?
(A) Appointment and transfer of judges in the higher judiciary
(B) Appointment and transfer of judges in the judiciary
(C) Appointment, transfer and removal of judges in the judiciary
(D) Appointment, transfer, recusal and removal of judges in the judiciary

PG 2019 10 B
61. The Vice – President of India can be removed from office by:
(A) Impeachment.
(B) Resolution passed by both Houses of Parliament.
(C) Resolution passed by the Council of States and agreed to by the House of the People.
(D) Resolution by both Houses and Order signed by the President of India.

62. The President decides on disqualification of Members of Parliament under Article 103 only
on the advice of:
(A) Council of Ministers
(B) Constitution Bench of the Supreme Court of India
(C) Election Commission
(D) Chief Justice of India.

63. The recent amendment to the Constitution of India paving way for 10% reservation for
Economically Weaker sections was done by amending:
(A) Article 14 (B) Articles 15 and 16
(C) Article 15 (D) Article 338

64. How many amendments have been made to the Indian Constitution so far?
(A) 102 (B) 121
(C) 103 (D) 126

65. The National Commission for Backward Classes was created by:
(A) The Constitution (One Hundred and First Amendment) Act, 2016
(B) The Constitution (One Hundred and Second Amendment) Act, 2018
(C) The Constitution (One Hundred and Third Amendment) Act, 2019
(D) The Constitution (One Hundredth Amendment) Act, 2015

66. Which among the following States has no Legislative Council?


(A) Andhra Pradesh (B) Telangana
(C) Bihar (D) Madhya Pradesh

67. Passive euthanasia under certain circumstance is permissible was upheld in the case of:
(A) Aruna Ramachandra Shanbaug v. Union of India
(B) Gian Kaur v. State of Punjab
(C) State of Maharashtra v. Maruty Sripaty Dubal
(D) P. Rathinam v. Union of India

B 11 PG 2019
68. Compensatory jurisprudence was invoked by the Supreme Court in:
(A) Keshavanada Bharathi v. State of Kerala
(B) Rudal Shah v. State of Bihar
(C) Indira Nehru Gandhi v. Rajnarain
(D) Kihota Hollohan v. Zachilhu

69. Basheshar Nath v. Commissioner of Income Tax, is often quoted with reference to the:
(A) Doctrine of Eclipse
(B) Doctrine of severability
(C) Doctrine of Waiver of Fundamental Rights
(D) Doctrine of territorial nexus

70. Gunupati v. Nafizul Hasan deals with:


(A) Presidents‘ election (B) Privileges of the legislature
(C) Pardoning power (D) Office of profit

71. Reservation in promotions with consequential seniority in favour of Scheduled Castes and
Scheduled Tribes is facilitated by:
(A) Art. 16 (4A) (B) Art. 16(3)
(C) Art. 16 (4B) (D) Art. 15 (3)

72. Art. 141 of the Indian Constitution provides:


(A) Law declared by it is not binding on High Courts
(B) Law declared by the Supreme Court shall be binding on all courts within the territory
of India
(C) Advisory opinion may be given
(D) Appeals from the High Court

73. Doctrine of legitimate expectation and wednesbury principles were read by Supreme Court
into:
(A) Art. 12 (B) Art.14
(C) Art. 23 (D) Art.22

74. A seven member bench of the Supreme Court unanimously struck down clauses 2(d) of
Art.323 A and Clause 3(d) of Art 323B of the Constitution relating to tribunals which
excluded the jurisdiction of High Court and Supreme Court. The court held that power of
judicial review over legislative action is vested in the High Court under Art.226 and in the
Supreme Court under Art.32. This is an integral part of the basic structure of the
constitution. Name the case:
(A) L. Chandra Kumar v. Union of India
(B) KihotaHollohon v. Zachilhu
(C) Nagaraj v. State of A.P.
(D) Rajendra Singh Rana v. Swami Prasad Maurya

PG 2019 12 B
75. The phrase ‗complete justice‘ is used in:
(A) Article 141 (B) Article 142
(C) Article 144 (D) Article 145

76. The maxim actio personalis moritur cum persona means:


(A) Personal action dies with the parties to the cause of action
(B) An action is not given to him who has received no damages
(C) No one is responsible for inevitable accidents
(D) An act done by me against my will, is not my act

77. Match the incorrect entries.


(A) Injuria sine damnum Ashby v. White
(B) Damnum sine injuria Gloucester‘s Case
(C) Remoteness of Damage Rylands v. Fletcher
(D) Negligence Donoghue v. Stevenson

78. In Rylands v. Fletcher, Justice Blackburn used the term(s):


(A) Strict liability (B) Absolute liability
(C) Strict and absolute liability (D) None of the above

79. Contributory negligence is a:


(A) Tort (B) Crime
(C) Defence (D) Right

80. Volenti non fit injuria is a:


(A) General defence (B) Particular defence
(C) Not a defence (D) Defence in Tort law

81. Test of directness for determining remoteness of damage was laid down in:
(A) Wagon Mound 1 (B) Wagon Mound 2
(C) Re Polemis (D) Rylands v. Fletcher

82. Following is not an essential of tort of negligence


(A) Duty of care on the part of plaintiff (B) Duty of care on the part of defendant
(C) Breach of duty (D) Damage to plaintiff

83. Defamation is:


(A) Both a tort and crime (B) Tort only
(C) Crime only (D) Neither a tort nor a crime

B 13 PG 2019
84. Defaming someone by words or by gestures is called in Torts as:
(A) Libel (B) Slander
(C) Nuisance (D) None

85. What defense could be used when an injury is caused to a person due to unforeseen or
unexpected events in spite of reasonable care taken by him?
(A) Inevitable accident (B) Act of third party
(C) Act of God (D) None

86. Under Section 57 of Indian Penal Code, in calculating fractions of terms of punishment,
‗imprisonment for life‘ shall be reckoned as equivalent to imprisonment for:
(A) twenty years (B) twelve years
(C) thirty years (D) imprisonment till death

87. ‗A‘ a soldier, fires on a mob by the order of his superior officer, in conformity with the
commands of the law. Choose the correct statement:
(A) A has committed the offence of murder
(B) A has committed the offence of culpable homicide
(C) A has committed no offence
(D) A has committed the offence of riot.

88. The principle that ―Nothing is an offence which is done by a child under seven years of age‖ is
provided under:
(A) Section 81 of I.P.C. (B) Section 82 of I.P.C.
(C) Section 83 of I.P.C. (D) Section 84 of I.P.C.

89. Every person has a right to defend —————— against any offence:
(A) his own body only
(B) his own body and the body of his relatives only
(C) his own body and the body of any other person
(D) he has no right to defend against any offence affecting the human body

90. X, under the influence of madness, attempts to kill Y.


(A) X has committed the offence of murder
(B) Y has no right of private defense as X is insane
(C) Y has right of private defense which he would have if X were sane
(D) All statements are correct.

91. The right of private defense of property against theft continues till:
(A) the offender has effected his retreat with the property
(B) the assistance of the public authorities is obtained
(C) the property has been recovered
(D) All of the above

PG 2019 14 B
92. A person abets the doing of a thing by:
(A) Instigating any person (B) Engages in any conspiracy
(C) Intentionally aiding (D) All of the above

93. When two or more persons agree to do an illegal act, such an act is known as:
(A) Abetment (B) Public Tranquility
(C) Criminal conspiracy (D) All of the above

94. To commit an affray, the minimum number of persons required is:


(A) Two (B) Three
(C) Five (D) Seven

95. Outraging the modesty of a woman is punishable under:


(A) Section 354, IPC (B) Section 363, IPC
(C) Section 509, IPC (D) Section 511, IPC

96. ‗Opinio juris‘ means:


(A) Opinion of the jurists (B) Opinion of law
(C) State practice (D) Law of opinion

97. ‗Jus cogens‘ means:


(A) Norms (B) Negotiable norms
(C) Non-derogable norms (D) Legal norms

98. ‘Jus gentium’ is a body of:


(A) International Statutes (B) Decisions of ICJ
(C) International Conventions (D) International Customs

99. World Intellectual Property Organization is a specialized agency of:


(A) UN (B) ICJ
(C) WTO (D) ILO

100. The Rome Statute of the International Criminal Court was adopted in the year:
(A) 1998 (B) 1999
(C) 2000 (D) 2001

––––––––––––––––

B 15 PG 2019
SPACE FOR ROUGH WORK

PG 2019 16 B
CONSORTIUM OF NATIONAL LAW UNIVERSITIES

LLM. – COMMON LAW ADMISSION TEST, 2019

PART-B

Maximum marks: 50

Instructions :

1. Answer any TWO of the following questions

2. All questions carry equal marks

3. Answer to each question shall not exceed 800 words

1. Briefly explain and critically evaluate the VVPAT and EVM in the light of contemporary
developments.

2. The Supreme Court observed that the Ordinance making power has been used to such an
extent that its use has become ‗fraud on the Constitution‘. Critically examine this statement
in the light of constitutional provisions, practices / conventions and judicial decisions.

3. Briefly explain and critically evaluate the concept of ―victims‘ compensation‖ in India in the
light of statutory provisions and judicial decisions.

4. What is meant by generic drugs? Explain its importance in the light of judicial decisions in
India.

5. Terrorism has become an international issue for long without finding any solution. Briefly
explain and critically evaluate the same in the light of contemporary developments.

————————

B 17 PG 2019
Notations :

Question Paper Name: Common Law Admission Test CLAT 2015 POST GRADUATE COURSES Actual
Subject Name:

Creation Date: 2015-04-28 11:06:51.0


Cut Off: 0
Duration: 120
Number of Questions: 150
Total Marks: 150.0

Group Number : 1
Group Id : 2
Group Maximum Duration : 0
Group Minimum Duration : 120
Revisit allowed for view? : No
Revisit allowed for edit? : No
Break time: 0
Group Marks: 150.0

Section Id : 6
Section Number : 1
Section type : Online
Mandatory or Optional: Mandatory
Number of Questions: 50
Number of Questions to be attempted: 50
Section Marks: 50.0

Sub-Section Number: 1
Sub-Section Id: 6
Question Shuffling Allowed : Yes

Question Number : 1 Question Id : 201 Question Type : MCQ Option Shuffling : No


Correct : 1.0 Wrong : 0.25

Options :
        
                                        

             

      

        
 !           !                              

             

      

        
 "           "                              

             

      

        
                                        

             

      

        
 #           #                              

             

      

        
 $           $                              
             

      

        
 %           %                              

             

      

        
 &           &                              

             

      

        
                                           

             

      

        
                                           

             

      

        
                                         

             
      

        
  !           !                              

             

      

        
  "           "                              

             

      

        
                                         

             

      

        
  #           #                              

             

      

        
  $           $                              

             

      
        
  %           %                              

             

      

        
  &           &                              

             

      

        
                                         

             

      

        
                                         

             

      

        
                                       
             

      

        
 !          !                              

             

      

        
 "          "                              

             

      

        
                                       

             

      

        
 #          #                              

             

      

        
 $          $                              
             

      

        
 %          %                              

             

      

        
 &          &                              

             

      

        
 !           !                               

             

      

        
 !           !                               

             

      
        
 !          !                              

             

      

        
 ! !          ! !                              

             

      

        
 ! "          ! "                              

             

      

        
 !          !                              

             

      

        
 ! #          ! #                              

             
      

        
 ! $          ! $                              

             

      

        
 ! %          ! %                              

             

      

        
 ! &          ! &                              

             

      

        
 "           "                               

             

      
        
 "           "                               

             

      

        
 "          "                              

             

      

        
 " !          " !                              

             

      

        
 " "          " "                              

             

      

        
 "          "                              

             

      
        
 " #          " #                              

             

      

        
 " $          " $                              

             

      

        
 " %          " %                              

             

      

        
 " &          " &                              

             

      

        
                                         

             

      
' ( ) * + , - . / 0 1

' ( ) * + , - 2 3 4 5 ( 6 0 7

' ( ) * + , - * 8 9 ( 0 : - ; + - (

- / * , 6 8 , 6 : 9 * + , - ; 0 - / * , 6 8
< = = = < = =

2 3 4 5 ( 6 , > ? 3 ( @ * + , - @ 0 A B

2 3 4 5 ( 6 , > ? 3 ( @ * + , - @ * , 5 ( * * ( 4 9 * ( / 0 A B
=

' ( ) * + , - 6 C @ 0 A B D B
< =

' 3 5 E ' ( ) * + , - 2 3 4 5 ( 6 0 F

' 3 5 E ' ( ) * + , - . / 0 1

? 3 ( @ * + , - ' G 3 > > ; + - H I ; ; , J ( / 0 K ( @

        
                                         

             

      

        
                                       

             

      

        
 !          !                              

             

      

        
 "          "                              

             

      
        
                                       

             

      

        
 #          #                              

             

      

        
 $          $                              

             

      

        
 %          %                              

             

      

        
 &          &                              

             

      

        
 #           #                               

             
      

        
 #           #                               

             

      

        
 #          #                              

             

      

        
 # !          # !                              

             

      

        
 # "          # "                              

             

      
        
 #          #                              

             

      

        
 # #          # #                              

             

      

        
 # $          # $                              

             

      

        
 # %          # %                              

             

      

        
 # &          # &                              

             

      
        
 $           $                               

             

      

        
 $           $                               

             

      

        
 $          $                              

             

      

        
 $ !          $ !                              

             

      

        
 $ "          $ "                              

             

      
        
 $          $                              

             

      

        
 $ #          $ #                              

             

      

        
 $ $          $ $                              

             

      

        
 $ %          $ %                              

             

      
        
 $ &          $ &                              

             

      

        
 %           %                               

             

      

        
 %           %                               

             

      

        
 %          %                              

             

      

        
 % !          % !                              

             
      

        
 % "          % "                              

             

      

        
 %          %                              

             

      
        
 % #          % #                              

             

      

        
 % $          % $                              

             

      

        
 % %          % %                              

             

      

        
 % &          % &                              

             

      

        
 &           &                               

             

      
        
 &           &                               

             

      

        
 &          &                              

             

      

        
 & !          & !                              

             

      

        
 & "          & "                              

             

      

        
 &          &                              
             

      

        
 & #          & #                              

             

      

        
 & $          & $                              

             

      

        
 & %          & %                              

             

      

        
 & &          & &                              

             

      
        
             !                                

             

      

' ( ) * + , - . / 0 L

' ( ) * + , - 2 3 4 5 ( 6 0 M

' ( ) * + , - * 8 9 ( 0 : - ; + - (

- / * , 6 8 , 6 : 9 * + , - ; 0 - / * , 6 8
< = = = < = =

2 3 4 5 ( 6 , > ? 3 ( @ * + , - @ 0 A B

2 3 4 5 ( 6 , > ? 3 ( @ * + , - @ * , 5 ( * * ( 4 9 * ( / 0 A B
=

' ( ) * + , - 6 C @ 0 A B D B
< =

' 3 5 E ' ( ) * + , - 2 3 4 5 ( 6 0 F

' 3 5 E ' ( ) * + , - . / 0 L

? 3 ( @ * + , - ' G 3 > > ; + - H I ; ; , J ( / 0 K ( @

        
             !                                

             

      

        
            !                               

             

      
        
   !          !  !                              

             

      

        
   "          !  "                              

             

      

        
            !                               

             

      

        
   #          !  #                              

             

      

        
   $          !  $                              

             

      
        
   %          !  %                              

             

      

        
   &          !  &                              

             

      

        
             !                                

             

      

        
             !                                

             

      

        
            !                               

             

      

        
   !          !  !                              

             
      

        
   "          !  "                              

             

      

        
            !                               

             

      

        
   #          !  #                              

             

      

        
   $          !  $                              

             

      

        
   %          !  %                              

             
      

        
   &          !  &                              

             

      

        
            !                               

             

      

        
            !                               

             

      
        
           !                              

             

      

        
  !          ! !                              

             

      

        
  "          ! "                              

             

      

        
           !                              

             

      
        
  #          ! #                              

             

      

        
  $          ! $                              

             

      

        
  %          ! %                              

             

      

        
  &          ! &                              

             
      

        
  !           ! !                               

             

      

        
  !           ! !                               

             

      

        
  !          ! !                              

             

      

        
  ! !          ! ! !                              
             

      

        
  ! "          ! ! "                              

             

      

        
  !          ! !                              

             

      

        
  ! #          ! ! #                              

             

      

        
  ! $          ! ! $                              

             

      

        
  ! %          ! ! %                              
             

      

        
  ! &          ! ! &                              

             

      

        
  "           ! "                               

             

      

        
  "           ! "                               

             

      

        
  "          ! "                              

             

      

        
  " !          ! " !                              

             

      
        
  " "          ! " "                              

             

      

        
  "          ! "                              

             

      

        
  " #          ! " #                              

             

      

        
  " $          ! " $                              

             

      

        
  " %          ! " %                              

             

      
        
  " &          ! " &                              

             

      

        
            !                               

             

      
CLAT - 2016
Section : Constitutional Law
 
  Linked  Answer  Question:  
  Direction:  Choose  the  most  appropriate  option  in  the  following  questions:    
  Sub  Question  No  :  1    
 
Q.1   The  total  number  of  members  in  the  Legislative  Council  of  a  State  having  such  a  Council  shall  not  exceed:          
     
 

  Ans      
1.  two-­‐third  of  the  total  number  of  members  in  the  Legislative  Assembly  of  that  State.    
 
2.  the  total  number  of  members  in  the  Legislative  Assembly  of  that  State.    
 
3.  one-­‐half  of  the  total  number  of  members  in  the  Legislative  Assembly  of  that  State.  
   
 
4.  one-­‐third  of  the  total  number  of  members  in  the  Legislative  Assembly  of  that  State.      
 

    Linked  Answer  Question:  


  Direction:  Choose  the  most  appropriate  option  in  the  following  questions:    
  Sub  Question  No  :  2    
 
Q.2   Right  to  enforce  which  Articles  of  the  Constitution  cannot  be  suspended  during  emergency?  
             
 

  Ans      
1.  Articles  19  and  21    
 
2.  Articles  20  and  21    
 
3.  Articles  21  and  22    
 
4.  Articles  19  and  20      
 

    Linked  Answer  Question:  


  Direction:  Choose  the  most  appropriate  option  in  the  following  questions:    
  Sub  Question  No  :  3    
 
Q.3   Which  Article  of  the  Constitution  provides  that  "Subject  to  the  provisions  of  any  law  made  by  Parliament  or  any  rules  made  under  Article  145,  the  Supreme          
Court  shall  have  the  power  to  review  any  judgement  pronounced  or  order  made  by  it"?        
 

  Ans      
1.  Article  134    
 
2.  Article  142    
 
3.  Article  137    
 
 
4.  Article  135      
    Linked  Answer  Question:  
  Direction:  Choose  the  most  appropriate  option  in  the  following  questions:    
  Sub  Question  No  :  4    
 
Q.4   Which  Article  of  the  Constitution  says  that  Law  declared  by  the  Supreme  Court  shall  be  binding  on  all  courts  within  the  territory  of  India?          
     
 

  Ans      
1.  Article  142    
 
2.  Article  141    
 
3.  Article  140    
 
 
4.  Article  146      
    Linked  Answer  Question:  
  Direction:  Choose  the  most  appropriate  option  in  the  following  questions:    
  Sub  Question  No  :  5    
 
Q.5   The  concept  of  curative  petition  was  applied  by  the  Supreme  Court  in  the  case  of:          
     
 

  Ans      
1.  P.A.  Inamdar  v.  State  of  Maharashtra  (2005)    
 
2.  Ashok  Kumar  Thakur  v.  UOI  (2008)    
 
3.  Rupa  Ashok  Hurra  v.  Ashok  Hurra  (2002)    
 
4.  Om  Kumar  v.  UOI  (2002)      
 

    Linked  Answer  Question:  


  Direction:  Choose  the  most  appropriate  option  in  the  following  questions:    
  Sub  Question  No  :  6    
 
Q.6   All  doubts  and  disputes  arising  out  of  or  in  connection  with  the  election  of  the  President  shall  be  inquired  into  and  decided  by  the:          
     
 

  Ans      
1.  Supreme  Court    
 
2.  Elected  members  of  Parliament  and  State  Legislative  Assemblies    
 
3.  Election  Commission  of  India    
 
 
4.  Parliament  in  consultation  with  Election  Commission  of  India      
    Linked  Answer  Question:  
  Direction:  Choose  the  most  appropriate  option  in  the  following  questions:    
  Sub  Question  No  :  7    
 
Q.7   Under  which  Article  of  the  Constitution,  a  Commission  to  investigate  the  conditions  of  Backward  Classes  can  be  appointed  by  the  President  of  India?  
             
 

  Ans      
1.  Article  340    
 
2.  Article  341    
 
3.  Article  342    
 
4.  Article  16  (4)      
 

    Linked  Answer  Question:  


  Direction:  Choose  the  most  appropriate  option  in  the  following  questions:    
  Sub  Question  No  :  8    
 
Q.8   Which  of  the  following  statement  is  correct?          
     
 

  Ans      
1.  A  law  put  into  the  IX  Schedule  of  the  Constitution  cannot  be  challenged  on  the  ground  of  violation  of  basic  structure    
 
2.  Only  those  laws  which  have  been  put  into  the  IX  Schedule  after  24th  April,  1973  can  be  challenged  on  the  ground  of  violation  of  basic  structure    
 
3.  A  law  put  into  the  IX  Schedule  of  the  Constitution  can  be  challenged  on  the  ground  of  violation  of  basic  structure,  irrespective  of  the  fact  when  the  
law  was  put  in  the  IX  Schedule    
 
4.  Only  those  laws  which  have  been  put  into  the  IX  Schedule  after  10th  May,  1973  can  be  challenged  on  the  ground  of  violation  of  basic  structure      
 

    Linked  Answer  Question:  


  Direction:  Choose  the  most  appropriate  option  in  the  following  questions:    
  Sub  Question  No  :  9    
 
Q.9   Which  is  the  correct  language  of  Article  20  (2)  of  the  Constitution?          
     
 

  Ans      
1.  No  person  shall  be  prosecuted  or  punished  for  the  same  offence  more  than  once.    
 
2.  No  person  shall  be  prosecuted  and  punished  for  the  same  offence  more  than  once.    
 
3.  No  person  shall  be  punished  for  the  same  offence  more  than  once.    
 
4.  No  person  shall  be  prosecuted  for  the  same  offence  more  than  once.      
 

    Linked  Answer  Question:  


  Direction:  Choose  the  most  appropriate  option  in  the  following  questions:    
  Sub  Question  No  :  10    
 
Q.10   In  which  case  Supreme  Court  held  that  "the  mental  privacy"  is  a  part  of  Article  21  of  the  Constitution?  
             
 

  Ans      
1.  Kesavananda  Bharti  v.  State  of  Kerala,  AIR  1973  SC  1461.    
 
2.  Aruna  Shanbaug  v.  UOI  (2011)  4  SCC  454    
 
3.  Selvi  v.  State  of  Karnataka  (2010)  7  SCC  263    
 
 
4.  Bachpan  Bachao  Andolan  v.  UOI  (2011)  5  SCC  1      
    Linked  Answer  Question:  
  Direction:  Choose  the  most  appropriate  option  in  the  following  questions:    
  Sub  Question  No  :  11    
 
Q.11   The  conclusive  part  of  Article  31  C,  i.e.  "and  no  law  containing  a  declaration  that  it  is  for  giving  effect  to  such  policy  shall  be  called  in  question  in  any  court  on      
       
the  ground  that  it  does  not  give  effect  to  such  policy"  was  struck  down  by  the  Supreme  Court  in  the  case  of:    
 

  Ans      
1.  Kesavananda  Bharti  v.  State  of  Kerala,  AIR  1973  SC  1461    
 
2.  Bhim  Singh  v.  UOI,  AIR  1981  SC  234    
 
3.  Golak  Nath  v.  State  of  Punjab,  AIR  1967  SC  1643    
 
 
4.  Minerva  Mills  v.  UOI,  AIR  1980  SC  1789      
    Linked  Answer  Question:  
  Direction:  Choose  the  most  appropriate  option  in  the  following  questions:    
  Sub  Question  No  :  12    
 
Q.12   In  which  case  Articles  323  A  (2)  (d)  and  Article  323  B  (3)  (d)  were  declared  unconstitutional  to  the  extent  they  exclude  the  jurisdiction  of  the  High  Courts  and      
       
the  Supreme  Court  under  Articles  226/227  and  32  of  the  Constitution?    
 

  Ans      
1.  L.  Chandra  Kumar  v.  Union  of  India  (1997)    
 
2.  I.R.  Coelho  v.  State  of  Tamil  Nadu  (2007)    
 
3.  Maneka  Gandhi  v.  Union  of  India  (1978)    
 
4.  S.P.  Sampath  Kumar  v.  Union  of  India  (1987)      
 

    Linked  Answer  Question:  


  Direction:  Choose  the  most  appropriate  option  in  the  following  questions:    
  Sub  Question  No  :  13    
 
Q.13   Financial  emergency  under  Article  360  of  the  Constitution  once  imposed,  after  the  approval  by  both  the  Houses  of  Parliament  within  the  period  of  two          
months,  shall  remain  valid  for  the  period  of:        
 

  Ans      
1.  one  year    
 
2.  till  it  is  revoked  by  the  Parliament    
 
3.  six  months    
 
4.  three  year      
 

    Linked  Answer  Question:  


  Direction:  Choose  the  most  appropriate  option  in  the  following  questions:    
  Sub  Question  No  :  14    
 
Q.14   The  Constitution  (One  Hundredth  Amendment)  Act,  2015  gave  effect  to  the  exchange  of:          
     
 

  Ans      
1.  51  Bangladeshi  and  51  Indian  enclaves.    
 
2.  111  Indian  and  111  Bangladeshi  Enclaves.    
 
3.  51  Bangladeshi  and  111  Indian  Enclaves.    
 
4.  111  Bangladeshi  and  51  Indian  Enclaves.      
 

    Linked  Answer  Question:  


  Direction:  Choose  the  most  appropriate  option  in  the  following  questions:    
  Sub  Question  No  :  15    
 
Q.15   Which  case  is  popularly  known  as  "Cash  for  Question"  case?          
     
 

  Ans      
1.  Rajnath  Singh  v.  Hon'ble  Speaker  Lok  Sabha  (2008)    
 
2.  In  re  Keshav  Singh  case  (1965)    
 
3.  Amrinder  Singh  v.  Punjab  Vidhan  Sabha  (2010)    
 
 
4.  Raja  Ram  Pal  v.  Hon'ble  Speaker  Lok  Sabha  (2007)      
    Linked  Answer  Question:  
  Direction:  Choose  the  most  appropriate  option  in  the  following  questions:    
  Sub  Question  No  :  16    
 
Q.16   If  any  question  arises  as  to  whether  a  member  of  either  House  of  Parliament  has  become  subject  to  any  of  the  disqualifications  mentioned  in  Clause  (1)  of          
Article  102  of  the  Constitution,  the  question  shall  be  referred  for  the  decision  of  the  _______  and  his  decision  shall  be  final.        
 

  Ans      
1.  Prime  Minister    
 
2.  Vice-­‐President    
 
3.  Speaker/Chairman  of  the  House    
 
 
4.  President      
    Linked  Answer  Question:  
  Direction:  Choose  the  most  appropriate  option  in  the  following  questions:    
  Sub  Question  No  :  17    
 
Q.17   Which  Article  of  the  Constitution  puts  obligation  on  the  State  to  promote  with  special  care  the  educational  and  economic  interests  of  the  weaker  sections  of      
the  people,  and,  in  particular,  of  the  Scheduled  Castes  and  the  Scheduled  Tribes?            
 

  Ans      
1.  Article  46    
 
2.  Article  43    
 
3.  Article  45    
 
4.  Article  21-­‐A      
 

    Linked  Answer  Question:  


  Direction:  Choose  the  most  appropriate  option  in  the  following  questions:    
  Sub  Question  No  :  18    
 
Q.18   The  President  of  India  may  be  removed  from  the  office  by  impeachment  on  the  ground  of:          
     
 

  Ans      
1.  proved  misbehavior  or  incapacity    
 
2.  proved  misbehavior  and  misconduct    
 
3.  violation  of  the  Constitution    
 
 
4.  proved  misbehavior      
    Linked  Answer  Question:  
  Direction:  Choose  the  most  appropriate  option  in  the  following  questions:    
  Sub  Question  No  :  19    
 
Q.19   An  Ordinance  promulgated  by  the  President  under  Article  123  of  the  Constitution  shall  cease  to  operate  at  the  expiration  of  ____  from  the  reassembly  of          
Parliament,  or,  if  before  the  expiration  of  that  period  resolutions  disapproving  it  are  passed  by  both  the  Houses  of  Parliament  and  in  such  case  upon  the      
 

passing  of  the  second  of  those  resolutions;  and  may  be  withdrawn  at  any  time  by  the  President.    
  Ans      
1.  two  months    
 
2.  six  months    
 
3.  six  weeks    
 
 
4.  one  month      
    Linked  Answer  Question:  
  Direction:  Choose  the  most  appropriate  option  in  the  following  questions:    
  Sub  Question  No  :  20    
 
Q.20   By  which  amendment  to  the  Constitution,  Article  16  (4B)  was  added?          
     
 

  Ans      
1.  The  Constitution  (Eighty-­‐fifth  Amendment)  Act,  2001    
 
2.  The  Constitution  (Ninetieth  Amendment)  Act,  2003    
 
3.  The  Constitution  (Eighty-­‐first  Amendment)  Act,  2000    
 
 
4.  The  Constitution  (Eighty-­‐second  Amendment)  Act,  2000      
    Linked  Answer  Question:  
  Direction:  Choose  the  most  appropriate  option  in  the  following  questions:    
  Sub  Question  No  :  21    
 
Q.21   Which  Schedule  of  the  Constitution  of  India  deals  with  the  provisions  as  to  disqualification  on  ground  of  defection?  
             
 

  Ans      
1.  X    
 
2.  VIII    
 
3.  VII    
 
4.  IX      
 

    Linked  Answer  Question:  


  Direction:  Choose  the  most  appropriate  option  in  the  following  questions:    
  Sub  Question  No  :  22    
 
Q.22   Article  21-­‐A  was  inserted  in  the  Indian  Constitution  by:          
     
 

     
Ans  
1.  the  Constitution  (Ninety-­‐second  Amendment)  Act,  2003    
 
2.  the  Constitution  (  Eighty-­‐fifth  Amendment)  Act,  2001    
 
3.  the  Constitution  (Forty-­‐second  Amendment)  Act,  1976    
 
 
4.  the  Constitution  (Eighty-­‐sixth  Amendment)  Act,  2002      
    Linked  Answer  Question:  
  Direction:  Choose  the  most  appropriate  option  in  the  following  questions:    
  Sub  Question  No  :  23    
 
Q.23   The  doctrine  of  "occupied  field"  was  applied  by  the  Supreme  Court  in  the  case  of:          
     
 

  Ans      
1.  Union  of  India  v.  Tulsi  Ram  Patel  (1985)    
 
2.  A.  B.  Krishna  v.  State  of  Karnataka  (1998)    
 
3.  I.  R.  Coelho  v.  Union  of  India  (2007)    
 
4.  Kesavananda  Bharti  v.  State  of  Kerala  )1973)      
 

    Linked  Answer  Question:  


  Direction:  Choose  the  most  appropriate  option  in  the  following  questions:    
  Sub  Question  No  :  24    
 
Q.24   In  which  Article  of  the  Constitution  the  "Contingency  Fund"  of  India  is  defined?          
     
 

  Ans      
1.  Article  267    
 
2.  Article  266    
 
3.  Article  269    
 
4.  Article  268      
 

    Linked  Answer  Question:  


  Direction:  Choose  the  most  appropriate  option  in  the  following  questions:    
  Sub  Question  No  :  25    
 
Q.25   In  case  of  a  deadlock  over  an  Ordinary  Bill  between  two  Houses  in  a  State  Legislature,  the  Bill  is  deemed  to  have  been  passed:          
     
 

  Ans      
1.  after  the  Bill  has  been  passed  for  the  second  time  by  the  Legislative  Assembly  and  more  than  six  months  elapse  from  the  date  on  which  the  Bill  is  laid  
for  the  second  time  before  the  Legislative  Council  without  the  Bill  being  passed  by  it.    
 
2.  after  the  Bill  has  been  passed  for  the  second  time  by  the  Legislative  Assembly  and  more  than  three  months  elapse  from  the  date  on  which  the  Bill  is  
laid  for  the  second  time  before  the  Legislative  Council  without  the  Bill  being  passed  by  it.    
 
3.  only  in  the  form  in  which  it  is  passed  in  the  joint  sitting  of  both  the  Houses  of  the  State  Legislature.    
 
4.  after  the  Bill  has  been  passed  for  the  second  time  by  the  Legislative  Assembly  and  more  than  one  month  elapses  from  the  date  on  which  the  Bill  is  laid  
for  the  second  time  before  the  Legislative  Council  without  the  Bill  being  passed  by  it.      
 

    Linked  Answer  Question:  


  Direction:  Choose  the  most  appropriate  option  in  the  following  questions:    
  Sub  Question  No  :  26    
 
Q.26   Election  to  fill  a  vacancy  in  the  office  of  President  occurring  by  reason  of  his  death,  resignation  or  removal,  or  otherwise,  shall  be  held  as  soon  as  possible          
after,  and  in  no  case  later  than:        
 

  Ans      
1.  two  months  from  the  date  of  occurrence  of  the  vacancy    
 
2.  six  months  from  the  date  of  occurrence  of  the  vacancy    
 
3.  one  month  from  the  date  of  occurrence  of  the  vacancy    
 
4.  three  months  from  the  date  of  occurrence  of  the  vacancy      
 

    Linked  Answer  Question:  


  Direction:  Choose  the  most  appropriate  option  in  the  following  questions:    
  Sub  Question  No  :  27    
 
Q.27   Who  in  respect  of  every  financial  year  cause  to  be  laid  before  both  the  Houses  of  Parliament,  a  statement  of  the  estimated  receipts  and  expenditure  of  the  
           
Government  of  India  for  that  year,  which  is  known  as  "annual  financial  statement"?    
 

  Ans      
1.  Speaker  of  the  House    
 
2.  Prime  Minister    
 
3.  President    
 
 
4.  Finance  Minister      
    Linked  Answer  Question:  
  Direction:  Choose  the  most  appropriate  option  in  the  following  questions:    
  Sub  Question  No  :  28    
 
Q.28   By  which  amendment  the  ground  of  "armed  rebellion"  for  imposing  national  emergency  was  substituted  for  the  ground  "internal  disturbance"?          
     
 

  Ans      
1.  The  Constitution  (Thirty-­‐ninth  Amendment)  Act,  1975    
 
2.  The  Constitution  (Forty-­‐second  Amendment)  Act,  1976    
 
3.  The  Constitution  (Fiftieth  Amendment)  Act,  1984    
 
4.  The  Constitution  (Forty-­‐fourth  Amendment)  Act,  1978      
 

    Linked  Answer  Question:  


  Direction:  Choose  the  most  appropriate  option  in  the  following  questions:    
  Sub  Question  No  :  29    
 
Q.29   Article  30  of  the  Constitution  gives  the  right  to  establish  and  administer  educational  institutions  of  their  choice  to  all  minorities  whether  based  on:          
     
 

  Ans      
1.  religion  and  language    
 
2.  language    
 
3.  religion  or  language    
 
4.  religion      
 

    Linked  Answer  Question:  


  Direction:  Choose  the  most  appropriate  option  in  the  following  questions:    
  Sub  Question  No  :  30    
 
Q.30   Which  of  the  following  judgment  of  the  Supreme  Court  is  related  to  rights  of  third  gander?  
             
 

  Ans      
1.  Democratic  Union  v.  Govt.  of  India  (2015)    
 
2.  Ramprakash  v.  State  of  UP  (2013)    
 
3.  Peoples  Union  for  Civil  Liberty  v.  State  of  Maharashtra  (2000)    
 
 
4.  National  Legal  Services  Authority  v.  Union  of  India  (2014)      
    Linked  Answer  Question:  
  Direction:  Choose  the  most  appropriate  option  in  the  following  questions:    
  Sub  Question  No  :  31    
 
Q.31   In  which  case  the  Supreme  Court  declared  Clauses  (4)  and  (5)  of  Article  368  of  the  Constitution  as  unconstitutional?          
     
 

  Ans      
1.  Minerva  Mills  Ltd.  v.  Union  of  India  (1980)    
 
2.  I.R.  Coelho  v.  Union  of  India  (2007)    
 
3.  Indira  Nehru  Gandhi  v.  Raj  Narain  (1976)    
 
 
4.  L.  Chandra  Kumar  v.  Union  of  India  (1997)      
    Linked  Answer  Question:  
  Direction:  Choose  the  most  appropriate  option  in  the  following  questions:    
  Sub  Question  No  :  32    
 
Q.32   In  which  case  the  Supreme  Court  of  India  held  that  "right  to  life"  does  not  include  "right  to  die"?          
     
 

  Ans      
1.  Gian  Kaur  v.  State  of  Punjab  (1996)    
 
2.  State  of  Maharashtra  v.  Maruti  Sripati  (1987)    
 
3.  Mithu  v.  State  of  Punjab  (1983)    
 
4.  P.  Rathinam  v.  Union  of  India  (1994)      
 

    Linked  Answer  Question:  


  Direction:  Choose  the  most  appropriate  option  in  the  following  questions:    
  Sub  Question  No  :  33    
 
Q.33   Under  Article  355  of  the  Constitution  it  shall  be  the  duty  of  the  Union  to  protect  every  State  against:          
     
 

  Ans      
1.  war,  external  aggression  and  armed  rebellion    
 
2.  external  aggression    
 
3.  external  aggression  and  armed  rebellion    
 
4.  external  aggression  and  internal  disturbance      
 

    Linked  Answer  Question:  


  Direction:  Choose  the  most  appropriate  option  in  the  following  questions:    
  Sub  Question  No  :  34    
 
Q.34   In  the  Constitution  of  India,  disqualification  for  appointment  on  remunerative  political  post  is  provided  in:          
     
 

  Ans      
1.  Article  361  -­‐  A    
 
2.  Article  361    
 
3.  Article  361  -­‐  B    
 
 
4.  Article  362      
    Linked  Answer  Question:  
  Direction:  Choose  the  most  appropriate  option  in  the  following  questions:    
  Sub  Question  No  :  35    
 
Q.35   Article  29  (2)  of  the  Constitution  provides  that  "No  citizen  shall  be  denied  admission  into  any  educational  institution  maintained  by  the  State  or  receiving  aid      
       
out  of  State  funds  on  grounds  only  of:    
 

  Ans      
1.  religion,  race,  caste,  place  of  birth,  language  or  any  of  them.    
 
2.  religion,  race,  caste,  language  or  any  of  them.    
 
3.  race,  caste,  sex,  language  or  any  of  them.    
 
 
4.  religion  race,  caste,  sex,  language  or  any  of  them.      
    Linked  Answer  Question:  
  Direction:  Choose  the  most  appropriate  option  in  the  following  questions:    
  Sub  Question  No  :  36    
 
Q.36   The  joint  sitting  of  both  the  Houses  of  Parliament  in  certain  cases  can  be  called  by  the  President  under  Article:          
     
 

  Ans      
1.  107    
 
2.  110    
 
3.  108    
 
4.  109      
 

    Linked  Answer  Question:  


  Direction:  Choose  the  most  appropriate  option  in  the  following  questions:    
  Sub  Question  No  :  37    
 
Q.37   For  the  election  of  the  President,  the  electoral  college  shall  consist  of:          
     
 

  Ans      
1.  Elected  members  of  both  the  Houses  of  Parliament,  members  of  the  Legislative  Assemblies  of  the  States  and  elected  members  of  the  Legislative  
Assemblies  of  Delhi  and  Pondicherry.    
 
2.  Members  of  both  the  Houses  of  Parliament,  members  of  Legislative  Assemblies  of  States  and  Legislative  Assemblies  of  Delhi  and  Pondicherry.    
 
3.  Elected  members  of  both  the  Houses  of  Parliament,  members  of  the  Legislative  Assemblies  of  States  including  National  Capital  Territory  of  Delhi  and  
the  Union  Territory  of  Pondicherry.    
 
4.  Elected  members  of  both  the  Houses  of  Parliament  and  elected  members  of  Legislative  Assemblies  of  the  States  including  National  Capital  Territory  of  
Delhi  and  the  Union  Territory  of  Pondicherry.      
 

    Linked  Answer  Question:  


  Direction:  Choose  the  most  appropriate  option  in  the  following  questions:    
  Sub  Question  No  :  38    
 
Q.38   In  which  case  the  Supreme  Court  ruled  that  the  defence  of  "sovereign  immunity"  is  alien  to  the  concept  of  guarantee  of  fundamental  rights?          
     
 

     
Ans  
1.  Rudal  Sah  v.  State  of  Bihar  (1983)    
 
2.  PUDR  v.  Union  of  India  (1983)    
 
3.  Nilabati  Behera  v.  State  of  Orissa  (1993)    
 
 
4.  Maneka  Gandhi  v.  Union  of  India  (1978)      
    Linked  Answer  Question:  
  Direction:  Choose  the  most  appropriate  option  in  the  following  questions:    
  Sub  Question  No  :  39    
 
Q.39   In  which  case  the  Supreme  Court  observed  that  there  is  no  fundamental  right  to  strike?          
     
 

  Ans      
1.  Sodan  Hussain  v.  N.D.M.C.  (1989)    
 
2.  Delhi  Police  Non-­‐Gazetted  Karamchari  Union  v.  Union  of  India  (1987)    
 
3.  T.  K.  Rangarajan  v.  State  of  Tamil  Nadu  (2003)    
 
4.  Manohar  Lal  v.  State  of  Punjab  (1961)      
 

    Linked  Answer  Question:  


  Direction:  Choose  the  most  appropriate  option  in  the  following  questions:    
  Sub  Question  No  :  40    
 
Q.40   In  which  case  the  administration  had  approached  the  High  Court  seeking  order  for  termination  of  pregnancy  of  an  orphan  women,  suffering  from  mild          
mental  retardation  and  found  pregnant  (allegedly  on  having  been  raped)?        
 

  Ans      
1.  Savita  Kumari  v.  Chandigarh  Admn.    
 
2.  Orphan  Society  v.  Delhi  Admn.    
 
3.  Suchita  Srivastava  v.  Chandigarh  Admn.    
 
4.  Suchita  Krishnan  v.  Delhi  Admn.      
 

    Linked  Answer  Question:  


  Direction:  Choose  the  most  appropriate  option  in  the  following  questions:    
  Sub  Question  No  :  41    
 
Q.41   The  operative  part  of  Article  310  (1)  of  the  Constitution  is:          
     
 

  Ans      
1.  Subject  to  the  provisions  of  this  Constitution    
 
2.  Notwithstanding  anything  provided  by  this  Constitution    
 
3.  Except  as  expressly  provided  by  this  Constitution    
 
 
4.  Subject  to  the  provision  of  Article  311      
    Linked  Answer  Question:  
  Direction:  Choose  the  most  appropriate  option  in  the  following  questions:    
  Sub  Question  No  :  42    
 
Q.42   Under  which  Articles  of  the  Constitution  the  rights  given  therein  are  available  only  to  the  citizens  of  India?          
     
 

  Ans      
1.  Articles  15,  16,  19  and  17    
 
2.  Articles  15,  16,  19  and  21    
 
3.  Articles  15,  16,  19  and  29    
 
 
4.  Articles  15,  16,  19  and  25  (2)      
    Linked  Answer  Question:  
  Direction:  Choose  the  most  appropriate  option  in  the  following  questions:    
  Sub  Question  No  :  43    
 
Q.43   Which  Article  of  the  Constitution  puts  a  duty  on  every  citizen  of  India  to  renounce  practices  derogatory  to  the  dignity  of  the  woman?          
     
 

  Ans      
1.  Article  21    
 
2.  Article  51  A  (g)    
 
3.  Article  51  A  (e)    
 
 
4.  Article  19      
    Linked  Answer  Question:  
  Direction:  Choose  the  most  appropriate  option  in  the  following  questions:    
  Sub  Question  No  :  44    
 
Q.44   In  which  case  the  Supreme  Court  upheld  the  use  of  Aadhaar  Card  for  certain  government  schemes?          
     
 

  Ans      
1.  K.  S.  Puttaswamy  v.  UOI    
 
2.  Sandeep  v.  UOI    
 
3.  M.  C.  Mehta  v.  UOI    
 
4.  PUDR  v.  UOI      
 

    Linked  Answer  Question:  


  Direction:  Choose  the  most  appropriate  option  in  the  following  questions:    
  Sub  Question  No  :  45    
 
Q.45   Under  which  Article  of  the  Constitution  the  Prime  Minister  is  duty  bound  to  furnish  information  to  the  President  regarding  all  decisions  of  Council  of  
           
Ministers  relating  to  the  administration  of  the  affairs  of  the  Union  and  proposals  for  legislation  as  the  President  may  call  for?    
 

  Ans      
1.  Article  76    
 
2.  Article  78    
 
3.  Article  79    
 
4.  Article  77      
 

    Linked  Answer  Question:  


  Direction:  Choose  the  most  appropriate  option  in  the  following  questions:    
  Sub  Question  No  :  46    
 
Q.46   President  can  appoint  distinguished  jurist  as  a  Judge:          
     
 

  Ans      
1.  of  the  High  Court  only    
 
2.  of  the  High  Court  as  well  as  Supreme  Court    
 
3.  neither  of  the  High  Court  nor  of  the  Supreme  Court    
 
 
4.  of  the  Supreme  Court  only      
    Linked  Answer  Question:  
  Direction:  Choose  the  most  appropriate  option  in  the  following  questions:    
  Sub  Question  No  :  47    
 
Q.47   Which  of  the  following  statement  is  correct?          
     
 

  Ans      
1.  Some  provisions  of  the  Constitution  came  into  force  on  26.11.1949  and  the  remaining  provisions  came  into  force  on  26.01.1950.    
 
2.  All  the  provisions  of  the  Constitution  came  into  force  on  15.08.1947.    
 
3.  Some  provisions  of  the  Constitution  came  into  force  on  15.08.1947  and  the  remaining  provisions  came  into  force  on  26.11.1949.    
 
 
4.  All  the  provisions  of  the  Constitution  came  into  force  on  26.01.1950.      
    Linked  Answer  Question:  
  Direction:  Choose  the  most  appropriate  option  in  the  following  questions:    
  Sub  Question  No  :  48    
 
Q.48   By  which  amendment  to  the  Constitution,  Article  13  (4)  was  added?          
     
 

  Ans      
1.  The  Constitution  (Twenty-­‐fifth  Amendment)  Act,  1971    
 
2.  The  Constitution  (Twenty-­‐fourth  Amendment)  Act,  1971    
 
3.  The  Constitution  (Twenty-­‐ninth  Amendment)  Act,  1972    
 
4.  The  Constitution  (Twenty-­‐third  Amendment)  Act,  1969      
 

    Linked  Answer  Question:  


  Direction:  Choose  the  most  appropriate  option  in  the  following  questions:    
  Sub  Question  No  :  49    
 
Q.49   The  total  number  of  ministers,  including  the  Prime  Minister,  in  the  Council  of  Ministers  shall  not  exceed:          
     
 

     
Ans  
1.  15%  of  the  total  number  of  members  of  the  Parliament    
 
2.  15%  of  the  total  number  of  members  of  House  of  the  People    
 
3.  15%  of  the  total  number  of  elected  members  of  House  of  the  People  and  Council  of  States    
 
 
4.  15%  of  the  total  number  of  members  of  the  Council  of  States      
    Linked  Answer  Question:  
  Direction:  Choose  the  most  appropriate  option  in  the  following  questions:    
  Sub  Question  No  :  50    
 
Q.50   In  which  case  the  Supreme  Court  upheld  the  Haryana  Panchayati  Raj  Act  prescribing  minimum  educational  qualifications  for  contesting  elections  of          
Panchayats?        
 

  Ans      
1.  Chaudhry  Ranbir  Singh  v.  State  of  Haryana    
 
2.  Ravibala  v.  State  of  Haryana    
 
3.  Rajbala  v.  State  of  Haryana    
 
4.  Chaudhry  Ravi  Singh  v.  State  of  Haryana      
 

Section : Jurisprudence
 
  Linked  Answer  Question:  
  Direction:  Choose  the  most  appropriate  option  in  the  following  questions:    
  Sub  Question  No  :  1    
 
Q.1   The  Indian  Penal  Code  is  a:          
     
 

  Ans      
1.  customary  code    
 
2.  consolidating  code    
 
3.  refined  code    
 
4.  creative  code      
 

    Linked  Answer  Question:  


  Direction:  Choose  the  most  appropriate  option  in  the  following  questions:    
  Sub  Question  No  :  2    
 
Q.2   Who  defined  'administration  of  justice'  as  the  'maintenance  of  right  within  a  political  community  by  means  of  the  physical  force  of  the  State'?          
     
 

  Ans      
1.  Duguit    
 
2.  Keeton    
 
3.  Ihering    
 
 
4.  Salmond      
    Linked  Answer  Question:  
  Direction:  Choose  the  most  appropriate  option  in  the  following  questions:    
  Sub  Question  No  :  3    
 
Q.3   "  The  movement  of  progressive  societies  has  hitherto  been  a  movement  from  status  to  contract".  This  was  stated  by:  
             
 

  Ans      
1.  Savigny    
 
2.  Maitland    
 
3.  Savigny  and  Maine    
 
 
4.  Maine      
    Linked  Answer  Question:  
  Direction:  Choose  the  most  appropriate  option  in  the  following  questions:    
  Sub  Question  No  :  4    
 
Q.4   The  absolutist  theories  of  sovereignty  were  given  by:          
     
 

  Ans      
1.  Bodin,  Hobbes  and  Austin    
 
2.  Austin;  Almond  and  Maine    
 
3.  Austin,  Kelsen  and  Maine    
 
 
4.  Austin;  Bentham  and  Savigny      
    Linked  Answer  Question:  
  Direction:  Choose  the  most  appropriate  option  in  the  following  questions:    
  Sub  Question  No  :  5    
 
Q.5   While  differentiating  between  judicial  and  legislative  creativity,  it  was  stated  that  "the  creative  power  of  the  courts  is  limited  by  existing  legal  material  at          
their  command.  They  find  the  material  and  shape  it.  The  legislature  may  manufacture  entirely  new  material".  Who  made  this  statement?        
 

  Ans      
1.  Savigny    
 
2.  Allen    
 
3.  Salmond    
 
4.  Austin      
 

    Linked  Answer  Question:  


  Direction:  Choose  the  most  appropriate  option  in  the  following  questions:    
  Sub  Question  No  :  6    
 
Q.6   Find  out  the  odd  man  out  among  the  following:          
     
 

  Ans      
1.  Comte    
 
2.  Hegel    
 
3.  Savigny    
 
4.  Puchta      
 

    Linked  Answer  Question:  


  Direction:  Choose  the  most  appropriate  option  in  the  following  questions:    
  Sub  Question  No  :  7    
 
Q.7   That  "Jurisprudence  then,  in  the  present  hypothesis,  is  the  Lawyers  extra  version"  was  observed  by:          
     
 

  Ans      
1.  Julius  Stone    
 
2.  John  Locke    
 
3.  Kelsen    
 
 
4.  Ihering      
    Linked  Answer  Question:  
  Direction:  Choose  the  most  appropriate  option  in  the  following  questions:    
  Sub  Question  No  :  8    
 
Q.8   Which  of  the  following  is  said  to  be  "Nine  Points  in  Law"?          
     
 

  Ans      
1.  Ownership    
 
2.  Right    
 
3.  Possession    
 
 
4.  Might      
    Linked  Answer  Question:  
  Direction:  Choose  the  most  appropriate  option  in  the  following  questions:    
  Sub  Question  No  :  9    
 
Q.9   Who  said  that  "Certainty  of  law  is  a  legal  myth"?          
     
 

  Ans      
1.  Kelsen    
 
2.  Salmond    
 
3.  Jerome  Frank    
 
4.  Holmes      
 

    Linked  Answer  Question:  


  Direction:  Choose  the  most  appropriate  option  in  the  following  questions:    
  Sub  Question  No  :  10    
 
Q.10   A  Right  is:          
     
 

     
Ans  
1.  A  righteous  claim    
 
2.  A  legal  claim    
 
3.  A  just  claim    
 
 
4.  A  moral  claim      
    Linked  Answer  Question:  
  Direction:  Choose  the  most  appropriate  option  in  the  following  questions:    
  Sub  Question  No  :  11    
 
Q.11   According  to  the  "Purpose  theory"  of  personality:          
     
 

  Ans      
1.  The  dignity  of  being  a  "juristic  person"  as  having  to  be  conceded  by  the  State.    
 
2.  beneficiary  of  a  foundation  is  a  person.    
 
3.  person  is  applicable  only  to  human  beings.    
 
4.  corporate  entity  is  a  person.      
 

    Linked  Answer  Question:  


  Direction:  Choose  the  most  appropriate  option  in  the  following  questions:    
  Sub  Question  No  :  12    
 
Q.12   Rights  in  're  propria'  mean:          
     
 

  Ans      
1.  Intangible  rights    
 
2.  Rights  in  one's  own  things    
 
3.  Tangible  rights    
 
4.  Rights  in  the  things  of  others      
 

    Linked  Answer  Question:  


  Direction:  Choose  the  most  appropriate  option  in  the  following  questions:    
  Sub  Question  No  :  13    
 
Q.13   The  physical  control  of  an  object  is:          
     
 

  Ans      
1.  possessio  civilis    
 
2.  practor's  interdicts    
 
3.  possessio  naturales    
 
 
4.  possessio  interdicts      
    Linked  Answer  Question:  
  Direction:  Choose  the  most  appropriate  option  in  the  following  questions:    
  Sub  Question  No  :  14    
 
Q.14   Which  of  the  following  was  the  famous  work  of  Jermy  Bentham?          
     
 

  Ans      
1.  Limits  of  Jurisprudence  Defined    
 
2.  The  Concept  of  Law    
 
3.  The  Spirit  of  Law    
 
 
4.  Province  of  Law  Determined      
    Linked  Answer  Question:  
  Direction:  Choose  the  most  appropriate  option  in  the  following  questions:    
  Sub  Question  No  :  15    
 
Q.15   According  to  Historical  School,  the  most  important  source  of  law  is:          
     
 

  Ans      
1.  Judicial  Precedent    
 
2.  Legislation    
 
3.  Custom    
 
 
4.  Morality      
    Linked  Answer  Question:  
  Direction:  Choose  the  most  appropriate  option  in  the  following  questions:    
  Sub  Question  No  :  16    
 
Q.16   Who  stated,  "Common  law  is  essentially  judge  made  law"?          
     
 

  Ans      
1.  Salmond    
 
2.  Pollock    
 
3.  Holmes    
 
4.  Blackstone      
 

    Linked  Answer  Question:  


  Direction:  Choose  the  most  appropriate  option  in  the  following  questions:    
  Sub  Question  No  :  17    
 
Q.17   One  of  the  important  aspects  of  Sociological  School  is  to  study:  
             
 

  Ans      
1.  Effect  of  law  on  international  order    
 
2.  Effect  of  law  on  society    
 
3.  Effect  of  law  on  morality    
 
4.  Effect  of  law  on  religion      
 

    Linked  Answer  Question:  


  Direction:  Choose  the  most  appropriate  option  in  the  following  questions:    
  Sub  Question  No  :  18    
 
Q.18   Which  theory  of  law  is  known  as  "Vienna  School"  of  legal  thought?          
     
 

  Ans      
1.  Pure    
 
2.  Natural    
 
3.  Historical    
 
 
4.  Philosophical      
    Linked  Answer  Question:  
  Direction:  Choose  the  most  appropriate  option  in  the  following  questions:    
  Sub  Question  No  :  19    
 
Q.19   Which  of  the  following  is  not  a  characteristic  of  sovereignty  as  per  Austin's  analysis?          
     
 

  Ans      
1.  Essentiality    
 
2.  Indivisibility    
 
3.  Illimitability    
 
4.  Divinity      
 

    Linked  Answer  Question:  


  Direction:  Choose  the  most  appropriate  option  in  the  following  questions:    
  Sub  Question  No  :  20    
 
Q.20   Who  said  that  the  task  of  law  is  'social  engineering'?          
     
 

  Ans      
1.  Durkheim    
 
2.  Roscoe  Pound    
 
3.  Henry  Marne    
 
4.  Ehrlich      
 

    Linked  Answer  Question:  


  Direction:  Choose  the  most  appropriate  option  in  the  following  questions:    
  Sub  Question  No  :  21    
 
Q.21   "Ignorantia  facit  execusat"  means:  
             
 

     
Ans  
1.  Ignorance  of  fact  is  no  excuse    
 
2.  Ignorance  of  law  is  excuse    
 
3.  Ignorance  of  law  is  no  excuse    
 
 
4.  Ignorance  of  fact  is  excuse      
    Linked  Answer  Question:  
  Direction:  Choose  the  most  appropriate  option  in  the  following  questions:    
  Sub  Question  No  :  22    
 
Q.22   Who  was  the  author  of  "The  Province  of  Jurisprudence  Determined"?          
     
 

  Ans      
1.  Kelsen    
 
2.  Dias    
 
3.  Austin    
 
4.  Salmond      
 

    Linked  Answer  Question:  


  Direction:  Choose  the  most  appropriate  option  in  the  following  questions:    
  Sub  Question  No  :  23    
 
Q.23   In  Pure  Theory  of  Law,  a  legal  norm  derives  its  validity  from:          
     
 

  Ans      
1.  Sovereign    
 
2.  Higher  Legal  Norm    
 
3.  Higher  Moral  Norm    
 
4.  Higher  Social  Norm      
 

    Linked  Answer  Question:  


  Direction:  Choose  the  most  appropriate  option  in  the  following  questions:    
  Sub  Question  No  :  24    
 
Q.24   Who  said  that  "the  essence  of  legal  right  seems  to  me  to  be  not  legally  guaranteed  power  to  realize  an  interest"?          
     
 

  Ans      
1.  Salmond    
 
2.  Allen    
 
3.  Hegel    
 
 
4.  Paton      
    Linked  Answer  Question:  
  Direction:  Choose  the  most  appropriate  option  in  the  following  questions:    
  Sub  Question  No  :  25    
 
Q.25   Fill  in  the  blanks  by  choosing  the  correct  option:          
     
 

_________________  justice  prescribes  equality.  There  has  to  be  equal  distribution  among  equals,  unequal  distribution  among  unequals.  _____________    
justice  seeks  to  restore  equality,  when  this  has  been  shaken,  on  the  assumption  that  the  situation  that  has  been  upset  was  distributively  first.    

  Ans      
1.  Distributive,  Corrective    
 
2.  Classic,  Corrective    
 
3.  Particular,  Distributive    
 
 
4.  Universal,  Classic      
    Linked  Answer  Question:  
  Direction:  Choose  the  most  appropriate  option  in  the  following  questions:    
  Sub  Question  No  :  26    
 
Q.26   In  India,  which  of  the  following  cannot  logically  be  said  to  be  the  manifestation  of  "VOLKSGEIST"?          
     
 

  Ans      
1.  GURUKUL  Education    
 
2.  Community  Labour  (SHRAMDAN)    
 
3.  Untouchability    
 
 
4.  Panchayat  system      
    Linked  Answer  Question:  
  Direction:  Choose  the  most  appropriate  option  in  the  following  questions:    
  Sub  Question  No  :  27    
 
Q.27   Who  stated  that  "I  recognize  without  hesitation  that  judges  must  and  do  legislate,  but  they  do  so  only  interstitially;  they  are  confined  from  molar  to          
molecular  motions"?        
 

  Ans      
1.  Justice  Holmes    
 
2.  Grey    
 
3.  Kant    
 
4.  Justice  Cardozo      
 

    Linked  Answer  Question:  


  Direction:  Choose  the  most  appropriate  option  in  the  following  questions:    
  Sub  Question  No  :  28    
 
Q.28   According  to  Kelsen          
     
 

  Ans      
1.  Law  is  to  serve  social  purpose    
 
2.  Law  is  a  means  not  an  end    
 
3.  Law  is  the  command  of  the  sovereign    
 
4.  Law  is  a  normative  science      
 

    Linked  Answer  Question:  


  Direction:  Choose  the  most  appropriate  option  in  the  following  questions:    
  Sub  Question  No  :  29    
 
Q.29   Four  elements  of  State  are:          
     
 

  Ans      
1.  Population,  Territory,  Sovereignty  and  Divinity    
 
2.  Population,  Territory,  Government  and  Sovereignty    
 
3.  Population,  Sovereignty  and  Territory  and  Equality    
 
 
4.  Sovereignty,  Government  and  Territory  and  Morality      
    Linked  Answer  Question:  
  Direction:  Choose  the  most  appropriate  option  in  the  following  questions:    
  Sub  Question  No  :  30    
 
Q.30   Savigny,  Salmond  and  Dicey  were  the  main  supporters  of  which  theory  of  personality?          
     
 

  Ans      
1.  Organic  theory    
 
2.  Concession  theory    
 
3.  Bracket  theory    
 
 
4.  Fiction  theory      
    Linked  Answer  Question:  
  Direction:  Choose  the  most  appropriate  option  in  the  following  questions:    
  Sub  Question  No  :  31    
 
Q.31   According  to  Hohfeld  what  is  the  "jural  opposite"  of  power?          
     
 

  Ans      
1.  Right    
 
2.  Liability    
 
3.  Disability    
 
4.  Duty      
 

    Linked  Answer  Question:  


  Direction:  Choose  the  most  appropriate  option  in  the  following  questions:    
  Sub  Question  No  :  32    
 
Q.32   Who  is  the  author  of  the  famous  work  "Law  in  the  Making"?  
             
 

  Ans      
1.  C.K.  Allen    
 
2.  Kelsen    
 
3.  Upendra  Baxi    
 
 
4.  Henry  Maine      
    Linked  Answer  Question:  
  Direction:  Choose  the  most  appropriate  option  in  the  following  questions:    
  Sub  Question  No  :  33    
 
Q.33   According  to  Hohfeld  if  'X'  has  a  power,  'Y'  has  a  liability.  A  liability  in  'Y'  means  the  absence  of  immunity  in  him.  Therefore,:          
     
 

  Ans      
1.  There  is  no  relation  between  power  and  liability.    
 
2.  Power  and  liability  are  "jural  contradicts".    
 
3.  Power  and  liability  are  "jural  opposites".    
 
 
4.  Power  and  liability  are  "jural  correlatives.      
    Linked  Answer  Question:  
  Direction:  Choose  the  most  appropriate  option  in  the  following  questions:    
  Sub  Question  No  :  34    
 
Q.34   Salmond  distinguished  between  "corporeal  possession"  and  "incorporeal  possession".  According  to  him  corporeal  possession  refers  to:          
     
 

  Ans      
1.  Possession  of  physical  objects    
 
2.  Possession  of  rights    
 
3.  intention  to  possess  physical  objects    
 
4.  intention  to  possess  rights      
 

    Linked  Answer  Question:  


  Direction:  Choose  the  most  appropriate  option  in  the  following  questions:    
  Sub  Question  No  :  35    
 
Q.35   Who  defined  law  as  'a  social  institution  to  satisfy  social  wants"?          
     
 

  Ans      
1.  Duguit    
 
2.  Ihering    
 
3.  Pound    
 
4.  Kelsen      
 

    Linked  Answer  Question:  


  Direction:  Choose  the  most  appropriate  option  in  the  following  questions:    
  Sub  Question  No  :  36    
 
Q.36   Who  said  that  "Jurisprudence  was  the  first  of  the  social  sciences  to  be  born"?          
     
 

  Ans      
1.  Savigny    
 
2.  Duguit    
 
3.  Wurzel    
 
 
4.  Hegel      
    Linked  Answer  Question:  
  Direction:  Choose  the  most  appropriate  option  in  the  following  questions:    
  Sub  Question  No  :  37    
 
Q.37   Who  summarized  Austin's  thesis  by  stating  that  "this,  at  first  sight,  looks  like  a  circular  reasoning.  Law  is  law  since  it  is  made  by  the  Sovereign.  The  Sovereign      
       
is  Sovereign  because  he  makes  the  law"?    
 

  Ans      
1.  Fuller    
 
2.  Buckland    
 
3.  Holland    
 
 
4.  Hart      
    Linked  Answer  Question:  
  Direction:  Choose  the  most  appropriate  option  in  the  following  questions:    
  Sub  Question  No  :  38    
 
Q.38   Who  used  the  "social  solidarity"  as  a  criterion  of  the  validity  of  Laws?          
     
 

  Ans      
1.  Duguit    
 
2.  Pound    
 
3.  Kelsen    
 
4.  Ihering      
 

    Linked  Answer  Question:  


  Direction:  Choose  the  most  appropriate  option  in  the  following  questions:    
  Sub  Question  No  :  39    
 
Q.39   X  was  allowed  to  put  her  goods  in  certain  rooms  in  Y's  house.  X  sent  them  by  agent,  who  locked  them  in  the  rooms  allotted  for  that  purpose  in  Y's  house  by      
       
Y,  and  took  away  the  key.    
 

  Ans      
1.  X  was  in  possession  of  the  rooms    
 
2.  The  agent  of  'X'  was  in  possession  of  the  rooms    
 
3.  The  agent  of  'X'  and  'Y'  were  in  possession  of  the  rooms.    
 
4.  'Y'  was  in  possession  of  the  rooms      
 

    Linked  Answer  Question:  


  Direction:  Choose  the  most  appropriate  option  in  the  following  questions:    
  Sub  Question  No  :  40    
 
Q.40   According  to  Analytical  School,  pre-­‐existence  of  which  of  the  following  is  necessary  for  the  growth  of  Positive  Law?          
     
 

  Ans      
1.  Family    
 
2.  State    
 
3.  Tribe    
 
 
4.  Clan      
    Linked  Answer  Question:  
  Direction:  Choose  the  most  appropriate  option  in  the  following  questions:    
  Sub  Question  No  :  41    
 
Q.41   Which  school  regards  the  judge  made  law  as  "an  unauthorized  encroachment  upon  the  powers  of  the  legislators  to  make  law"?          
     
 

  Ans      
1.  Realist  school    
 
2.  Historical  school    
 
3.  Sociological  school    
 
 
4.  Analytical  school      
    Linked  Answer  Question:  
  Direction:  Choose  the  most  appropriate  option  in  the  following  questions:    
  Sub  Question  No  :  42    
 
Q.42   What  is  the  correlative  of  "Immunity",  according  to  Hohfeld?          
     
 

  Ans      
1.  Duty    
 
2.  Liability    
 
3.  Disability    
 
4.  Power      
 

    Linked  Answer  Question:  


  Direction:  Choose  the  most  appropriate  option  in  the  following  questions:    
  Sub  Question  No  :  43    
 
Q.43   American  Realist  movement  is  a  combination  of  the:  
             
 

  Ans      
1.  natural  Law  positivist  and  sociological  approaches    
 
2.  analytical  positivist  and  sociological  approaches    
 
3.  historical  positivist  and  sociological  approaches    
 
 
4.  natural  Law  positivist  and  historical  approaches      
    Linked  Answer  Question:  
  Direction:  Choose  the  most  appropriate  option  in  the  following  questions:    
  Sub  Question  No  :  44    
 
Q.44   Right  to  one's  Reputation  is  a:          
     
 

  Ans      
1.  right  in  rem    
 
2.  right  in  personam    
 
3.  an  easement  right    
 
 
4.  property  right      
    Linked  Answer  Question:  
  Direction:  Choose  the  most  appropriate  option  in  the  following  questions:    
  Sub  Question  No  :  45    
 
Q.45   Cicero  defined  law  as:          
     
 

  Ans      
1.  The  highest  reason  implanted  in  nature.    
 
2.  The  aggregate  of  rules  set  by  men  or  politically  superior,  or  sovereign,  to  men  or  politically  subject.    
 
3.  A  social  institution  to  satisfy  social  wants.    
 
4.  The  form  of  the  guarantee  of  the  conditions  of  life  of  society  assured  by  state's  power  of  constraint.      
 

    Linked  Answer  Question:  


  Direction:  Choose  the  most  appropriate  option  in  the  following  questions:    
  Sub  Question  No  :  46    
 
Q.46   Who  is  the  propounder  of  "Bracket  theory"  of  personality?          
     
 

  Ans      
1.  Ihering    
 
2.  Baker    
 
3.  Savigny    
 
4.  Salmond      
 

    Linked  Answer  Question:  


  Direction:  Choose  the  most  appropriate  option  in  the  following  questions:    
  Sub  Question  No  :  47    
 
Q.47   That  "Life  of  Law  has  not  been  logic,  it  has  been  experience"  was  observed  by:          
     
 

  Ans      
1.  Hohfield    
 
2.  Holmes    
 
3.  Holland    
 
 
4.  Henry  Maine      
    Linked  Answer  Question:  
  Direction:  Choose  the  most  appropriate  option  in  the  following  questions:    
  Sub  Question  No  :  48    
 
Q.48   Who  is  the  author  of  the  famous  book  "The  Concept  of  Law"?          
     
 

  Ans      
1.  Kelsen    
 
2.  Upendra  Baxi    
 
3.  Austin    
 
 
4.  Hart      
    Linked  Answer  Question:  
  Direction:  Choose  the  most  appropriate  option  in  the  following  questions:    
  Sub  Question  No  :  49    
 
Q.49   Who  is  called  the  "father  of  English  Jurisprudence"?          
     
 

  Ans      
1.  Kelsen    
 
2.  John  Austin    
 
3.  Ihering    
 
4.  Justice  Holmes      
 

    Linked  Answer  Question:  


  Direction:  Choose  the  most  appropriate  option  in  the  following  questions:    
  Sub  Question  No  :  50    
 
Q.50   The  reaction  against  which  law  theories  provided  a  rich  bed  in  which  the  seeds  of  historical  scholarship  took  root  and  spread?          
     
 

  Ans      
1.  Realist  law  theories    
 
2.  Natural  law  theories    
 
3.  Analytical  law  theories    
 
4.  Pure  law  theories      
 

Section : Other Subjects such as Contract , Torts , Criminal Law , International Law, IPR etc
 
  Linked  Answer  Question:  
  Direction:  Choose  the  most  appropriate  option  in  the  following  questions:    
  Sub  Question  No  :  1    
 
Q.1   'A'  signs  his  own  name  to  a  bill  of  exchange,  intending  that  it  may  be  believed  that  the  bill  was  drawn  by  another  person  of  the  same  name.  'A'  commits:          
     
 

  Ans      
1.  attempt  of  cheating    
 
2.  forgery    
 
3.  attempt  of  forgery    
 
 
4.  cheating      
    Linked  Answer  Question:  
  Direction:  Choose  the  most  appropriate  option  in  the  following  questions:    
  Sub  Question  No  :  2    
 
Q.2   Article  51  of  the  UN  Charter  saves  which  of  the  following  rights  as  being  inherent?          
     
 

  Ans      
1.  Right  to  free  trade  with  member-­‐States    
 
2.  Right  to  enter  into  relations  with  other  States    
 
3.  Right  of  self-­‐defence    
 
4.  Right  of  self-­‐determination      
 

    Linked  Answer  Question:  


  Direction:  Choose  the  most  appropriate  option  in  the  following  questions:    
  Sub  Question  No  :  3    
 
Q.3   A,  being  executor  to  the  will  of  a  deceased  person,  dishonestly  disobeys  the  law  which  directs  him  to  divide  the  assets  according  to  the  will,  and  appropriates      
       
the  assets  to  his  own  use.  A  commits:    
 

  Ans      
1.  criminal  breach  of  trust    
 
2.  theft    
 
3.  cheating    
 
4.  forgery      
 

    Linked  Answer  Question:  


  Direction:  Choose  the  most  appropriate  option  in  the  following  questions:    
  Sub  Question  No  :  4    
 
Q.4   Under  the  Patents  Act,  1970  a  patent  is  granted  for:          
     
 

  Ans      
1.  invention    
 
2.  discovery    
 
3.  observation    
 
4.  finding      
 

    Linked  Answer  Question:  


  Direction:  Choose  the  most  appropriate  option  in  the  following  questions:    
  Sub  Question  No  :  5    
 
Q.5   'A'  gives  authority  to  'B'  to  sell  A's  land  and  to  pay  himself,  out  of  the  proceeds,  the  debts  due  to  him  from  'A':          
     
 

  Ans      
1.  A  cannot  revoke  this  authority.    
 
2.  The  authority  can  be  terminated  after  the  death  of  'A'.    
 
3.  A  can  revoke  this  authority  subsequently.    
 
 
4.  The  authority  can  be  terminated  on  the  ground  of  insanity  of  'A'.      
    Linked  Answer  Question:  
  Direction:  Choose  the  most  appropriate  option  in  the  following  questions:    
  Sub  Question  No  :  6    
 
Q.6   'A'  voluntarily  throws  into  a  river  a  ring  belonging  to  'Z',  with  the  intention  of  thereby  causing  wrongful  loss  to  'Z'.  'A'  has  committed:          
     
 

  Ans      
1.  extortion    
 
2.  theft    
 
3.  cheating    
 
4.  mischief      
 

    Linked  Answer  Question:  


  Direction:  Choose  the  most  appropriate  option  in  the  following  questions:    
  Sub  Question  No  :  7    
 
Q.7   The  Judges  of  International  Court  of  Justice  are  elected  for  nine  years  and  the  court  shall  elect  its  President  or  Vice-­‐President  for  a  term  of:          
     
 

  Ans      
1.  Nine  years    
 
2.  Three  years    
 
3.  Five  years    
 
4.  Seven  years      
 

    Linked  Answer  Question:  


  Direction:  Choose  the  most  appropriate  option  in  the  following  questions:    
  Sub  Question  No  :  8    
 
Q.8   A  married  couple  has  adopted  a  son  under  the  provisions  of  the  Hindu  Adoptions  and  Maintenance  Act,  1956.  While  the  adopted  son  is  alive,  the  couple  is          
willing  to  adopt  another  son  as  the  said  adopted  son  was  not  medically  fit  to  produce  a  child.  The  couple-­‐        
 

  Ans      
1.  can  adopt  another  son  with  the  permission  of  the  Court    
 
2.  cannot  adopt  another  son    
 
3.  can  adopt  another  son  with  the  consent  of  the  adopted  son    
 
4.  can  adopt  another  son      
 

    Linked  Answer  Question:  


  Direction:  Choose  the  most  appropriate  option  in  the  following  questions:    
  Sub  Question  No  :  9    
 
Q.9   A  nine  year  old  boy  bought  petrol  in  a  Can  from  the  defendant,  a  petrol  dealer  by  falsely  stating  that  his  mother  needed  it  for  her  car.  In  fact,  he  used  it  to          
play  with  it,  and,  in  doing  so,  sustained  burn  injuries.  The  defendant  is:        
 

  Ans      
1.  there  was  contributory  negligence  of  the  boy,  therefore  defendant  is  not  liable.    
 
2.  not  liable  in  negligence  because  possession  of  the  Can  and  money  proved  that  boy  acted  as  a  reasonable  person.    
 
3.  not  liable  in  negligence  as  the  boy  had  made  a  false  statement  and  the  defendant  believed  it  to  be  true.    
 
 
4.  liable  in  negligence  for  supplying  petrol  to  so  young  a  boy  who  was  not  expected  to  know  the  properties  of  petrol.      
    Linked  Answer  Question:  
  Direction:  Choose  the  most  appropriate  option  in  the  following  questions:    
  Sub  Question  No  :  10    
 
Q.10   A,  B,  C,  D,  and  E,  while  carrying  away  property  obtained  by  theft  committed  in  a  house  fired  shots  in  the  air  with  a  view  to  scare  away  the  inmates,  from          
pursuing  them.  They  are  guilty  of:        
 

  Ans      
1.  extortion    
 
2.  dacoity    
 
3.  roit    
 
4.  theft      
 

    Linked  Answer  Question:  


  Direction:  Choose  the  most  appropriate  option  in  the  following  questions:    
  Sub  Question  No  :  11    
 
Q.11   Who  said  that  the  Public  International  Law  is  "a  Vanishing  Point  of  Jurisprudence"?  
             
 

  Ans      
1.  Fawcett    
 
2.  Brierly    
 
3.  Holland    
 
4.  Grotius      
 

    Linked  Answer  Question:  


  Direction:  Choose  the  most  appropriate  option  in  the  following  questions:    
  Sub  Question  No  :  12    
 
Q.12   'A'  supplies  to  the  dependent  wife  and  children  of  a  lunatic  'B',  necessities  suitable  to  their  needs  and  conditions  of  life.  'A'  is:          
     
 

  Ans      
1.  entitled  to  reimbursement  from  B's  property.    
 
2.  entitled  to  partial  reimbursement  only.    
 
3.  not  entitled  to  any  reimbursement.    
 
 
4.  entitled  to  reimbursement  at  the  discretion  of  'B'.      
    Linked  Answer  Question:  
  Direction:  Choose  the  most  appropriate  option  in  the  following  questions:    
  Sub  Question  No  :  13    
 
Q.13   A'  agrees  to  buy  from  'B'  a  certain  horse.  It  turns  out  that  horse  was  dead  at  the  time  of  the  bargain,  though  neither  party  was  aware  of  the  fact.  The          
agreement  is:        
 

  Ans      
1.  valid  for  claim  of  compensation    
 
2.  voidable    
 
3.  valid    
 
 
4.  void      
    Linked  Answer  Question:  
  Direction:  Choose  the  most  appropriate  option  in  the  following  questions:    
  Sub  Question  No  :  14    
 
Q.14   An  accused  head  master  of  a  school  threatens  a  lady  teacher  to  sign  certain  blank  papers  and  if  she  does  not  do  so,  he  would  defame  her.  The  head  master  is      
guilty  of:            
 

  Ans      
1.  extortion    
 
2.  attempt  to  commit  defamation    
 
3.  criminal  intimidation    
 
 
4.  robbery      
    Linked  Answer  Question:  
  Direction:  Choose  the  most  appropriate  option  in  the  following  questions:    
  Sub  Question  No  :  15    
 
Q.15   The  essential  tests  for  the  existence  of  international  custom  are:          
     
 

  Ans      
1.  uniform  states  practice  and  opinio  juris  sive  necessitatis    
 
2.  ancient  nature    
 
3.  ancient  nature,  uniform  states  practice  and  opinio  juris  sive  necessitatis    
 
4.  ancient  nature,  uniform  states  practice      
 

    Linked  Answer  Question:  


  Direction:  Choose  the  most  appropriate  option  in  the  following  questions:    
  Sub  Question  No  :  16    
 
Q.16   Indian  Copyright  Act,  1957  protects  the  literary  work  for  following  term:  
             
 

  Ans      
1.  Life  of  author  plus  25  Years  after  his/her  death    
 
2.  Full  life  only    
 
3.  Life  of  author  plus  50  Years  after  his/her  death    
 
4.  Life  of  author  plus  60  Years  after  his  /her  death      
 

    Linked  Answer  Question:  


  Direction:  Choose  the  most  appropriate  option  in  the  following  questions:    
  Sub  Question  No  :  17    
 
Q.17   In  his  definition  of  'Law  of  Tort',  Winfield  supports  the  principle  given  in  the  maxim:          
     
 

  Ans      
1.  Ubi  remedium  ibi  jus    
 
2.  Ubi  jus  ibi  remedium    
 
3.  Respondeat  superior    
 
 
4.  Damnum  sine  injuria      
    Linked  Answer  Question:  
  Direction:  Choose  the  most  appropriate  option  in  the  following  questions:    
  Sub  Question  No  :  18    
 
Q.18   Emission  Trading  mechanism  is  a  market  based  mechanism  provided  in:          
     
 

  Ans      
1.  Warsaw  Amendment,  2013    
 
2.  Kyoto  Protocol,  1997    
 
3.  U.N.  Convention  on  Climate  Change,  1992    
 
4.  Lima  Agreement,  2014      
 

    Linked  Answer  Question:  


  Direction:  Choose  the  most  appropriate  option  in  the  following  questions:    
  Sub  Question  No  :  19    
 
Q.19   Basic  principles  of  award  of  compensation  for  breach  of  contract  were  laid  down  in:          
     
 

  Ans      
1.  Hadley  v.  Baxendale    
 
2.  Solomon  v.  Solomon    
 
3.  Harvey  v.  Facie    
 
4.  Carllil  v.  Carbolic  Smoke  Ball  Co.      
 

    Linked  Answer  Question:  


  Direction:  Choose  the  most  appropriate  option  in  the  following  questions:    
  Sub  Question  No  :  20    
 
Q.20   'A'  lets  to  'B',  for  hire,  a  horse  for  his  riding.  'B'  uses  the  horse  to  drive  the  carriage:  
             
 

     
Ans  
1.  The  bailment  is  automatically  terminated.    
 
2.  The  termination  of  bailment  is  at  the  option  of  'A'.    
 
3.  The  bailment  can  be  terminated  at  the  option  of  'B'.    
 
 
4.  The  bailment  cannot  be  terminated.      
    Linked  Answer  Question:  
  Direction:  Choose  the  most  appropriate  option  in  the  following  questions:    
  Sub  Question  No  :  21    
 
Q.21   'A'  inserts  his  finger  into  the  anus  of  a  woman  against  her  will.  Under  which  one  of  the  following  Sections  of  the  Indian  Penal  Code,  1860  'A'  is  punishable?          
     
 

  Ans      
1.  Section  377    
 
2.  Section  354    
 
3.  Section  376    
 
4.  Section  509      
 

    Linked  Answer  Question:  


  Direction:  Choose  the  most  appropriate  option  in  the  following  questions:    
  Sub  Question  No  :  22    
 
Q.22   For  the  offence  of  cheating,  which  one  of  the  following  state(s)  of  guilty  mind/s  is/are  to  be  proved?          
     
 

  Ans      
1.  Fraudulently  or  dishonestly    
 
2.  Dishonestly    
 
3.  Fraudulently    
 
4.  Fraudulently  and  dishonestly      
 

    Linked  Answer  Question:  


  Direction:  Choose  the  most  appropriate  option  in  the  following  questions:    
  Sub  Question  No  :  23    
 
Q.23   Who  is  called  father  of  Public  International  Law?          
     
 

  Ans      
1.  Grotius    
 
2.  Holland    
 
3.  Starke    
 
 
4.  Oppenheim      
    Linked  Answer  Question:  
  Direction:  Choose  the  most  appropriate  option  in  the  following  questions:    
  Sub  Question  No  :  24    
 
Q.24   A'  proposes,  by  letter,  to  sell  a  house  to  'B'  at  a  certain  price.  'B'  accepts  the  proposal  of  'A'  by  posting  a  letter  to  'A'.  Select  the  correct  option:          
     
 

  Ans      
1.  There  cannot  be  an  agreement  between  'A'  and  'B'  by  post.    
 
2.  The  communication  of  the  acceptance  is  complete  against  'B'  when  the  letter  is  posted  by  'B'.    
 
3.  The  communication  of  the  acceptance  is  complete  against  'B'  when  the  letter  is  received  by  'A'.    
 
 
4.  The  communication  of  the  proposal  is  complete  when  letter  of  proposal  was  posted  by  'A'.      
    Linked  Answer  Question:  
  Direction:  Choose  the  most  appropriate  option  in  the  following  questions:    
  Sub  Question  No  :  25    
 
Q.25   India  acceded  to  the  International  Covenant  on  Civil  and  Political  Rights  in:          
     
 

  Ans      
1.  1972    
 
2.  1966    
 
3.  1979    
 
 
4.  1976      
    Linked  Answer  Question:  
  Direction:  Choose  the  most  appropriate  option  in  the  following  questions:    
  Sub  Question  No  :  26    
 
Q.26   Under  the  Copyright  Act,  1957  copyright  can  be  granted  to?          
     
 

  Ans      
1.  invention    
 
2.  handicrafts    
 
3.  design  of  a  bottle    
 
4.  literary,  dramatic,  musical  and  artistic  work      
 

    Linked  Answer  Question:  


  Direction:  Choose  the  most  appropriate  option  in  the  following  questions:    
  Sub  Question  No  :  27    
 
Q.27   Which  one  of  the  following  is  not  a  defence  to  the  strict  liability  rule  in  tort?  
             
 

  Ans      
1.  Plaintiff's  own  default.    
 
2.  Inevitable  accident.    
 
3.  Act  of  God.    
 
4.  Escape  of  dangerous  things  through  the  unforeseeable  act  of  a  stranger.      
 

    Linked  Answer  Question:  


  Direction:  Choose  the  most  appropriate  option  in  the  following  questions:    
  Sub  Question  No  :  28    
 
Q.28   Which  one  of  the  following  is  not  an  element  of  the  tort  of  malicious  prosecution?          
     
 

  Ans      
1.  Prosecution  of  the  plaintiff  by  the  defendant.    
 
2.  Damage  to  the  plaintiff  due  to  malicious  prosecution.    
 
3.  Termination  of  the  criminal  case  in  favour  of  the  defendant.    
 
 
4.  Absence  of  reasonable  and  probable  cause  for  prosecution.      
    Linked  Answer  Question:  
  Direction:  Choose  the  most  appropriate  option  in  the  following  questions:    
  Sub  Question  No  :  29    
 
Q.29   Which  one  of  the  following  is  accepted  as  mens  rea  for  constituting  the  offence  of  theft?          
     
 

  Ans      
1.  voluntarily    
 
2.  Dishonestly    
 
3.  Fraudulently    
 
4.  knowingly      
 

    Linked  Answer  Question:  


  Direction:  Choose  the  most  appropriate  option  in  the  following  questions:    
  Sub  Question  No  :  30    
 
Q.30   Where  the  principle  of  "Common  but  differentiated  responsibilities"  finds  a  place?          
     
 

  Ans      
1.  Paris  Agreement,  2015    
 
2.  Montreal  Protocol,  1987    
 
3.  U.N.  Convention  on  Biodiversity,  1992    
 
4.  Warsaw  Amendment,  2013      
 

    Linked  Answer  Question:  


  Direction:  Choose  the  most  appropriate  option  in  the  following  questions:    
  Sub  Question  No  :  31    
 
Q.31   When,  U.N.  Human  Rights  Council,  which  is  successor  to  the  U.N.  Commission  on  Human  Rights,  was  established?  
             
 

     
Ans  
1.  2010    
 
2.  2012    
 
3.  2013    
 
 
4.  2006      
    Linked  Answer  Question:  
  Direction:  Choose  the  most  appropriate  option  in  the  following  questions:    
  Sub  Question  No  :  32    
 
Q.32   In  execution  of  a  decree  for  the  maintenance,  salary  of  a  person  can  be  attached  to  the  extent  of:          
     
 

  Ans      
1.  two  third    
 
2.  one  half    
 
3.  one  fourth    
 
4.  one  third      
 

    Linked  Answer  Question:  


  Direction:  Choose  the  most  appropriate  option  in  the  following  questions:    
  Sub  Question  No  :  33    
 
Q.33   If  International  Law  were  only  a  kind  of  morality,  the  framers  of  State  Papers  Concerning  Foreign  Policy  would  throw  all  their  strength  on  moral  argument.          
Who  made  this  observation?        
 

  Ans      
1.  Fitzmaurice    
 
2.  J.G.  Starke    
 
3.  John  Austin    
 
4.  Frederick  Pollock      
 

    Linked  Answer  Question:  


  Direction:  Choose  the  most  appropriate  option  in  the  following  questions:    
  Sub  Question  No  :  34    
 
Q.34   The  maxim  'qui  facit  per  alium  facit  per  se'  means:          
     
 

  Ans      
1.  he  who  acts  through  others  is  deemed  in  law  as  doing  it  himself.    
 
2.  responsibility  must  not  be  that  of  the  superior.    
 
3.  a  person  is  responsible  for  his  wrongful  acts.    
 
 
4.  an  agent  is  not  liable  to  principal.      
    Linked  Answer  Question:  
  Direction:  Choose  the  most  appropriate  option  in  the  following  questions:    
  Sub  Question  No  :  35    
 
Q.35   The  International  Criminal  Court  was  established  by:          
     
 

  Ans      
1.  Rome  Statute,  1998    
 
2.  UN  Charter,  1945    
 
3.  League  of  Nations,  1919    
 
 
4.  Hague  Conference,  1907      
    Linked  Answer  Question:  
  Direction:  Choose  the  most  appropriate  option  in  the  following  questions:    
  Sub  Question  No  :  36    
 
Q.36   As  per  The  Copyright  Act  1957,  who  is  not  included  in  the  definition  of  Performer?          
     
 

  Ans      
1.  Snake  charmer    
 
2.  Acrobat    
 
3.  Cobbler    
 
 
4.  Singer      
    Linked  Answer  Question:  
  Direction:  Choose  the  most  appropriate  option  in  the  following  questions:    
  Sub  Question  No  :  37    
 
Q.37   'A'  agrees  to  sell  his  house  worth  Rs.  5  lac  for  Rs.  1000/-­‐.  'A'  denies  that  his  consent  to  the  agreement  was  freely  given.          
     
 

  Ans      
1.  The  contract  is  void  because  of  inadequacy  of  consideration.    
 
2.  'A'  cannot  take  this  plea  after  the  agreement.    
 
3.  The  court  should  take  into  account  the  inadequacy  of  the  consideration  in  considering  whether  or  not  'A's  consent  was  freely  given.    
 
4.  The  court  cannot  take  into  account  the  inadequacy  of  the  consideration  in  considering  whether  or  not  'A's  consent  was  freely  given.      
 

    Linked  Answer  Question:  


  Direction:  Choose  the  most  appropriate  option  in  the  following  questions:    
  Sub  Question  No  :  38    
 
Q.38   The  Supreme  Court  in  the  case  of  Novartis,  AG  v.  Union  of  India,  rejected  the  patent  of  which  drug?  
             
 

  Ans      
1.  Macsarin    
 
2.  Gleevec    
 
3.  Zimmerman    
 
4.  Aspirin      
 

    Linked  Answer  Question:  


  Direction:  Choose  the  most  appropriate  option  in  the  following  questions:    
  Sub  Question  No  :  39    
 
Q.39   'A'  and  'B'  agree  that  'A'  shall  pay  Rs.  20000  for  which  'B'  shall  deliver  either  rice  or  heroin.  The  contract  is:          
     
 

  Ans      
1.  void  as  to  delivery  of  heroin  and  valid  as  to  delivery  of  rice    
 
2.  void    
 
3.  voidable    
 
 
4.  valid      
    Linked  Answer  Question:  
  Direction:  Choose  the  most  appropriate  option  in  the  following  questions:    
  Sub  Question  No  :  40    
 
Q.40   An  invention  under  the  Patent  in  India  can  be  protected  for  a  term  of  -­‐          
     
 

  Ans      
1.  20  Years    
 
2.  50  Years    
 
3.  10  Years    
 
4.  25  Years      
 

    Linked  Answer  Question:  


  Direction:  Choose  the  most  appropriate  option  in  the  following  questions:    
  Sub  Question  No  :  41    
 
Q.41   In  which  of  the  following  case,  the  Supreme  Court  of  India  held  that  'if  a  Hindu  husband  converts  into  Islam  and  marries  again,  he  will  be  guilty  of  bigamy'?          
     
 

  Ans      
1.  Revanasiddappa  v.  Mallikarjun  AIR  2011  SC  (Supp)  155    
 
2.  Bhaurao  Shanker  Lokhande  v.  State  of  Maharashtra  AIR  1965  SC  1564    
 
3.  Sarla  Mudgal  v.  Union  of  India  AIR  1995  SC  1531    
 
4.  Bharatha  Matha  v.  R.  Vijaya  Renganathan  AIR  2010  SC  2685      
 

    Linked  Answer  Question:  


  Direction:  Choose  the  most  appropriate  option  in  the  following  questions:    
  Sub  Question  No  :  42    
 
Q.42   'A'  dies  leaving  behind  a  son  X  and  a  married  daughter  Y,  a  suit  filed  by  'A',  can  be  continued  by:  
             
 

     
Ans  
1.  'X'  alone  as  legal  representative    
 
2.  'Y'  alone  as  legal  representative    
 
3.  'X',  'Y'  and  the  husband  of  Y  as  legal  representatives    
 
 
4.  'X'  and  'Y'  both,  as  legal  representatives      
    Linked  Answer  Question:  
  Direction:  Choose  the  most  appropriate  option  in  the  following  questions:    
  Sub  Question  No  :  43    
 
Q.43   Which  one  of  the  following  essentials  for  seeking  claim  of  patent  is  wrong?          
     
 

  Ans      
1.  Compulsory  Licensing    
 
2.  Non-­‐obviousness    
 
3.  Novelty    
 
4.  Industrial  Utility      
 

    Linked  Answer  Question:  


  Direction:  Choose  the  most  appropriate  option  in  the  following  questions:    
  Sub  Question  No  :  44    
 
Q.44   The  liability  to  pay  compensation,  of  an  enterprise  carrying  on  an  inherently  dangerous  and  hazardous  activity,  on  death  of  a  worker  due  to  leakage  of  lethal      
gas  is  based  on  the  principle  of:            
 

  Ans      
1.  fault  Liability    
 
2.  negligence    
 
3.  absolute  liability    
 
4.  wrongful  intention      
 

    Linked  Answer  Question:  


  Direction:  Choose  the  most  appropriate  option  in  the  following  questions:    
  Sub  Question  No  :  45    
 
Q.45   Which  body  of  the  United  Nations  requested  International  Court  of  Justice  to  give  an  advisory  opinion  on  the  issue  of  legality  of  use  or  threat  to  use  nuclear      
       
weapons  on  15  December  1994?    
 

  Ans      
1.  U.N.  General  Assembly    
 
2.  Security  Council    
 
3.  Economic  and  Social  Council    
 
4.  W.H.O.      
 

    Linked  Answer  Question:  


  Direction:  Choose  the  most  appropriate  option  in  the  following  questions:    
  Sub  Question  No  :  46    
 
Q.46   Which  element  is  not  required  in  the  tort  of  defamation?          
     
 

  Ans      
1.  Wrongful  intention    
 
2.  Defamatory  statement    
 
3.  Referring  to  the  Plaintiff    
 
 
4.  Publication      
    Linked  Answer  Question:  
  Direction:  Choose  the  most  appropriate  option  in  the  following  questions:    
  Sub  Question  No  :  47    
 
Q.47   For  fixing  vicarious  liability  of  master  in  tort  for  the  wrongs  committed  by  servant,  master  servant  relationship  is  determined  on  the  basis  of:          
     
 

  Ans      
1.  employment    
 
2.  payment  of  wages    
 
3.  conditions  of  service    
 
 
4.  control  of  master  over  servant's  acts      
    Linked  Answer  Question:  
  Direction:  Choose  the  most  appropriate  option  in  the  following  questions:    
  Sub  Question  No  :  48    
 
Q.48   Under  the  Indian  Contract  Act,  1872  a  finder  of  goods  is  subject  to  the  same  duties  as  a:          
     
 

  Ans      
1.  custodian    
 
2.  owner    
 
3.  bailee    
 
4.  trustee      
 

    Linked  Answer  Question:  


  Direction:  Choose  the  most  appropriate  option  in  the  following  questions:    
  Sub  Question  No  :  49    
 
Q.49   Which  one  of  the  following  is  not  an  essential  element  of  the  defence  of  fair  comment  in  tort  of  defamation?  
             
 

  Ans      
1.  The  comment  must  be  an  assertion  of  fact.    
 
2.  The  comment  must  be  fair.    
 
3.  The  matter  commented  on  must  be  of  public  interest.    
 
4.  The  comment  must  not  be  malicious.      
 

    Linked  Answer  Question:  


  Direction:  Choose  the  most  appropriate  option  in  the  following  questions:    
  Sub  Question  No  :  50    
 
Q.50   'A'  and  'B'  jointly  owe  Rs.  100/-­‐  to  'C'.  'A'  alone  pays  the  whole  amount  to  'C'  and  'B'  not  knowing  this  fact  also  pays  Rs.  100/-­‐  to  'C':          
     
 

  Ans      
1.  'C'  is  bound  to  repay  Rs.  100/-­‐  to  'B'.    
 
2.  C'  is  not  bound  to  repay  Rs.  100/-­‐  to  either  'A'  or  'B'  because  'B'  paid  on  his  own.    
 
3.  C'  is  bound  to  pay  Rs.  50/-­‐  to  'B'.    
 
 
4.  C'  is  bound  to  pay  Rs.  100/-­‐  each  to  'A'  and  'B'.      
 
Notations :

Question Paper Name: CLAT 2017 PG

Subject Name:
Creation Date: 2017-05-14 17:52:30
Duration: 120
Total Marks: 150
Display Marks: Yes
Calculator: None
Magnifying Glass Required?: No
Ruler Required?: No
Eraser Required?: No
Scratch Pad Required?: No
Rough Sketch/Notepad Required?: No
Protractor Required?: No

Group Number : 1
Group Id : 4611393
Group Maximum Duration : 0
Group Minimum Duration : 120
Revisit allowed for view? : No
Revisit allowed for edit? : No
Break time: 0
Mandatory Break time: No
Group Marks: 150

Section Id : 4611399
Section Number : 1
Section type : Online
Mandatory or Optional: Mandatory
Number of Questions: 1
Number of Questions to be attempted: 1
Section Marks: 50
Display Number Panel: Yes
Group All Questions: No

Sub-Section Number: 1
Sub-Section Id: 46113920
Question Shuffling Allowed : Yes
       
               
                          !   " # ! !  $ 
%   & '      (  '        

       
 

         (   ' 
)   *  +

   ,        

         (   '
       
              
           !   "
%   -    ! .           (   '
%      " ! 

               
         '     .    
0  '   1 . !
/

  ' '  1 
2 '   "
 3 4 *

      

         (   '
4        
    4           
           !   "
%   -    ! .           (   '
%      " ! 

               
         '     .    
0  '   1 . !
/

  ' '  1 
2 '   "
 3 4 *

      

         (   '
        
               
           !   "
%   -    ! .           (   '
%      " ! 

               
         '     .    
0  '   1 . !
/

  ' '  1 
2 '   "
 3 4 *

      

         (   '
       
              
           !   "
%   -    ! .           (   '
%      " ! 

               
         '     .    
0  '   1 . !
/

  ' '  1 
2 '   "
 3 4 *

      

         (   '
*        
    *           
           !   "
%   -    ! .           (   '
%      " ! 

               
         '     .    
0  '   1 . !
/

  ' '  1 
2 '   "
 3 4 *
      

         (   '
       
              
           !   "
%   -    ! .           (   '
%      " ! 

               
         '     .    
0  '   1 . !
/

  ' '  1 
2 '   "
 3 4 *

      

         (   '
        
               
           !   "
%   -    ! .           (   '
%      " ! 

               
         '     .    
0  '   1 . !
/

  ' '  1 
2 '   "
 3 4 *

      

         (   '
5        
    5           
           !   "
%   -    ! .           (   '
%      " ! 

               
         '     .    
0  '   1 . !
/

  ' '  1 
2 '   "
 3 4 *

      

         (   '
        
               
           !   "
%   -    ! .           (   '
%      " ! 

               
         '     .    
0  '   1 . !
/

  ' '  1 
2 '   "
 3 4 *

      

         (   '
        
   5            
           !   "
%   -    ! .           (   '
%  

   " !                 
         '     .    
0  '   1 . !
/

  ' '  1 
2 '   "
 3 4 *
      

         (   '
       
   5           
           !   "
%   -    ! .           (   '
%  

   " !                 
         '     .    
0  '   1 . !
/

  ' '  1 
2 '   "
 3 4 *

      

         (   '
4        
   5 4           
           !   "
%   -    ! .           (   '
%  

   " !                 
         '     .    
0  '   1 . !
/

  ' '  1 
2 '   "
 3 4 *

      

         (   '
        
   5            
           !   "
%   -    ! .           (   '
%  

   " !                 
         '     .    
0  '   1 . !
/

  ' '  1 
2 '   "
 3 4 *

      

         (   '
       
   5           
           !   "
%   -    ! .           (   '
%  

   " !                 
         '     .    
0  '   1 . !
/

  ' '  1 
2 '   "
 3 4 *

      

         (   '
*        
   5 *           
           !   "
%   -    ! .           (   '
%  

   " !                 
         '     .    
0  '   1 . !
/
  ' '  1 
2 '   "
 3 4 *

      

         (   '
       
   5           
           !   "
%   -    ! .           (   '
%  

   " !                 
         '     .    
0  '   1 . !
/

  ' '  1 
2 '   "
 3 4 *

      

         (   '
        
   5            
           !   "
%   -    ! .           (   '
%  

   " !                 
         '     .    
0  '   1 . !
/

  ' '  1 
2 '   "
 3 4 *

      

         (   '
5        
   5 5           
           !   "
%   -    ! .           (   '
%  

   " !                 
         '     .    
0  '   1 . !
/

  ' '  1 
2 '   "
 3 4 *

      

         (   '
        
   5            
           !   "
%   -    ! .           (   '
%  

   " !                 
         '     .    
0  '   1 . !
/

  ' '  1 
2 '   "
 3 4 *

      

         (   '
4         
               
           !   "
%   -    ! .           (   '
%  

   " !                 
         '     .    
0  '   1 . !
/

  ' '  1 
2 '   "
 3 4 *

      

         (   '
4        
              
           !   "
%   -    ! .           (   '
%  

   " !                 
         '     .    
0  '   1 . !
/

  ' '  1 
2 '   "
 3 4 *

      

         (   '
4 4        
    4           
           !   "
%   -    ! .           (   '
%  

   " !                 
         '     .    
0  '   1 . !
/

  ' '  1 
2 '   "
 3 4 *

      

         (   '
4         
               
           !   "
%   -    ! .           (   '
%  

   " !                 
         '     .    
0  '   1 . !
/

  ' '  1 
2 '   "
 3 4 *

      

         (   '
4        
              
           !   "
%   -    ! .           (   '
%  

   " !                 
         '     .    
0  '   1 . !
/

  ' '  1 
2 '   "
 3 4 *

      

         (   '
4 *        
    *           
           !   "
%   -    ! .           (   '
%  

   " !                 
         '     .    
0  '   1 . !
/

  ' '  1 
2 '   "
 3 4 *

      

         (   '
4        
              
           !   "
%   -    ! .           (   '
%  

   " !                 
         '     .    
0  '   1 . !
/

  ' '  1 
2 '   "
 3 4 *

      

         (   '
4         
               
           !   "
%   -    ! .           (   '
%  

   " !                 
         '     .    
0  '   1 . !
/

  ' '  1 
2 '   "
 3 4 *

      

         (   '
4 5        
    5           
           !   "
%   -    ! .           (   '
%  

   " !                 
         '     .    
0  '   1 . !
/

  ' '  1 
2 '   "
 3 4 *

      

         (   '
4         
               
           !   "
%   -    ! .           (   '
%  

   " !                 
         '     .    
0  '   1 . !
/

  ' '  1 
2 '   "
 3 4 *
      

         (   '
         
              
           !   "
%   -    ! .           (   '
%  

   " !                 
         '     .    
0  '   1 . !
/

  ' '  1 
2 '   "
 3 4 *

      

         (   '
        
             
           !   "
%   -    ! .           (   '
%  

   " !                 
         '     .    
0  '   1 . !
/

  ' '  1 
2 '   "
 3 4 *

      

         (   '
 4        
   4           
           !   "
%   -    ! .           (   '
%  

   " !                 
         '     .    
0  '   1 . !
/

  ' '  1 
2 '   "
 3 4 *

      

         (   '
         
              
           !   "
%   -    ! .           (   '
%  

   " !                 
         '     .    
0  '   1 . !
/

  ' '  1 
2 '   "
 3 4 *

      

         (   '
        
             
           !   "
%   -    ! .           (   '
%  

   " !                 
         '     .    
0  '   1 . !
/

  ' '  1 
2 '   "
 3 4 *

      

         (   '
 *        
   *           
           !   "
%   -    ! .           (   '
%  

   " !                 
         '     .    
0  '   1 . !
/

  ' '  1 
2 '   "
 3 4 *

      

         (   '
        
             
           !   "
%   -    ! .           (   '
%  

   " !                 
         '     .    
0  '   1 . !
/

  ' '  1 
2 '   "
 3 4 *

      

         (   '
         
              
           !   "
%   -    ! .           (   '
%  

   " !                 
         '     .    
0  '   1 . !
/

  ' '  1 
2 '   "
 3 4 *

      

         (   '
 5        
   5           
           !   "
%   -    ! .           (   '
%  

   " !                 
         '     .    
0  '   1 . !
/

  ' '  1 
2 '   "
 3 4 *

      

         (   '
         
              
           !   "
%   -    ! .           (   '
%  

   " !                 
         '     .    
0  '   1 . !
/

  ' '  1 
2 '   "
 3 4 *

      

         (   '
        
             
           !   "
%   -    ! .           (   '
%  

   " !                 
         '     .    
0  '   1 . !
/

  ' '  1 
2 '   "
 3 4 *

      

         (   '
       
            
           !   "
%   -    ! .           (   '
%  

   " !                 
         '     .    
0  '   1 . !
/

  ' '  1 
2 '   "
 3 4 *

      

         (   '
4        
  4           
           !   "
%   -    ! .           (   '
%  

   " !                 
         '     .    
0  '   1 . !
/

  ' '  1 
2 '   "
 3 4 *

      

         (   '
        
             
           !   "
%   -    ! .           (   '
%  

   " !                 
         '     .    
0  '   1 . !
/

  ' '  1 
2 '   "
 3 4 *

      

         (   '
       
            
           !   "
%   -    ! .           (   '
%  

   " !                 
         '     .    
0  '   1 . !
/

  ' '  1 
2 '   "
 3 4 *

      

         (   '
*        
  *           
           !   "
%   -    ! .           (   '
%  

   " !                 
         '     .    
0  '   1 . !
/

  ' '  1 
2 '   "
 3 4 *

      

         (   '
       
            
           !   "
%   -    ! .           (   '
%  

   " !                 
         '     .    
0  '   1 . !
/

  ' '  1 
2 '   "
 3 4 *

      

         (   '
        
             
           !   "
%   -    ! .           (   '
%  

   " !                 
         '     .    
0  '   1 . !
/

  ' '  1 
2 '   "
 3 4 *

      

         (   '
5        
  5           
           !   "
%   -    ! .           (   '
%  

   " !                 
         '     .    
0  '   1 . !
/

  ' '  1 
2 '   "
 3 4 *
      

         (   '
        
             
           !   "
%   -    ! .           (   '
%  

   " !                 
         '     .    
0  '   1 . !
/

  ' '  1 
2 '   "
 3 4 *

      

         (   '
*         
  4            
           !   "
%   -    ! .           (   '
%  

   " !                 
         '     .    
0  '   1 . !
/

  ' '  1 
2 '   "
 3 4 *

      

6 7 8 9 : ; < = > ? @ A B B C D B E

6 7 8 9 : ; < F G H I 7 J ? K

6 7 8 9 : ; < 9 L M 7 ? N < O : < 7

< > 9 ; J L ; J N M 9 : ; < O ? < > 9 ; J L


P Q Q Q P Q Q

F G H I 7 J ; R S G 7 T 9 : ; < T ? B

F G H I 7 J ; R S G 7 T 9 : ; < T 9 ; I 7 9 9 7 H M 9 7 > ? B
Q

6 7 8 9 : ; < J U T ? V E
P Q

: T M O L F G H I 7 J X < 7 O ? Y 7 T
W Q Q

Z J ; G M [ O O S G 7 T 9 : ; < T ? F ;

6 G I \ 6 7 8 9 : ; < F G H I 7 J ? B

6 G I \ 6 7 8 9 : ; < = > ? @ A B B C D K B

S G 7 T 9 : ; < 6 ] G R R O : < ^ [ O O ; _ 7 > ? Y 7 T

       
  4           
                          !   " # ! !  $ 
%   & '      (  '        

       
 

         (   ' 
) *     +
   ,        

         (   '
*        
  4 4           
           !   "
%   -    ! .           (   '
%  

   " !                 
         '     .    
0  '   1 . !
/

  ' '  1 
2 '   "
 3 4 *

      

         (   '
* 4        
  4            
           !   "
%   -    ! .           (   '
%  

   " !                 
         '     .    
0  '   1 . !
/

  ' '  1 
2 '   "
 3 4 *

      

         (   '
*         
  4           
           !   "
%   -    ! .           (   '
%  

   " !                 
         '     .    
0  '   1 . !
/

  ' '  1 
2 '   "
 3 4 *

      

         (   '
*        
  4 *           
           !   "
%   -    ! .           (   '
%  

   " !                 
         '     .    
0  '   1 . !
/

  ' '  1 
2 '   "
 3 4 *

      

         (   '
* *        
  4           
           !   "
%   -    ! .           (   '
%  

   " !                 
         '     .    
0  '   1 . !
/

  ' '  1 
2 '   "
 3 4 *

      

         (   '
*        
  4            
           !   "
%   -    ! .           (   '
%  

   " !                 
         '     .    
0  '   1 . !
/

  ' '  1 
2 '   "
 3 4 *

      

         (   '
*         
  4 5           
           !   "
%   -    ! .           (   '
%  

   " !                 
         '     .    
0  '   1 . !
/

  ' '  1 
2 '   "
 3 4 *

      

         (   '
* 5        
  4            
           !   "
%   -    ! .           (   '
%  

   " !                 
         '     .    
0  '   1 . !
/

  ' '  1 
2 '   "
 3 4 *

      

         (   '
*         
              
           !   "
%   -    ! .           (   '
%  

   " !                 
         '     .    
0  '   1 . !
/

  ' '  1 
2 '   "
 3 4 *

      

         (   '
        
             
           !   "
%   -    ! .           (   '
%  

   " !                 
         '     .    
0  '   1 . !
/

  ' '  1 
2 '   "
 3 4 *

      

         (   '
       
   4           
           !   "
%   -    ! .           (   '
%  

   " !                 
         '     .    
0  '   1 . !
/

  ' '  1 
2 '   "
 3 4 *

      

         (   '
4        
              
           !   "
%   -    ! .           (   '
%  

   " !                 
         '     .    
0  '   1 . !
/

  ' '  1 
2 '   "
 3 4 *

      

         (   '
        
             
           !   "
%   -    ! .           (   '
%  

   " !                 
         '     .    
0  '   1 . !
/

  ' '  1 
2 '   "
 3 4 *

      

         (   '
       
   *           
           !   "
%   -    ! .           (   '
%  

   " !                 
         '     .    
0  '   1 . !
/

  ' '  1 
2 '   "
 3 4 *

      

         (   '
*        
             
           !   "
%   -    ! .           (   '
%  

   " !                 
         '     .    
0  '   1 . !
/

  ' '  1 
2 '   "
 3 4 *

      

         (   '
       
              
           !   "
%   -    ! .           (   '
%  

   " !                 
         '     .    
0  '   1 . !
/

  ' '  1 
2 '   "
 3 4 *

      

         (   '
        
   5           
           !   "
%   -    ! .           (   '
%  

   " !                 
         '     .    
0  '   1 . !
/

  ' '  1 
2 '   "
 3 4 *

      

         (   '
5        
              
           !   "
%   -    ! .           (   '
%  

   " !                 
         '     .    
0  '   1 . !
/

  ' '  1 
2 '   "
 3 4 *

      

         (   '
        
             
           !   "
%   -    ! .           (   '
%  

   " !                 
         '     .    
0  '   1 . !
/

  ' '  1 
2 '   "
 3 4 *

      

         (   '
         
            
           !   "
%   -    ! .           (   '
%  

   " !                 
         '     .    
0  '   1 . !
/

  ' '  1 
2 '   "
 3 4 *
      

         (   '
        
  4           
           !   "
%   -    ! .           (   '
%  

   " !                 
         '     .    
0  '   1 . !
/

  ' '  1 
2 '   "
 3 4 *

      

         (   '
 4        
             
           !   "
%   -    ! .           (   '
%  

   " !                 
         '     .    
0  '   1 . !
/

  ' '  1 
2 '   "
 3 4 *

      

         (   '
         
            
           !   "
%   -    ! .           (   '
%  

   " !                 
         '     .    
0  '   1 . !
/

  ' '  1 
2 '   "
 3 4 *

      

         (   '
        
  *           
           !   "
%   -    ! .           (   '
%  

   " !                 
         '     .    
0  '   1 . !
/

  ' '  1 
2 '   "
 3 4 *

      

         (   '
 *        
            
           !   "
%   -    ! .           (   '
%  

   " !                 
         '     .    
0  '   1 . !
/

  ' '  1 
2 '   "
 3 4 *
      

         (   '
        
             
           !   "
%   -    ! .           (   '
%  

   " !                 
         '     .    
0  '   1 . !
/

  ' '  1 
2 '   "
 3 4 *

      

         (   '
         
  5           
           !   "
%   -    ! .           (   '
%  

   " !                 
         '     .    
0  '   1 . !
/

  ' '  1 
2 '   "
 3 4 *

      

         (   '
 5        
             
           !   "
%   -    ! .           (   '
%  

   " !                 
         '     .    
0  '   1 . !
/

  ' '  1 
2 '   "
 3 4 *

      

         (   '
         
  *            
           !   "
%   -    ! .           (   '
%  

   " !                 
         '     .    
0  '   1 . !
/

  ' '  1 
2 '   "
 3 4 *

      

         (   '
5         
  *           
           !   "
%   -    ! .           (   '
%  

   " !                 
         '     .    
0  '   1 . !
/

  ' '  1 
2 '   "
 3 4 *
      

         (   '
5        
  * 4           
           !   "
%   -    ! .           (   '
%  

   " !                 
         '     .    
0  '   1 . !
/

  ' '  1 
2 '   "
 3 4 *

      

         (   '
5 4        
  *            
           !   "
%   -    ! .           (   '
%  

   " !                 
         '     .    
0  '   1 . !
/

  ' '  1 
2 '   "
 3 4 *

      

         (   '
5         
  *           
           !   "
%   -    ! .           (   '
%  

   " !                 
         '     .    
0  '   1 . !
/

  ' '  1 
2 '   "
 3 4 *

      

         (   '
5        
  * *           
           !   "
%   -    ! .           (   '
%  

   " !                 
         '     .    
0  '   1 . !
/

  ' '  1 
2 '   "
 3 4 *

      

         (   '
5 *        
  *           
           !   "
%   -    ! .           (   '
%  

   " !                 
         '     .    
0  '   1 . !
/

  ' '  1 
2 '   "
 3 4 *
      

         (   '
5        
  *            
           !   "
%   -    ! .           (   '
%  

   " !                 
         '     .    
0  '   1 . !
/

  ' '  1 
2 '   "
 3 4 *

      

         (   '
5         
  * 5           
           !   "
%   -    ! .           (   '
%  

   " !                 
         '     .    
0  '   1 . !
/

  ' '  1 
2 '   "
 3 4 *

      

         (   '
5 5        
  *            
           !   "
%   -    ! .           (   '
%  

   " !                 
         '     .    
0  '   1 . !
/

  ' '  1 
2 '   "
 3 4 *

      

         (   '
5         
             
           !   "
%   -    ! .           (   '
%  

   " !                 
         '     .    
0  '   1 . !
/

  ' '  1 
2 '   "
 3 4 *

      

         (   '
         
            
           !   "
%   -    ! .           (   '
%  

   " !                 
         '     .    
0  '   1 . !
/

  ' '  1 
2 '   "
 3 4 *

      

         (   '
        
  4           
           !   "
%   -    ! .           (   '
%  

   " !                 
         '     .    
0  '   1 . !
/

  ' '  1 
2 '   "
 3 4 *

      

         (   '
 4        
             
           !   "
%   -    ! .           (   '
%  

   " !                 
         '     .    
0  '   1 . !
/

  ' '  1 
2 '   "
 3 4 *

      

         (   '
         
            
           !   "
%   -    ! .           (   '
%  

   " !                 
         '     .    
0  '   1 . !
/

  ' '  1 
2 '   "
 3 4 *

      

         (   '
        
  *           
           !   "
%   -    ! .           (   '
%  

   " !                 
         '     .    
0  '   1 . !
/

  ' '  1 
2 '   "
 3 4 *

      

         (   '
 *        
            
           !   "
%   -    ! .           (   '
%  

   " !                 
         '     .    
0  '   1 . !
/

  ' '  1 
2 '   "
 3 4 *

      

         (   '
        
             
           !   "
%   -    ! .           (   '
%  

   " !                 
         '     .    
0  '   1 . !
/

  ' '  1 
2 '   "
 3 4 *

      

         (   '
         
  5           
           !   "
%   -    ! .           (   '
%  

   " !                 
         '     .    
0  '   1 . !
/

  ' '  1 
2 '   "
 3 4 *

      

         (   '
 5        
             
           !   "
%   -    ! .           (   '
%  

   " !                 
         '     .    
0  '   1 . !
/

  ' '  1 
2 '   "
 3 4 *

      

         (   '
         
              
           !   "
%   -    ! .           (   '
%  

   " !                 
         '     .    
0  '   1 . !
/

  ' '  1 
2 '   "
 3 4 *

      

         (   '
         
             
           !   "
%   -    ! .           (   '
%  

   " !                 
         '     .    
0  '   1 . !
/

  ' '  1 
2 '   "
 3 4 *

      

6 7 8 9 : ; < = > ? @ A B B C D B B

6 7 8 9 : ; < F G H I 7 J ? C

6 7 8 9 : ; < 9 L M 7 ? N < O : < 7

< > 9 ; J L ; J N M 9 : ; < O ? < > 9 ; J L


P Q Q Q P Q Q

F G H I 7 J ; R S G 7 T 9 : ; < T ? B

F G H I 7 J ; R S G 7 T 9 : ; < T 9 ; I 7 9 9 7 H M 9 7 > ? B
Q

6 7 8 9 : ; < J U T ? V E
P Q

: T M O L F G H I 7 J X < 7 O ? Y 7 T
W Q Q

Z J ; G M [ O O S G 7 T 9 : ; < T ? F ;

6 G I \ 6 7 8 9 : ; < F G H I 7 J ? B

6 G I \ 6 7 8 9 : ; < = > ? @ A B B C D K K

S G 7 T 9 : ; < 6 ] G R R O : < ^ [ O O ; _ 7 > ? Y 7 T

       
   4           
                          !   " # ! !  $ 
%   & '      (  '        

       
 

         (   ' 
)    *  +

   ,        

         (   '
        
              
           !   "
%   -    ! .           (   '
%  

   " !                 
         '     .    
0  '   1 . !
/

  ' '  1 
2 '   "
 3 4 *

      

         (   '
 4        
             
           !   "
%   -    ! .           (   '
%  

   " !                 
         '     .    
0  '   1 . !
/

  ' '  1 
2 '   "
 3 4 *

      

         (   '
         
   *           
           !   "
%   -    ! .           (   '
%  

   " !                 
         '     .    
0  '   1 . !
/

  ' '  1 
2 '   "
 3 4 *

      

         (   '
        
             
           !   "
%   -    ! .           (   '
%  

   " !                 
         '     .    
0  '   1 . !
/

  ' '  1 
2 '   "
 3 4 *

      

         (   '
 *        
              
           !   "
%   -    ! .           (   '
%  

   " !                 
         '     .    
0  '   1 . !
/

  ' '  1 
2 '   "
 3 4 *

      

         (   '
        
   5           
           !   "
%   -    ! .           (   '
%  

   " !                 
         '     .    
0  '   1 . !
/

  ' '  1 
2 '   "
 3 4 *

      

         (   '
         
              
           !   "
%   -    ! .           (   '
%  

   " !                 
         '     .    
0  '   1 . !
/

  ' '  1 
2 '   "
 3 4 *
      

         (   '
 5        
  5            
           !   "
%   -    ! .           (   '
%  

   " !                 
         '     .    
0  '   1 . !
/

  ' '  1 
2 '   "
 3 4 *

      

         (   '
         
  5           
           !   "
%   -    ! .           (   '
%  

   " !                 
         '     .    
0  '   1 . !
/

  ' '  1 
2 '   "
 3 4 *

      

         (   '
        
  5 4           
           !   "
%   -    ! .           (   '
%  

   " !                 
         '     .    
0  '   1 . !
/

  ' '  1 
2 '   "
 3 4 *

      

         (   '
       
  5            
           !   "
%   -    ! .           (   '
%  

   " !                 
         '     .    
0  '   1 . !
/

  ' '  1 
2 '   "
 3 4 *

      

         (   '
4        
  5           
           !   "
%   -    ! .           (   '
%  

   " !                 
         '     .    
0  '   1 . !
/

  ' '  1 
2 '   "
 3 4 *
      

         (   '
        
  5 *           
           !   "
%   -    ! .           (   '
%  

   " !                 
         '     .    
0  '   1 . !
/

  ' '  1 
2 '   "
 3 4 *

      

         (   '
       
  5           
           !   "
%   -    ! .           (   '
%  

   " !                 
         '     .    
0  '   1 . !
/

  ' '  1 
2 '   "
 3 4 *

      

         (   '
*        
  5            
           !   "
%   -    ! .           (   '
%  

   " !                 
         '     .    
0  '   1 . !
/

  ' '  1 
2 '   "
 3 4 *

      

         (   '
       
  5 5           
           !   "
%   -    ! .           (   '
%  

   " !                 
         '     .    
0  '   1 . !
/

  ' '  1 
2 '   "
 3 4 *

      

         (   '
        
  5            
           !   "
%   -    ! .           (   '
%  

   " !                 
         '     .    
0  '   1 . !
/
  ' '  1 
2 '   "
 3 4 *

      

         (   '
5        
              
           !   "
%   -    ! .           (   '
%  

   " !                 
         '     .    
0  '   1 . !
/

  ' '  1 
2 '   "
 3 4 *

      

         (   '
        
             
           !   "
%   -    ! .           (   '
%  

   " !                 
         '     .    
0  '   1 . !
/

  ' '  1 
2 '   "
 3 4 *

      

         (   '
4         
   4           
           !   "
%   -    ! .           (   '
%  

   " !                 
         '     .    
0  '   1 . !
/

  ' '  1 
2 '   "
 3 4 *

      

         (   '
4        
              
           !   "
%   -    ! .           (   '
%  

   " !                 
         '     .    
0  '   1 . !
/

  ' '  1 
2 '   "
 3 4 *

      

         (   '
4 4        
             
           !   "
%   -    ! .           (   '
%  

   " !                 
         '     .    
0  '   1 . !
/

  ' '  1 
2 '   "
 3 4 *

      

         (   '
4         
   *           
           !   "
%   -    ! .           (   '
%  

   " !                 
         '     .    
0  '   1 . !
/

  ' '  1 
2 '   "
 3 4 *

      

         (   '
4        
             
           !   "
%   -    ! .           (   '
%  

   " !                 
         '     .    
0  '   1 . !
/

  ' '  1 
2 '   "
 3 4 *

      

         (   '
4 *        
              
           !   "
%   -    ! .           (   '
%  

   " !                 
         '     .    
0  '   1 . !
/

  ' '  1 
2 '   "
 3 4 *

      

         (   '
4        
   5           
           !   "
%   -    ! .           (   '
%  

   " !                 
         '     .    
0  '   1 . !
/

  ' '  1 
2 '   "
 3 4 *

      

         (   '
4         
              
           !   "
%   -    ! .           (   '
%  

   " !                 
         '     .    
0  '   1 . !
/

  ' '  1 
2 '   "
 3 4 *

      

         (   '
4 5        
  *             
           !   "
%   -    ! .           (   '
%  

   " !                 
         '     .    
0  '   1 . !
/

  ' '  1 
2 '   "
 3 4 *

      

         (   '
4         
  *            
           !   "
%   -    ! .           (   '
%  

   " !                 
         '     .    
0  '   1 . !
/

  ' '  1 
2 '   "
 3 4 *

      

         (   '
         
  *  4           
           !   "
%   -    ! .           (   '
%  

   " !                 
         '     .    
0  '   1 . !
/

  ' '  1 
2 '   "
 3 4 *

      

         (   '
        
  *             
           !   "
%   -    ! .           (   '
%  

   " !                 
         '     .    
0  '   1 . !
/

  ' '  1 
2 '   "
 3 4 *

      

         (   '
 4        
  *            
           !   "
%   -    ! .           (   '
%  

   " !                 
         '     .    
0  '   1 . !
/

  ' '  1 
2 '   "
 3 4 *

      

         (   '
         
  *  *           
           !   "
%   -    ! .           (   '
%  

   " !                 
         '     .    
0  '   1 . !
/

  ' '  1 
2 '   "
 3 4 *

      

         (   '
        
  *            
           !   "
%   -    ! .           (   '
%  

   " !                 
         '     .    
0  '   1 . !
/

  ' '  1 
2 '   "
 3 4 *

      

         (   '
 *        
  *             
           !   "
%   -    ! .           (   '
%  

   " !                 
         '     .    
0  '   1 . !
/

  ' '  1 
2 '   "
 3 4 *

      

         (   '
        
  *  5           
           !   "
%   -    ! .           (   '
%  

   " !                 
         '     .    
0  '   1 . !
/

  ' '  1 
2 '   "
 3 4 *
      

         (   '
         
  *             
           !   "
%   -    ! .           (   '
%  

   " !                 
         '     .    
0  '   1 . !
/

  ' '  1 
2 '   "
 3 4 *

      

         (   '
 5        
  *            
           !   "
%   -    ! .           (   '
%  

   " !                 
         '     .    
0  '   1 . !
/

  ' '  1 
2 '   "
 3 4 *

      

         (   '
         
  *           
           !   "
%   -    ! .           (   '
%  

   " !                 
         '     .    
0  '   1 . !
/

  ' '  1 
2 '   "
 3 4 *

      

         (   '
        
  * 4           
           !   "
%   -    ! .           (   '
%  

   " !                 
         '     .    
0  '   1 . !
/

  ' '  1 
2 '   "
 3 4 *

      

         (   '
       
  *            
           !   "
%   -    ! .           (   '
%  

   " !                 
         '     .    
0  '   1 . !
/

  ' '  1 
2 '   "
 3 4 *
      

         (   '
4        
  *           
           !   "
%   -    ! .           (   '
%  

   " !                 
         '     .    
0  '   1 . !
/

  ' '  1 
2 '   "
 3 4 *

      

         (   '
        
  * *           
           !   "
%   -    ! .           (   '
%  

   " !                 
         '     .    
0  '   1 . !
/

  ' '  1 
2 '   "
 3 4 *

      

         (   '
       
  *           
           !   "
%   -    ! .           (   '
%  

   " !                 
         '     .    
0  '   1 . !
/

  ' '  1 
2 '   "
 3 4 *

      

         (   '
*        
  *            
           !   "
%   -    ! .           (   '
%  

   " !                 
         '     .    
0  '   1 . !
/

  ' '  1 
2 '   "
 3 4 *

      

         (   '
       
  * 5           
           !   "
%   -    ! .           (   '
%  

   " !                 
         '     .    
0  '   1 . !
/

  ' '  1 
2 '   "
 3 4 *
      

         (   '
        
  *            
           !   "
%   -    ! .           (   '
%  

   " !                 
         '     .    
0  '   1 . !
/

  ' '  1 
2 '   "
 3 4 *

      

         (   '
5        
  * 4            
           !   "
%   -    ! .           (   '
%  

   " !                 
         '     .    
0  '   1 . !
/

  ' '  1 
2 '   "
 3 4 *

      

         (   '
        
  * 4           
           !   "
%   -    ! .           (   '
%  

   " !                 
         '     .    
0  '   1 . !
/

  ' '  1 
2 '   "
 3 4 *

      

         (   '
*         
  * 4 4           
           !   "
%   -    ! .           (   '
%  

   " !                 
         '     .    
0  '   1 . !
/

  ' '  1 
2 '   "
 3 4 *

      

CLAT 2018 - PG

1. Which of the following judgments was not over-ruled by the judgment in K. S.


Puttaswamy v. Union of India (2017) 10 SCC 1

A. M. P. Sharma v. Satish Chandra (1954) SCR 1077


B. Kharak Singh v. State of Uttar Pradesh (1964) 1 SCR 332
C. ADM Jabalpur v.ShivakantShukla (1976) 2 SCC 521
D. Govind v. State of M. P. (1975) 2 SCC 148

2. Rights over resources within the territorial waters, continental shelf and exclusive
economic zone, vest in the Union of India, under which provision of the Constitution of
India?

A. Article 297
B. Article 51
C. Article 253
D. Article 260

3. Which of the following is not a special procedure or feature in respect of a Money Bill?

A. It shall not be introduced in the Council of States.


B. The House of the People may not accept any recommendations of the Council
of States and shall return the Money Bill to the Council of States requesting
that the Council of States reconsider its recommendations.
C. A Bill only imposing pecuniary penalties shall not be deemed to be a Money Bill.
D. The House of the People may not accept any recommendations of the Council of
States and the Money Bill shall be deemed to have been passed by both Houses.

4. Which of the following shall not be eligible for further office under the Government of
India or Government of any State?

A. Speaker of the House of the People


B. Comptroller and Auditor-General of India
C. Attorney General for India
D. Member of a Public Service Commission

5. Part IX of the Constitution of India providing for constitution of Panchayats was


introduced to give effect to which provision of the Constitution?

A. Article 40
B. Article 21A

1
C. Article 23
D. Article 48

6. Which Constitution (Amendment) Act brought about modifications to Fundamental


Rights, Directive Principles of State Policy as well as Fundamental Duties?

A. The Constitution (Eightieth Amendment) Act, 1999


B. The Constitution (Eighty-sixth Amendment) Act, 2002
C. The Constitution (One Hundredth Amendment) Act, 2015
D. The Constitution (Ninety-third Amendment) Act, 2005

7. On which of the following matter in List II to Seventh Schedule of the Constitution of


India is the Legislative Assembly for the National Capital Territory of Delhi empowered
to make laws?

A. Banking
B. Police
C. Rights in land
D. Officers and servants of the High Court

8. In which of the following judgments was it held that the jurisdiction conferred upon the
High Courts under Article 226 and upon the Supreme Court under Article 32 of the
Constitution is part of the inviolable basic structure of the Constitution of India?

A. Common Cause v. Union of India (1999) 6 SCC 667


B. L. Chandra Kumar v. Union of India (1997) 3 SCC 261
C. In re Special Courts Bill (1979) 1 SCC 380
D. S. R. Bommai v. Union of India (1994) 3 SCC 1

9. The National Judicial Appointments Commission Act, 2014 was declared as


unconstitutional and void in:

A. Youth Bar Association of India v. Union of India (2016) 9 SCC 473


B. Common Cause v. Union of India (2015) 6 SCC 332
C. Madras Bar Association v. Union of India, AIR 2015 SC 1571
D. Supreme Court Advocate-on-Record Association v. Union of India (2016) 5 SCC 1

10. Which of the following is the correct position in law regarding administration of narco-
analysis techniques, in the light of the judgment in Selvi v. State of Karnataka (2010) 7
SCC 263?
(i) Compulsory administration constitutes cruel, inhuman or degrading treatment in
the context of Article 21.
(ii) Compulsory administration violates the right against self-incrimination under
Article 20(3).

2
(iii) Compulsory administration may, however, be justifiable in compelling public
interest in combatting terrorist activities.
(iv) Information subsequently discovered with the help of voluntarily administered
narco-analysis results may be admitted in evidence.

Select the correct answer from the following:

A. (ii) and (iv) only


B. (i) and (ii) only
C. (i), (ii) and (iv) only
D. (iii) and (iv) only

11. The Constitution (One Hundred and First) Amendment Act, 2016 was required to be
ratified by the Legislature of not less than one half of the State Legislatures because it
sought to make changes in:
(i) Chapter I of Part XI of the Constitution dealing with legislative relations
between the Union and the States.
(ii) Lists I and II of the Seventh Schedule to the Constitution.
(iii) the Sixth Schedule to the Constitution.
(iv) Part XIII of the Constitution dealing with trade, commerce and intercourse
within India.

Select the correct answer from the following:

A. All the four reasons


B. It was not required to be ratified by the State Legislatures before being
presented to the President for assent.
C. (ii) only
D. (i) and (ii) only

12. Omitted for Valuation

13. A majority of not less than two-third of the membership of the House concerned is not
necessary for removal of which of the following authority?

A. A Judge of the High Court


B. The President of India
C. The Chief Election Commissioner
D. The Speaker of the House of the People

14. Which of the following Commission(s) has/have a constitutional status, as of April,


2017?
(i) National Commission for Scheduled Castes
(ii) National Commission for Backward Classes
(iii) National Commission for Scheduled Tribes

3
(iv) National Human Rights Commission

A. (i), (ii) and (iii) only


B. (i) and (iii) only
C. (i), (ii), (iii) and (iv)
D. (i) only

15. Which of the following reason(s) was/were considered necessary for the Constitution
(Seventy-Seventh) Amendment Act, 1995 inserting Article 16(4A)?
(i) Inadequacy of representation of the Scheduled Castes and the Scheduled Tribes
in services in the States.
(ii) Reservation in promotion was held constitutionally impermissible in Indra
Sawhney v. Union of India, AIR 1993 SC 477.
(iii) To prevent qualitative exclusion of the Scheduled Castes and the Scheduled
Tribes in services in the States.
(iv) To achieve inter-locking in horizontal and vertical reservation among the
Scheduled Castes and the Scheduled Tribes in services in the States.

Select the correct answer from the following:

A. (i) only
B. (i) and (ii) only
C. (i), (ii) and (iv) only
D. (i), (ii), (iii) and (iv)

16. Which of the following provision(s) applicable to the Supreme Court is/are also
applicable to the High Courts?
(i) Article 124(4).
(ii) Article 124(5).
(iii) Article 129
(iv) Article 144

Select the correct answer from the following:

A. (i) and (ii) only


B. (i), (ii) and (iv) only
C. (iii) and (iv) only
D. (iii) only

17. Which of the following statement(s) is/are correct, regarding the duties and rights of the
Attorney-General for India in performance of his duties?
(i) The Attorney-General for India shall give advice to the Government of
India upon legal matters.
(ii) The Attorney-General for India shall have a right to audience in all Courts
in India.

4
(iii) The Attorney-General for India shall have the right to speak in the
proceedings of either House.
(iv) The Attorney-General for India shall not be liable to any proceedings in
any court in respect of anything said by him in Parliament or any
committee thereof.

A. (i) and (ii) only


B. (i), (ii) and (iii) only
C. All of these statements are correct
D. (i) only

18. Which of the following Fundamental Rights is enforceable against private individuals?

(i) Article 17
(ii) Article 23
(iii) Article 24
(iv) Article 21

Choose the correct answer from the following:

A. Fundamental Rights are not enforceable against private individuals


B. All are enforceable against private individuals
C. (i), (ii) and (iii) only
D. (ii) and (iii) only

19. Which of the following procedure was adopted to give effect to the land boundary
agreement between India and Bangladesh in 2015?
A. A law made by Parliament under Article 3 of the Constitution of India.
B. The Constitution (100th Amendment) Act.
C. The Constitution (98th Amendment) Act.
D. By an executive action.

20. Which of the following statements is incorrect in view of the judgment in Pramati
Educational and Cultural Trust v. Union of India (2014) 8 SCC 1?

A. Article 15(5) does not violate the freedom of private unaided educational
institutions under Article 19(1)(g) of the Constitution.
B. Articles 15(5) and 21A do not alter the basic structure of the Constitution and are
constitutionally valid.
C. A law made under Article 15(5) of the Constitution shall not be open to
challenge under Article 14, if it treats private aided and unaided educational
institutions alike.
D. Article 15(5), insofar as it is applicable to minority educational institutions
referred to in Article 30(1) of the Constitution, is ultra vires the Constitution.

5
21. From where did the Drafting Committee borrow the expression “except according to
procedure established by law” for Article 21 of the Indian Constitution?

A. Irish Constitution
B. Swedish Constitution
C. Australian Constitution
D. Japanese Constitution

22. In which of the following decisions the Supreme Court framed guidelines for
safeguarding the interest of the death row convicts?

A. Shatrughan Chauhan v. Union of India (2014) 3 SCC 1


B. Epuru Sudhakar v. Govt. of A.P. (2006) 8 SCC 161
C. People’s Union for Civil Liberties v. State of Maharashtra (2014) 10 SCC 635
D. Sunil Batra v. Delhi Administration (1978) 4 SCC 494

23. Which of the following does not follow from the law declared in the National Legal
Services Authority v. Union of India (2014) 5 SCC 438, in respect of rights of the
members of the transgender community?

A. Recognition to gender identity as male, female or as third gender.


B. Steps to treat the third gender as a socially and economically backward class.
C. Extending special provisions relating to certain classes as contained in Article
330 of the Constitution.
D. Extending reservation in cases of admission in educational institutions and public
appointments.

24. Which of the following were relevant considerations for prohibition on sale of liquor
along national and state highways in State of Tamil Nadu v. K. Balu (2017) 2 SCC 281?
(i) There is no fundamental right under Article 19(1)(g) to trade in liquor.
(ii) Under Article 47, the State is under a duty to raise the level of nutrition,
standard of living and improve public health.
(iii) An exception to permit the sale of liquor along a stretch of the highway which
passes through the limits of city is arbitrary and violative of Article 14.
(iv) Right to life under Article 21 signifies a right to live with human dignity, free of
noxious substances and intoxicants.

Choose the correct answer from the following:

A. All the four statements are correct


B. (i) and (iii) only
C. (i), (ii) and (iii) only
D. (i), (ii) and (iv) only

6
25. Which of the following reasons led to the setting aside of the practice of ‘talaq-e-
biddat’, triple talaq in Shayara Bano v. Union of India (2017) 9 SCC 1?
(i) Section 2 of the Muslim Personal Law (Shariat) Application Act, 1937 was a pre-
constitutional legislation within the meaning of Article 13(1) and violative of
Article 14 of the Constitution of India on the ground of arbitrariness.
(ii) Section 2 of the Muslim Personal Law (Shariat) Application Act, 1937 was a pre-
constitutional legislation within the meaning of Article 13(1) and violative of
Article 15 of the Constitution of India, being discriminatory.
(iii) Muslim Personal Law (Shariat) Application Act, 1937 is not a law regulating
triple talaq but the practice of triple talaq was against the basic tenets of the
Holy Quran and no practice against the tenets of Quran was permissible.
(iv) ‘Talaq-e-biddat’ is integral to the religious denomination but the Supreme Court
may injunct this practice as a means for severing matrimonial relationship.

A. (i) and (ii) only


B. (i), (ii) and (iii) only
C. (i) and (iv) only
D. (i) and (iii) only

26. Which is the correct position in law on an Ordinance promulgated under Article 123 of
the Constitution of India, following the judgment in Krishna Kumar v. State of Bihar
(2017) 3 SCC 1?
(i) Power conferred upon the President under Article 123 is legislative in character
and the satisfaction of the President is immune from judicial review.
(ii) An Ordinance promulgated under Article 123 must be laid before the
Parliament, which is a mandatory constitutional obligation cast upon the
government.
(iii) An Ordinance promulgated in exercise of powers under Article 213 cannot
create enduring rights in favour of individuals
(iv) Re-promulgation of an Ordinance is a fraud on the Constitution.

Choose the correct answer from the following:

A. All the four statements are correct


B. (ii), (iii) and (iv) only
C. (ii) and (iv) only
D. (i) only

27. Article 29 of the Constitution protects the rights of:

A. Minorities based on religion or language


B. Citizens with distinct script or culture
C. Religious denominations

7
D. Socially and educationally backward class of citizens

28. In which case, the Supreme Court struck down that part of the Constitution (Forty-second
Amendment) Act, 1976 by which Article 31C was amended?

A. Kesavananda Bharti v. State of Kerala, (1973) 4 SCC 225.


B. L.C. Golak Nath v. State of Punjab, AIR 1967 SC 1643.
C. Minerva Mills Ltd. v. Union of India, AIR 1980 SC 1789.
D. S.P. Sampath Kumar v. Union of India, 1987 SCR (3) 233.

29. In which among the following cases did the Supreme Court hold that the petitioners have
no fundamental right within the meaning of Art 25 or 26 to perform ‘Tandava dance’ in
public streets and public places.
A. Commr. HRE v. Sri Lakshmindra Thirtha Swamiar of Sri Shirur Mutt, AIR 1954 SC 282
B. Tilkayat Shri Govindlalji Maharaj v. State of Rajasthan, AIR 1963 SC 1638
C. N. Adithayan v. Travancore Devasom Board, (2002) 8 SCC 106
D. Commr. of Police v. Acharya Jagadishwarananda Avadhuta, (2004) 12 SCC 770

30. Omitted for Valuation

31. The idea of single citizenship in India was taken from the

A. British Constitution
B. US Constitution
C. Australian Constitution
D. Weimar Constitution

32. With which amendment was clause 4 to Article 13, Constitution of India, inserted?

A. The Constitution (Twenty-second Amendment) Act, 1969.


B. The Constitution (Twenty-third Amendment) Act, 1969.
C. The Constitution (Twenty-fourth Amendment) Act, 1971.
D. The Constitution (Twenty-fifth Amendment) Act, 1971.

33. Which Constitution Amendment Act brought into force the goods and services tax?
A. 98th
B. 99th
C. 100th
D. 101st

8
34. Which one of the following statements is not correct?

A. The Jammu and Kashmir Code of Criminal Procedure has no provision for
transfer of a case from a Court in Jammu and Kashmir to any Court outside that
State and, therefore, no criminal case can be transferred from the State of
Jammu and Kashmir to any other Court outside that State.
B. Article 370 of the Constitution of India confers merely a “Temporary” status on
the State of Jammu and Kashmir.
C. Article 371-D and Presidential Orders issued thereunder override any other
provision of the Constitution of India.
D. The citizens of India have a fundamental right to reside and settle in any part of
India except the State of Jammu and Kashmir.

35. Which of the following questions is not relevant in determining the existence of a
constitutional convention?

A. What are precedents?


B. Did the actors in the precedent believe that they were bound by a rule?
C. Are they justified in terms of morality?
D. Is there any reason for the rule?

36. In which of the following cases, the Supreme Court issued detailed directions for the
protection of Good Samaritans?

A. Savelife Foundation v. Union of India, AIR 2016 SC 1617.


B. Sabu Mathew George v. Union of India (2017) 2 SCC 514.
C. Voluntary Health Association of Punjab v. Union of India, AIR 2016 SC 5122.
D. Devika Biswas v. Union of India, AIR 2016 SC 4405.

37. In which of the following cases, the Supreme Court refused to give any relief to the
petitioner(s) under Article 32?

A. Swaraj Abhiyan v. Union of India, AIR 2016 SC 2929.


B. Bachpan Bachao Andolan v. Union of India, 2016 (12) SCALE 751.
C. Supreme Court Women Lawyers Association v. Union of India, AIR 2016 SC 358.
D. Suresh Chand Gautam v. State of Uttar Pradesh, AIR 2016 SC 1321.

38. Right to life includes right to reputation was recognised in:

A. Archbishop Raphael Cheenath S.V.D. v. State of Orissa, AIR 2016 SC 3639


B. Shyam Narayan Chouksey v. Union of India (2017) 1 SCC 421
C. Dr. Subramanian Swamy v. Union of India, AIR 2016 SC 2728
D. Karma Dorji v. Union of India, AIR 2017 SC 113

9
39. Preventive Detention under article 22 of the Constitution can be enforced:
A. Only during the time of communal violence
B. Only when a proclamation of national emergency is made
C. Only when a proclamation of internal emergency is made
D. At any time during peace or war

40. Article 233A was introduced into the Constitution of India to validate the appointments of,
and judgments, etc. delivered by certain District Judges. In which among the following
cases were those appointments invalidated in a writ of Quo-warranto?

A. Chandra Mohan v. State of U.P, AIR 1966 SC 1987


B. Chandramouleshwar Prasad v. Patna High Court, AIR 1970 SC370
C. G.D Karkare v. T.L Shevde, AIR 1952 Nag 330
D. Anand Bihari Mishra v. Ram Sahay, AIR 1952 MB 31

41. Which one of the following statements is correct?


A. The Court has no jurisdiction to see whether the procedure established by law in
Parliament is reasonable or not.
B. The procedure which is arbitrary or oppressive is not a valid procedure.
C. The requirement of compliance with the principles of natural justice is not implicit
in Art. 21 of the Constitution.
D. The Doctrine of 'Due Process' is not implied in Art. 21.

42. In which case was it held that the authority which “embarks upon a post-decisional hearing
will naturally proceed with a closed mind and there is hardly any chance of getting a proper
consideration of the representation at such a post-decisional opportunity”?

A. Maneka Gandhi v. Union of India (1978) 1 SCC 248


B. Liberty Oil Mills v. Union of India, AIR 1984 SC 1271
C. Charan Lal Sahu v. Union of India, AIR 1990 SC 1480
D. H.L. Trehan v. Union of India, AIR 1989 SC 568

43. Which one of the following statements is incorrect?

A. Parliament has exclusive power to make laws with respect to goods and services tax
where the supply of goods, or of services, or both takes place in the course of inter-
State trade or commerce.
B. The tax collected by the Union under Clause (1) of Article 246A shall not be
distributed between the Union and the States.
C. The Goods and Services Tax Council has power to establish a mechanism to
adjudicate any dispute between the Government of India and one or more States.

10
D. Article 279-A is an entrenched provision under Article 368 of the Constitution of
India.

44. In which of the following cases the reservation for Jats was struck down?

A. Ram Singh v. Union of India, 2015 (3) SCALE 570


B. U.P. Power Corpn. Ltd. v. Rajesh Kumar, AIR 2012 SC 2728
C. Prem Kumar Singh v. State of U.P., 2011 (3) ALJ 343
D. Dr. Gulshan Prakash v. State of Haryana (2009) 14 SCALE 290

45. Is a writ petition for mandamus maintainable in any of the following cases?

A. Against a citizen for performance of his fundamental duties.


B. Against a private University established by an Act of State Legislature.
C. Against a private trust running coaching classes.
D. Against a lawyer restraining him from filing public interest litigations.

46. For prevention of corruption, provisions have been made under which of the legislations?
(i) The Prevention of Corruption Act, 1988
(ii) The Lokpal and Lokayuktas Act, 2013
(iii) The Central Vigilance Commission Act, 2003

Choose the correct answer from the following:

A. (i) only.
B. (i) and (iii) only.
C. (iii) only.
D. (i), (ii) and (iii).

47. The Supreme Court may review the award given by the Inter-State Water Dispute Tribunal
by invoking
A. Article 142
B. Article 131
C. Article 262
D. Article 136

48. For approval or extension of emergency under Article 352 it is required that there should be

11
A. Resolution by both Houses of Parliament by a majority of total membership of that
House and by a majority of not less than two-thirds of members present and
voting.
B. Resolution by both Houses of Parliament by a simple majority of members present
and voting.
C. Resolution by both Houses of Parliament by a majority of total membership of that
House and by a majority of not less than two-thirds of members present and voting
and ratification by not less than one-half of the States.
D. Resolution by both Houses of Parliament by a majority of total membership of that
House and by a majority of not less than three-fourth of members present and
voting.

49. Double Jeopardy means:

A. No person shall be prosecuted and compelled to be a witness in the same case


simultaneously.
B. No person shall be sued for a civil wrong and prosecuted for an offence simultaneously.
C. No person shall be prosecuted for two offences simultaneously
D. No person shall be prosecuted and punished for the same offence more than once.

50. Omitted for Valuation

12
Juris Set 1 (Qs 51 – 100)

1. Eight Principles of Internal Morality of Law were propounded by

A. Hart
B. Hobbes
C. Kelsen
D. Lon L Fuller

2. Who stated: “The life of law has not been logic; it has been experience”

A. Oliver Holmes
B. Dworkin
C. Hart
D. Roscoe Pound

3. Roberto Mangabeira Unger was the main propounder of


A. Feminist Theory
B. Legal Textualism
C. Legal Realism
D. Critical Legal Studies

4. The Preamble to the UN Charter uses the expression

A. We the People
B. We the Nations
C. We the Peoples
D. We the Founding Nations

5. Jus Cogens means:

A. Peremptory norm of International Law


B. Judgments of the ICJ
C. Just Resolution of Disputes
D. Justice for all

6. The first Jurist to use the word Sociology was

A. Austin
B. Roscoe Pound
C. Duguit
D. Auguste Comte

13
7. The concept of Dialectical Materialism is a concept commonly associated with which of the
following Schools of Thought?

A. Marxism
B. Liberal Theory
C. Post Modernism
D. Utilitarianism

8. Who authored, “The Indian School of Synthetic Jurisprudence: An Idea Relating to Its
Purposes”?

A. Kautilya
B. Manu
C. M. J. Sethna
D. UpendraBaxi

9. Bridges v. Hawkesworth is a popular case on

A. Ownership
B. Possession
C. Transfer of ownership
D. Atonement

10. Hedonistic calculus as a measure of pain and pleasure was hypothesized by


A. Austin
B. Bentham
C. Salmond
D. Holland

11. Jurisprudence should be ideally divided into censorial and expository jurisprudence. This
view was given by
A. Auguste Comte
B. Bentham
C. Emile Durkheim
D. Holland

12. Law is a combination of Primary and Secondary rules. This principle was propounded by
A. Kelsen
B. Ronald Dworkin
C. Hart
D. Fuller

14
13. As per the Anthropological approach towards the study of Jurisprudence, the Twelve Tables
of Rome may be considered an example of
A. Religious law
B. Divine law
C. Published law
D. Personal command

14. Who made the statement, "Law is uncertain and certainty of law is a legal myth"?

A. Fuller

B. Jerome Frank

C. Llewellyn

D. Holmes

15. As per Kelsen, if a norm remains permanently inefficacious, then it is deprived of its validity.
This is known as
A. Nomodynamics
B. Condition per quam
C. Desuetudo
D. Effective Repeal

16. The theory “justice as fairness” was propounded by


A. Robert Nozick
B. Ronald Dworkin
C. F. Engels
D. John Rawls

17. The term ‘transcendental idealism’ is applied to the epistemology of


A. Hegel
B. Stammler
C. Immanuel Kant
D. Karl Marx

18. The famous essay Vom Bneruf was written by Savigny as a reaction to
A. Thibaut’s proposal for a Code on the lines of the Napoleonic Code
B. The King’s proposal for introducing British common law in Germany
C. The Parliament’s proposal to introduce Roman law in Germany

15
D. The revival of Justinian’s Code
19. Who among the following is called as “Darwinian before Darwin”, because of his
contribution to apply the evolutionary principle to the development of the legal system?
A. Gluckman
B. Malinowski
C. Savigny
D. Puchta

20. In his endeavour to discover a pattern of legal development, Maine was led to distinguish
between societies as
A. Static and progressive societies
B. Dependent and independent societies
C. Subjective and objective societies
D. Customary and statutory societies.

21. Pound’s theory of Social Engineering mainly talks about


A. General consciousness being the primary source of law
B. Legislative power
C. Balancing of competing interests in society
D. Difference between law creation and law application

22. Which theory of punishment implies “An eye for an eye, a tooth for a tooth” approach?
A. Expiatory
B. Preventive
C. Deterrent
D. Retributive

23. As per the rule of English law, a custom to be recognized as law, must be proved to be in
existence since times immemorial. However, for customs that are old and whose time of
origin cannot be ascertained, which year has been fixed as the date before which it must
have started?
A. 1169 AD
B. 1189 AD
C. 1369 AD
D. 1389 AD
24. Which of the following is an accessory right?
A. Easementary rights
B. Ownership rights
C. Right to reputation

16
D. Right to personal security
25. Ownership of goodwill of a business is
A. Corporeal ownership
B. Incorporeal ownership
C. Business ownership
D. Beneficial ownership

26. The law applying to controversies involving non-citizens of Rome was known as
A. Jus civilie
B. Jus gentium
C. Nicomachean ethics
D. Corpus civilie

27. Which one of the following philosophers conceived justice as ‘some sort of equality’ and
divided the concept of justice as ‘corrective justice’ and ‘distributive justice’?
A. Aristotle
B. Rawls
C. Robert Nozick
D. Ronald Dworkin

28. “Sociology of law must begin with the ascertainment of the living law, the concrete which
can be observed and not the abstract”. Who said so?
A. Duguit
B. Ihering
C. Ehrlich
D. Durkheim

29. Who said that, “The law of nature or natural law is the general will of the people who come
together to form an association called State”?
A. Hobbes
B. Locke
C. Rousseau
D. Kant
30. Which one of the following conceived ‘integrative jurisprudence’ as combined jurisprudence
of positivist, naturalist and sociological descriptions and an understanding of the value
components of legal ordering?
A. Jerome Hall
B. John Finnis
C. Clarence Morris

17
D. Leon Duguit
31. De Jure Belli ac Pacis is the celebrated work of-
A. Hugo Grotius
B. Niccolo Machiavelli
C. Samuel Pufendorf
D. Jean Jacques Burlamaqui

32. Which one of the following drafted the hypothetical Case of the Speluncean Explorers?
A. Lon L Fuller
B. Rudolph Stammler
C. John Rawls
D. Clarence Morris

33. Which of the following is not absolute duty according to John Austin?
A. Duties towards lower animals
B. Duties towards community
C. self-regarding duties
D. Duties towards sovereign

34. Who said, "A precedent, therefore, is a judicial decision which contains in itself a principle.
The underlying principle which thus forms its authoritative element is often termed the
ratio decidendi.”?
A. Salmond
B. Keeton
C. Rupert Cross
D. Osborn

35. Which of the following statements does not suit the Hohfeldian analysis of right?
A. Power in ‘X’ means presence of liability upon ‘Y’
B. Immunity in ‘X’ means absence of liability in ‘X’
C. Right in ‘X’ means presence of ‘No-right’ in ‘Y’
D. Liberty in ‘X’ means absence of duty in ‘X’

36. Which of the following is not a circumstance of strengthening the authority of a precedent?
A. Approval of the decision in later cases and by the profession at large
B. Closely divided opinion of the Court
C. Evidence in the report that the issue was fully argued by counsel and the Court took
time to deliberate
D. The decision has stood the test of time

18
37. A rule which says, “the literal sense of words should be adhered to, unless this would lead
to absurdity, in which case the literal meaning may be modified”, can be called
A. Mischief rule
B. Literal rule
C. Golden rule
D. Compromise approach

38. Who defined ownership as ‘a right – indefinite in point of user – unrestricted in point of
disposition – and unlimited in point of duration’?
A. Salmond
B. Austin
C. Marx
D. Glaniville Williams

39. Which of the following is not correct in the context of valid usage?
A. Usage must be so well established as notorious.
B. In case of conflict between usage and law, law prevails.
C. In case of conflict between express provision of a contract and usage, contract
prevails.
D. Usage shall be a reasonable one.

40. “A dictate of right reason, which points out that an act, according as it is or is not in
conformity with rational nature, has in it a quality of moral baseness or moral necessity.”
Who made this observation?
A. Cicero
B. Aristotle
C. Plato
D. Grotius

41. “Private property is robbery, and a state based on private property is a state of robbers who
are fighting for a share of the spoils.” Who made this observation?
A. Lenin
B. Marx
C. Renner
D. Pashukanis

42. In response to a petition, the Supreme Court transferred a case on constitutionality of a


Central Act to the High Court of Bombay. The Bombay High Court upheld it. In a

19
constitutional challenge of the same Act, pending before the High Court of Madras what the
latter High Court cannot do with regard to Bombay High Court judgment is

A. Abstaining from applying it


B. Applying it
C. Overruling it
D. Distinguishing it

43. Which of the following characteristics of trust is considered as a problematic factor in a legal
system?
A. Group property can be applied for the desired purpose.
B. Pooling up of economic power to neutralise competition.
C. Enables endowments and gifts by vesting rights in trustees.
D. Helps in settling family property in favour of the vulnerable.

44. Which of the following is a right in re aliena?


A. Literary copyright
B. Moveable property
C. Servitude
D. Land

45. Which of the following is not correct about the will theory of contract?
A. It believes in the dictum of status to contract.
B. It puts emphasis on expectations reasonably aroused by conduct rather than state
of mind.
C. It relies on person’s mind which can be proved only from his own evidence or by
inferences from his conduct.
D. It leads to such a subjective view of mistake that the security of transactions is
thereby imperilled.

46. “The suppression of vice is as much the law’s business as the suppression of subversive
activities.” Who made this observation?
A. H L A Hart
B. J S Mill
C. Lord Devlin
D. James Stephens
47. Which of the following is not correct about the Critical Legal School?
A. It strongly believes in the inescapable power of the intellectual climate’s search for
value neutrality.

20
B. It shows the way in which the legal system works at many different levels to shape
popular consciousness towards accepting the legitimacy of the status quo.
C. It rejects formalism.
D. Its central thrust of attack is on legal liberalism.

48. Which of the following propositions do not suit the approaches of American Realism?
A. It is a combination of the analytical positivist and sociological approaches.
B. It lays emphasis on empirical study of the behaviour of judges.
C. It distrusts the sufficiency of legal rules and concepts as descriptive of what Courts
do.
D. It combines the ‘is’ and ‘ought’ aspects in the study of law and how it works.

49. Who stated that, “Law is the sum of the conditions of social life in the widest sense of the
term, as secured by the power of the State through the means of external compulsion.”
A. Roscoe Pound
B. Jeremy Bentham
C. Ehrlich
D. Ihering

50. Which of the following is not emphasised by Katherine Bartlette as a part of feminist legal
research?
A. Asking the woman question
B. Consciousness –raising
C. Difference theory
D. Positionality

21
LLM General Set 1 Qs 101 – 150)

1. In which case was it observed that “The dividing line between an administrative power and
a quasi-judicial power is quite thin and is being gradually obliterated”?

A. Maneka Gandhi v. Union of India (1978) 1 SCC 248.


B. State of Orissa v. Dr. (Miss) Binapani Dei, AIR 1967 SC 1269.
C. A.K. Kraipak v. Union of India, AIR 1970 SC 170.
D. Swadeshi Cotton Mills v. Union of India, AIR 1981 SC 818.

2. In which of the following cases did the Supreme Court hold that the Wednesbury rule was
applicable in administrative actions?

A. Indian Railway Construction Co. Ltd. v. Ajay Kumar, (2003) 4 SCC 579
B. Shrilekha Vidyarthi v. State of U.P., AIR 1991 SC 537
C. State of Punjab v. V.K. Khanna, AIR 2001 SC 343
D. A.N. Parasuraman v. State of Tamil Nadu, AIR 1990 SC 40

3. Which one of the following statements with regard to the Lokpal and Lokayuktas Act, 2013
is not correct?

A. It makes certain amendments to the Code of Criminal Procedure, 1973.


B. It contains detailed provisions for the appointment of Lokayuktas in States.
C. It confers supervisory jurisdiction on Delhi Special Police Establishment.
D. It extends to the whole of India.

4. Which one of the following has powers similar to those of a Court having power to issue a
writ of certiorari under the express statutory provisions?

A. Administrative tribunals established under the Administrative Tribunals Act, 1985.


B. Industrial Tribunals established under the Industrial Disputes Act, 1947.
C. Income Tax Appellate Tribunal established under the Income Tax Act, 1961.
D. State Consumer Disputes Redressal Commission established under the Consumer
Protection Act, 1986.

5. Which one of the following statements with regard to a Commission of Inquiry is incorrect?

A. A Commission of Inquiry can be headed only by a sitting or retired judge.


B. A Commission of Inquiry cannot compel a person to disclose any secret process
of manufacture of any goods unless the subject matter of inquiry relates to that
process.
C. The report of the Commission of Inquiry is not binding on the Government.
D. The Central Government is not always restrained from appointing a Commission
of Inquiry when on the same subject a Commission appointed by a State
Government is already working.

22
6. Which one of the following statements with regard to the Right to Information Act, 2005 is
not correct?

A. No reasons are required to be given for seeking information.


B. Private Universities are bound to give information.
C. No fee is to be paid if information is supplied after five weeks from the date of
application.
D. Intelligence Bureau is not bound to give any information.

7. Which among the following was the first case before the Supreme Court of India where
the constitutionality of death penalty was challenged?
A. Rajendra Prasad v. State of U. P.
B. Jagmohan Singh v. State of U. P.
C. Bachan Singh v. State of Punjab
D. Machhi Singh and Others v. State of Punjab

8. Which among the following is/are not true with respect to the decision of the Supreme
Court of India in Vellore Citizens Welfare Forum v. Union of India (AIR 1996 SC 2715)?
i. This was a PIL against the pollution caused by discharge of untreated effluent from
tanneries and other industries.
ii. The Court refused to recognise that a balance must be struck between the economy
and the environment.
iii. The Court accepted the traditional concept that development and ecology are
opposed to each other.
iv. The Court reviewed the development of the concept of sustainable development in
the international sphere.
v. The Supreme Court held that the precautionary principle and the polluter pays
principle are part of International environmental law and not of municipal law.
A. Only i, iii and v
B. Only ii, iii and v
C. Only iii, iv and v
D. Only iv and v

9. Identify the correct statement with respect to the Law of Partnership?

A. Though registration of the Partnership is not mandatory, the registration of


the Partnership deed is essential.
B. The Limited Liability Partnership is part of the Indian Partnership Act, 1932.
C. The existence of a partnership can only be determined by the partnership
deed.

23
D. Unless there is a contract to the contrary, death of a partner will
automatically lead to the dissolution of the partnership.

10. In which among the following cases did the Supreme Court observe:
“We cannot allow our judicial thinking to be constricted by reference to the law as it
prevails in England or for that matter, in any other foreign country. We no longer need
the crutches of a foreign legal order.”

A. Sri Ram Food Fertilizers Case (AIR 1987 SC 965)


B. M.C Mehta v. Union of India (AIR 1987 SC 1086)
C. Union Carbide Corporation v. Union of India(AIR 1990 SC 273)
D. M.P. Electricity Board v. Shail Kumar (AIR 2002 SC 551)

11. Which among the following authorities is vested with the power to investigate on matters of
professional misconduct committed by a Chartered Accountant or a Chartered Accountant’s Firm
under Section 132 of the Companies Act, 2013?

A. National Advisory Committee on Accounting Standards


B. Institute of Chartered Accountants of India
C. The Serious Fraud Investigation Office
D. National Financial Reporting Authority

12. Which among the following is not true with respect to negotiable instruments?

A. There must be an unconditional order or promise for payment.


B. A cheque cannot be considered as a bill of exchange.
C. If the time of payment is linked to the death of a person, it is nevertheless a
negotiable instrument.
D. It is freely transferable and delivery of the instrument is essential.

13. In which case did the Supreme Court place the Bangalore Water Supply and Sewerage
Board v. A. Rajappa before the Chief Justice of India for reconsideration by a larger
bench?

A. State of U.P v. Jai Bir Singh, 2005 (5) SCC 1.


B. Coir Board v. Indira Devi,[2000] 1 SCC 224.
C. Aeltemesh Rein v. Union of India, [1988] 4 SCC 54].

24
D. A.K. Roy v. Union of India, [1982] 1 SCC 271.

14. The legal phrase ‘pari passu’ means.

A. On equal footing.
B. Like angels do.
C. Repay without delay.
D. Return promptly.

15. When a person dies intestate

A. He dies in his home in his estate.


B. He dies without leaving any property.
C. He dies without leaving a will.
D. He dies by committing suicide.

16. A foreigner can acquire Indian citizenship through

A. Registration.
B. Naturalisation.
C. Birth.
D. Descent.

17. Which of the following writs can be used against a person believed to be holding a public
office, he is not entitled to hold?
A. Writ of mandamus.
B. Writ of certiorari.
C. Writ of prohibition.
D. Writ of quo warranto.

18. The principles of natural justice are applicable

A. only to judicial and quasi judicial proceedings and not to administrative


proceedings.

25
B. only to judicial, quasi judicial and administrative proceedings but not to
the law making procedure.
C. only to judicial proceedings.
D. to all judicial, quasi judicial, administrative and law making procedures.

19. What is the minimum number of judges required to constitute a Constitution Bench in the
Supreme Court of India?

A. Five.
B. Three.
C. Seven.
D. Nine.

20. According to the Hindu Marriage Act, a marriage is

A. An agreement.
B. A contract.
C. A sacrament.
D. A pious obligation.

21. Which one of the legal propositions is correct?

A. It is necessary for a contract to be valid that the consideration should be


adequate.
B. Inadequacy of consideration does not affect the validity of the
agreement.
C. Inadequacy of consideration renders the contract void ab initio.
D. An agreement made on account of natural love and affection, without
consideration is void.

22. A supplied to B, a minor, provisions such as wheat flour, rice and other food stuffs.
A. Since a minor's contract is void, A cannot recover the price of the provisions
from B.
B. A can recover the price from B as there is an agreement between A and B.
C. A is not entitled to recover the money as he voluntarily supplied the provisions
knowing that B was a minor.
D. Since the provisions supplied constituted necessaries, A can recover the
money.

26
23. A, from Kerala, writes a letter to B who lives in Delhi requesting the supply of certain goods
at a certain price. While accepting the offer by writing a letter on his printed letterhead B
writes at the top of the letterhead, "Subject to the jurisdiction of Delhi Courts." Later a
dispute arises and A files an action in Kerala.
A. A cannot file legal proceedings in Kerala since B has clearly indicated that
the jurisdiction is restricted to Delhi courts.
B. The agreement is void as it tends to defeat the legal process.
C. Since B has written the words at the top of his letterhead, they do not
become part of the contract and hence A is free to file the action in any
court which has jurisdiction.
D. Since the letterhead is part of the agreement of acceptance, the agreement
is enforceable.

24. A, a cultivator of green tea, agrees with B that he would not grow tea for a period of five
years in consideration of which B agrees to pay him Rs. 25,000/-
A. The agreement is valid.
B. The agreement is valid as the consideration is adequate.
C. The agreement is void as it is in restraint of trade.
D. The agreement is voidable at the option of A.

25. A entrusted his car to B for plying it as a taxi. B employed C as a cleaner. B gave the taxi to C
for taking a driving test and to obtain a driving licence. C while giving the driving test
seriously injured D. D filed a case against A, the owner of the vehicle.

A. A is not liable as he had neither permitted nor authorised C to drive the car.
B. A is liable because B was acting as his agent in giving the vehicle to C for taking it
for driving test.
C. A is liable because the accident happened during the course of employment.
D. A is liable because C was giving the driving test for the licence of A

26. A transport company issued a circular to all its drivers that they should not compete or
obstruct other buses plying in the route. Driver A, in contravention of the express
prohibition, obstructed another bus and while doing so met with an accident in which B was
injured.

A. The company is not liable as it had expressly prohibited the drivers from
competing and obstructing other buses.
B. The company is liable as the accident happened in the course of
employment.

27
C. The company is not liable because the driver had disobeyed the company's
instructions and thereby acted outside the course of employment.
D. The company is not liable because the driver had not acted for the benefit
of the employer, the company.

27. A was riding his bicycle on a rainy day on a road which had no street lights. He came in
contact with a live wire lying on the road which was inundated with water and got
electrocuted. A's wife brought an action against the Electricity Board for negligence.

A. The Board is not liable as the accident happened because of the negligence
of the cyclist.
B. The Board is not liable as the accident happened because of the negligence
of a third party, as the Board argued that a third party tried to siphon off
the electricity illegally, and negligently left the wire on the road.
C. The Board is not liable because of the rain and storm, and as such, it was an
Act of God.
D. The Board is liable as supplying electricity is an inherently dangerous act,
and as such, the Board is absolutely liable.

28. The Supreme Court in Indian Medical Association v. V P Shanta, (1995) 6 SCC 651, has held
that:

A. a hospital rendering free service to some patients and paid services to


others is not covered under the definition of service under the Consumer
Protection Act, as gratuitous services is excluded from the definition of
service under the Act.
B. a hospital which renders service to the patients is covered under the
Consumer Protection Act whether they are rendering a contract of service
or contract for service.
C. a hospital which is run by the government is not covered under the
Consumer Protection Act and the patient who suffers due to the negligence
of a doctor working in the hospital will have to file the case in a civil court
under the law of tort.
D. a hospital which renders free service to some patients and paid service to
some others is covered under the definition of service, as the patients
who receive free service are the beneficiaries of the service hired by the
patients who pay for the service.

29. Delegated Legislation is:

28
A. the law made by the Parliament or the State Legislature under delegated
authority from the Constitution.

B. the rule, regulations byelaws, etc., made by subordinate authorities


under delegated authority from the Parliament or State Legislature.

C. the Ordinance promulgated by the President or the Governor under


delegated authority from the Constitution.

D. the law declared by the judiciary while deciding cases brought before it.

30. The doctrine of 'Acting under Dictation' is applied in the case of:
A. Abuse of discretion
B. Failure to exercise discretion
C. Unreasonable exercise of discretion
D. Arbitrary exercise of discretion

31. Judicial Review deals with:


A. the supervision and monitoring of the activities of the officials of various
government departments.
B. the review by the judiciary of its own decisions.
C. the review by the judiciary of the administrative and legislative acts.
D. the review by the judiciary of the case laws with a view to apply in the case
at hand.

Directions: The question below consists of two statements, one labelled as ‘Assertion’ (A)
and another as ‘Reason’ (R). Examine these two statements carefully and select the
answers to these items from the codes given below.

32. Assertion (A): It is the legal and constitutional duty of the State to provide legal aid to poor.

Reason (R): No one should be denied justice by reason of his poverty.

Codes:

A. Both A and R are individually true and R is the correct explanation of A.


B. Both A and R are individually true but R is not the correct explanation of A.
C. A is true but R is false.
D. A is false but R is true.

29
33. A, a singer, enters into a contract with B, the manager of a theatre, to sing at his theatre
two nights every week during the following two months, and B agrees to pay Rupees 100/-
for each night's performance. On the sixth night A wilfully absents herself from the theatre.

A. B is at liberty to put an end to the contract.


B. B has to give notice to end the contract.
C. B cannot breach the contract.
D. It is A’s freedom to take leave, so B has to retain A

34. Under which provision of the Companies Act, 2013, will members be liable for a reduction of the
statutory minimum number of members, in the Company?

A. Sec. 3A
B. Sec. 7
C. Sec. 3
D. Sec. 4

35. Mohit lends Rs. One Lakh to Som where Narain was the surety. The contract in this
transaction provided that the liability of Narain is limited to Rs. 50,000/-. The contract is:

A. void

B. irregular

C. voidable

D. valid

36. The right of an accused to be defended by a legal practitioner of his/her choice commences
when:

A. the charge sheet is issued

B. the accused pleads not guilty

C. the accused is remanded

D. the accused is arrested

30
37. The transfer of 'spes successionis' under the Transfer of Property Act is

A. void ab initio
B. voidable
C. perfectly valid
D. valid only if approved by the court .

38. A mortgage by deposit of title deeds for which a loan of Rs 3 crores is raised

A. can be effected only with the help of a written deed


B. only with the help of a written and attested deed
C. can be effected only with the help of a written, attested and registered
document.
D. can be effected without an attested or registered document.

39. The rule of Daniels v Davison (1809) 16 Ves 249, has been given a statutory shape in the
Transfer of Property Act, 1882, under, which of the following titles?

A. Registration as constructive notice


B. Actual possession as constructive notice
C. Doctrine of animo attestendi
D. Doctrine of election

40. Under which of the following laws is the remedy of restitution of conjugal rights not
available?
A. Hindu Marriage Act, 1955
B. Indian Christian Marriage Act, 1872
C. Parsi Marriage and Divorce Act, 1936
D. Special Marriage Act, 1954

41. H and W, both professing Muslim faith marry under the Special Marriage Act, 1954. In
course of time they are blessed with two sons and two daughters. The eldest child, S1 was
very successful and made a lot of money while the rest of the family members were in dire

31
financial circumstances. He maintained a separate habitation, refused to share his money
and executed a Will of his total property in favour of a trust before his death at the age of
44 excluding completely his younger siblings and old parents. The Will would be
A. valid only to the extent of 1/3rd of his total property.
B. void as 2/3rd of his property must go by intestate succession.
C. valid as he can make a Will of his total property.
D. valid as he died a bachelor .

42. A 35 years old Hindu bachelor (single man) adopts a 14 years old girl under the Hindu
Adoptions and Maintenance Act, 1956. Such adoption under the law is:

A. valid
B. void
C. voidable
D. irregular.

43. An “International Application” under the Patent Co-operation Treaty:

A. is examined by a patent examiner at a designated office.


B. is searched against by a single search conducted at a designated office.
C. results in a single patent covering all the States for which protection is
sought.
D. is forwarded to the patent offices of all the States for which protection is
sought for search and examination.

44. The first of the major laws for the protection of environment, to be promulgated in India
was the:

A. Water Act.
B. Air Act.
C. Environmental Act.
D. Noise Pollution Rule.

45. What does the term ISSN displayed on Journals stand for?

A. International Sequence of Standard Numbers

32
B. International Standard Serial Number
C. Index of Standard Serial Numbers
D. Indexed Standard and Sequential Numbers

46. The fundamental principle of non-refoulement in International Law is applied in relation to:
A. Terrorists

B. Drug traffickers

C. Smugglers

D. Refugees

47. The last trial by jury held in a criminal case in India was that of
A. Rustom v. State of Gujarat
B. Ascentia Dawes v. Crown
C. Lily v. State of Bombay
D. K.M. Nanavati v. State of Maharashtra

48. The Right to Education Act 2009 (RTE) provides for free and compulsory education to:

A All illiterate children of India


B All citizens of India
C Children aged between 6 and 14 years
D All children up to the age of 10 years

49. An International treaty becomes binding on India, when:

A The Prime minister of India signs the treaty


B The President of India signs the treaty
C The Parliament of India makes special legislation for accepting the treaty
D When the Indian delegation signs the treaty

33
50. The Supreme Court of India recognised ‘transgenders’ as ‘third gender with all rights’ in the
case of:

A Bar Council of India v. A.K. Balaji, AIR 2018 SC 1382


B National Legal Services Authority v. Union of India, AIR 2014 SC 1863
C Suresh Kumar Koushal v. NAZ Foundation, AIR 2014 SC 563
D State of Punjab v. Jagjit Singh, AIR 2016 SC 5176

34

You might also like